Logo Passei Direto
Material

This is a file preview. Join to view the original file.

<p>PAU PORTUGAL</p><p>Matematica B (735) 2006-2020</p><p>Gerard Romo Garrido</p><p>Toomates Coolección vol. 32</p><p>Toomates</p><p>Los libros de Toomates son materiales digitales y gratuitos. Son digitales porque están pensados para ser consultados mediante un ordenador, tablet o móvil. Son</p><p>gratuitos porque se ofrecen a la comunidad educativa sin coste alguno. Los libros de texto pueden ser digitales o en papel, gratuitos o en venta, y ninguna de estas</p><p>opciones es necesariamente mejor o peor que las otras. Es más: Suele suceder que los mejores docentes son los que piden a sus alumnos la compra de un libro de</p><p>texto en papel, esto es un hecho. Lo que no es aceptable, por inmoral y mezquino, es el modelo de las llamadas "licencias digitales" con las que las editoriales</p><p>pretenden cobrar a los estudiantes, una y otra vez, por acceder a los mismos contenidos (unos contenidos que, además, son de una bajísima calidad). Este modelo</p><p>de negocio es miserable, pues impide el compartir un mismo libro, incluso entre dos hermanos, pretende convertir a los estudiantes en un mercado cautivo, exige a</p><p>los estudiantes y a las escuelas costosísimas líneas de Internet, pretende pervertir el conocimiento, que es algo social, público, convirtiéndolo en un producto de</p><p>propiedad privada, accesible solo a aquellos que se lo puedan permitir, y solo de una manera encapsulada, fragmentada, impidiendo el derecho del alumno de</p><p>poseer todo el libro, de acceder a todo el libro, de moverse libremente por todo el libro.</p><p>Nadie puede pretender ser neutral ante esto: Mirar para otro lado y aceptar el modelo de licencias digitales es admitir un mundo más injusto, es participar en la</p><p>denegación del acceso al conocimiento a aquellos que no disponen de medios económicos, y esto en un mundo en el que las modernas tecnologías actuales</p><p>permiten, por primera vez en la historia de la Humanidad, poder compartir el conocimiento sin coste alguno, con algo tan simple como es un archivo "pdf". El</p><p>conocimiento no es una mercancía.</p><p>El proyecto Toomates tiene como objetivo la promoción y difusión entre el profesorado y el colectivo de estudiantes de unos materiales didácticos libres, gratuitos</p><p>y de calidad, que fuerce a las editoriales a competir ofreciendo alternativas de pago atractivas aumentando la calidad de unos libros de texto que actualmente son</p><p>muy mediocres, y no mediante retorcidas técnicas comerciales.</p><p>Estos libros se comparten bajo una licencia “Creative Commons 4.0 (Atribution Non Commercial)”: Se permite, se promueve y se fomenta cualquier uso,</p><p>reproducción y edición de todos estos materiales siempre que sea sin ánimo de lucro y se cite su procedencia. Todos los libros se ofrecen en dos versiones: En</p><p>formato “pdf” para una cómoda lectura y en el formato “doc” de MSWord para permitir y facilitar su edición y generar versiones parcial o totalmente</p><p>modificadas. ¡Libérate de la tiranía y mediocridad de las editoriales! Crea, utiliza y comparte tus propios materiales didácticos</p><p>Problem Solving (en español):</p><p>Geometría Axiomática Problemas de Geometría 1 Problemas de Geometría 2 Introducción a la Geometría</p><p>Álgebra Teoría de números Combinatoria Probabilidad Trigonometría Desigualdades</p><p>Números complejos Funciones</p><p>Libros de texto (en catalán):</p><p>Nombres (Preàlgebra) Àlgebra Proporcionalitat Mesures geomètriques Geometria analítica</p><p>Combinatòria i Probabilitat Estadística Trigonometria Funcions Nombres Complexos</p><p>Àlgebra Lineal Geometria Lineal Càlcul Infinitesimal Programació Lineal Mates amb Excel</p><p>PAU españolas:</p><p>Cataluña TEC Cataluña CCSS Valencia Galicia País Vasco Baleares</p><p>PAU internacionales:</p><p>Portugal Italia Francia Pearson Edexcel International A Level Cambridge International A Level</p><p>International Baccalaureate (IB)</p><p>Pruebas de acceso:</p><p>ACM4 CFGS PAP</p><p>Competiciones matemáticas:</p><p>Canguro: España Cataluña Francia USA Reino Unido Austria</p><p>USA: Mathcounts AMC 8 10 12 AIME USAJMO USAMO TSTST TST ELMO Putnam</p><p>España: OME OMEFL OMEC OMEA OMEM CDP</p><p>Europa: OMI Arquimede HMMT BMO Balkan MO JBMO</p><p>Internacional: IMO IGO SMT INMO CMO HMMT</p><p>AHSME: Book 1 Book 2 Book 3 Book 4 Book 5 Book 6 Book 7 Book 8 Book 9</p><p>Otros materiales:</p><p>Pizzazz!: Book A Book B Book C Book D Book E Pre-Algebra Algebra , REOIM</p><p>¡Genera tus propias versiones de este documento! Siempre que es posible se ofrecen las versiones editables “MS Word” de todos</p><p>los materiales para facilitar su edición.</p><p>¡Ayuda a mejorar! Envía cualquier duda, observación, comentario o sugerencia a toomates@gmail.com</p><p>¡No utilices una versión anticuada! Todos estos libros se revisan y amplían constantemente. Descarga totalmente gratis la última</p><p>versión de estos documentos en los correspondientes enlaces superiores, en los que siempre encontrarás la versión más</p><p>actualizada.</p><p>Consulta el catálogo de libros completo en http://www.toomates.net</p><p>Descarga toda la biblioteca Toomates en un solo archivo Aquí</p><p>Visita mi Canal de Youtube: https://www.youtube.com/c/GerardRomo</p><p>Visita mi blog: https://toomatesbloc.blogspot.com/</p><p>Versión de este documento: 30/08/2024</p><p>http://www.toomates.net/biblioteca/GeometriaAxiomatica.pdf</p><p>http://www.toomates.net/biblioteca/ProblemasGeometria.pdf</p><p>http://www.toomates.net/biblioteca/ProblemasGeometria2.pdf</p><p>http://www.toomates.net/biblioteca/Geometria.pdf</p><p>http://www.toomates.net/biblioteca/ProblemasAlgebra.pdf</p><p>http://www.toomates.net/biblioteca/Aritmetica.pdf</p><p>http://www.toomates.net/biblioteca/Combinatoria.pdf</p><p>http://www.toomates.net/biblioteca/Probabilidad.pdf</p><p>http://www.toomates.net/biblioteca/ProblemasTrigonometria.pdf</p><p>http://www.toomates.net/biblioteca/Desigualdades.pdf</p><p>http://www.toomates.net/biblioteca/ProblemasNumerosComplejos.pdf</p><p>http://www.toomates.net/biblioteca/ProblemasFunciones.pdf</p><p>http://www.toomates.net/biblioteca/Nombres.pdf</p><p>http://www.toomates.net/biblioteca/Algebra.pdf</p><p>http://www.toomates.net/biblioteca/Proporcionalitat.pdf</p><p>http://www.toomates.net/biblioteca/MesuresGeometriques.pdf</p><p>http://www.toomates.net/biblioteca/GeometriaAnalitica.pdf</p><p>http://www.toomates.net/biblioteca/CombinatoriaProbabilitat.pdf</p><p>http://www.toomates.net/biblioteca/Estadistica.pdf</p><p>http://www.toomates.net/biblioteca/Trigonometria.pdf</p><p>http://www.toomates.net/biblioteca/Funcions.pdf</p><p>http://www.toomates.net/biblioteca/NombresComplexos.pdf</p><p>http://www.toomates.net/biblioteca/AlgebraLineal.pdf</p><p>http://www.toomates.net/biblioteca/GeometriaLineal.pdf</p><p>http://www.toomates.net/biblioteca/Calcul.pdf</p><p>http://www.toomates.net/biblioteca/ProgramacioLineal.pdf</p><p>http://www.toomates.net/biblioteca/MatesExcel.pdf</p><p>http://www.toomates.net/biblioteca/Pautec.pdf</p><p>http://www.toomates.net/biblioteca/Pauccss.pdf</p><p>http://www.toomates.net/biblioteca/Valencia.pdf</p><p>http://www.toomates.net/biblioteca/Galiciapau.pdf</p><p>http://www.toomates.net/biblioteca/Paisvascopau.pdf</p><p>http://www.toomates.net/biblioteca/Balears.pdf</p><p>http://www.toomates.net/biblioteca/Portugal.pdf</p><p>http://www.toomates.net/biblioteca/Italia.pdf</p><p>http://www.toomates.net/biblioteca/Francia.pdf</p><p>http://www.toomates.net/biblioteca/Edexcel.pdf</p><p>http://www.toomates.net/biblioteca/Cambridge.pdf</p><p>http://www.toomates.net/biblioteca/IB.pdf</p><p>http://www.toomates.net/biblioteca/CompendiumACM4.pdf</p><p>http://www.toomates.net/biblioteca/CompendiumCFGS.pdf</p><p>http://www.toomates.net/biblioteca/CompendiumPAP.pdf</p><p>http://www.toomates.net/biblioteca/Canguro.pdf</p><p>http://www.toomates.net/biblioteca/Cangur.pdf</p><p>http://www.toomates.net/biblioteca/CompendiumKangourou.pdf</p><p>http://www.toomates.net/biblioteca/CompendiumKangaroo.pdf</p><p>http://www.toomates.net/biblioteca/CompendiumKangarooUK.pdf</p><p>http://www.toomates.net/biblioteca/CompendiumKanguru.pdf</p><p>http://www.toomates.net/biblioteca/CompendiumMathcounts.pdf</p><p>http://www.toomates.net/biblioteca/CompendiumAMC8.pdf</p><p>http://www.toomates.net/biblioteca/CompendiumAMC10.pdf</p>
<p>níveis de desempenho, o</p><p>classificador deve enquadrar a resposta do examinando numa das descrições</p><p>apresentadas. O classificador não pode atribuir uma cotação diferente das indicadas.</p><p>6.4. No caso de o examinando cometer um erro numa das etapas, as etapas subsequentes</p><p>devem merecer a respectiva cotação, desde que o grau de dificuldade não tenha</p><p>diminuído, e o examinando as execute correctamente, de acordo com o erro que</p><p>cometeu.</p><p>6.5. Caso o examinando cometa, numa etapa, um erro que diminua o grau de dificuldade</p><p>das etapas subsequentes, cabe ao classificador decidir a cotação máxima a atribuir a</p><p>cada uma destas etapas. Em particular, se, devido a um erro cometido pelo</p><p>examinando, o grau de dificuldade das etapas seguintes diminuir significativamente, a</p><p>cotação máxima a atribuir a cada uma delas não deverá exceder metade da cotação</p><p>indicada.</p><p>6.6. Pode acontecer que o examinando, ao resolver um item, não percorra explicitamente</p><p>todas as etapas previstas nos critérios específicos. Todas as etapas não percorridas</p><p>explicitamente pelo examinando, mas cuja utilização e/ou conhecimento estejam</p><p>inequivocamente implícitos na resolução do item, devem receber a cotação indicada.</p><p>7. Existem, por vezes, etapas em que está previsto o recurso à calculadora. Nessas etapas, os</p><p>critérios específicos subdividem-se em: «Explicação do método utilizado» e «Apresentação</p><p>do(s) valor(es)».</p><p>7.1. Explicação do método utilizado:</p><p>De acordo com as instruções gerais para a realização da prova, o examinando deve</p><p>apresentar todos os elementos recolhidos na utilização da calculadora. Esta</p><p>apresentação deve ser cotada de acordo com o critério que se segue, no qual, para</p><p>cada nível de desempenho, é indicada uma percentagem. Esta percentagem deve ser</p><p>aplicada sobre a cotação prevista para a explicação do método utilizado, e o valor</p><p>obtido deve ser arredondado às unidades (por excesso, se a mantissa do número a</p><p>arredondar for 0,5 ou superior).</p><p>Apresentação correcta e completa de todos os elementos relevantes....................100%</p><p>Apresentação correcta, mas com ausência de alguns elementos</p><p>relevantes</p><p>ou</p><p>Apresentação completa, mas com algumas incorrecções (por exemplo,</p><p>não respeitar o domínio de uma função) .................................................................. 60%</p><p>Apresentação incompleta e com algumas incorrecções .......................................... 20%</p><p>Ausência de explicação ou simples referências do tipo «Vi na</p><p>calculadora»................................................................................................................. 0 %</p><p>7.2. Apresentação do(s) valor(es):</p><p>Para cada valor que o examinando deve apresentar, os critérios específicos podem</p><p>indicar um intervalo admissível. O valor apresentado pelo examinando pode pertencer,</p><p>ou não, a esse intervalo.</p><p>• Se o valor pertencer ao intervalo, deve ser atribuída a cotação máxima prevista</p><p>para essa apresentação, a menos de qualquer penalização prevista nos critérios</p><p>específicos, por desrespeito relativo ao número de casas decimais com que o</p><p>resultado deve ser apresentado.</p><p>• Se o valor não pertencer ao intervalo, deve ser atribuída a cotação de 0 (zero)</p><p>pontos.</p><p>735/C/4</p><p>8. Quando, num item, é pedida uma forma específica de apresentação do resultado final (por</p><p>exemplo, «em minutos», «em percentagem», etc.), este deve ser apresentado na forma</p><p>pedida. Se o resultado final apresentado pelo examinando não respeitar a forma pedida no</p><p>enunciado (por exemplo, se o enunciado pedir o resultado em minutos, e o examinando o</p><p>apresentar em horas), devem ser atribuídos 0 (zero) pontos à etapa correspondente ao</p><p>resultado final. No entanto, o examinando não deve ser penalizado se não indicar a unidade</p><p>em que é pedido o resultado (por exemplo, se o resultado final for 12 minutos, ou 12 metros,</p><p>e o examinando escrever simplesmente 12, não deve ser penalizado).</p><p>9. O examinando deve respeitar sempre a instrução relativa à apresentação de todos os</p><p>cálculos e de todas as justificações. Se, numa etapa, o examinando não respeitar esta</p><p>instrução, apresentando algo (valor, quadro, tabela, gráfico, etc.) que não resulte de trabalho</p><p>anterior, deve ser atribuída a cotação de 0 (zero) pontos a essa etapa. Todas as etapas</p><p>subsequentes que dela dependam devem ser igualmente cotadas com 0 (zero) pontos.</p><p>10. O examinando deve respeitar sempre qualquer instrução relativa ao método a utilizar na</p><p>resolução de um item (por exemplo, «equacione o problema», «resolva graficamente», etc.).</p><p>Na resolução apresentada pelo examinando, deve ser inequívoco, pela apresentação de</p><p>todos os cálculos e de todas as justificações, o cumprimento da instrução. Se tal não</p><p>acontecer, considera-se que o examinando não respeitou a instrução. A etapa em que se dá</p><p>o desrespeito e todas as subsequentes que dela dependam devem ser cotadas com 0 (zero)</p><p>pontos.</p><p>11. Se, na resolução de um item, o examinando utilizar simbologia, ou escrever uma expressão,</p><p>inequivocamente incorrecta do ponto de vista formal (por exemplo, se escrever o símbolo de</p><p>igualdade onde deveria estar o símbolo de equivalência), deve ser penalizado em um ponto,</p><p>na cotação total a atribuir a esse item. Esta penalização não se aplica no caso em que tais</p><p>incorrecções ocorram apenas em etapas cotadas com 0 (zero) pontos, nem a eventuais</p><p>utilizações do símbolo de igualdade, onde, em rigor, deveria estar o símbolo de igualdade</p><p>aproximada.</p><p>12. Existem itens em cujo enunciado é dada uma instrução relativa ao número mínimo de casas</p><p>decimais que o examinando deve conservar, sempre que, em cálculos intermédios, proceder</p><p>a arredondamentos. Indicam-se, a seguir, as penalizações a aplicar, na cotação total a</p><p>atribuir ao item, em caso de desrespeito dessa instrução e/ou de arredondamentos mal</p><p>efectuados.</p><p>Todos os valores intermédios estão de acordo com a instrução, mas existe,</p><p>pelo menos, um valor intermédio mal arredondado...................................................... -1 ponto</p><p>Todos os valores intermédios estão bem arredondados, mas existe, pelo</p><p>menos, um que não está de acordo com a instrução................................................... -1 ponto</p><p>Existe, pelo menos, um valor intermédio mal arredondado e existe, pelo</p><p>menos, um que não está de acordo com a instrução ................................................ -2 pontos</p><p>V.S.F.F.735/C/5</p><p>Critérios específicos</p><p>1.1.1. .................................................................................................................. 8</p><p>Determinar a média e o desvio padrão das classificações na</p><p>disciplina de Matemática ....................................................................................... 4</p><p>Explicação do método utilizado (ver critério geral 7.1.) ..................2</p><p>B œ ") ........................................................................................... 1</p><p>5 ¸ " #, .......................................................................................... 1</p><p>Determinar a média e o desvio padrão das classificações na</p><p>disciplina de Informática ........................................................................................ 4</p><p>Explicação do método utilizado (ver critério geral 7.1.) ..................2</p><p>B œ ") ........................................................................................... 1</p><p>5 ¸ " ', .......................................................................................... 1</p><p>1.1.2. ................................................................................................................. 7</p><p>Apresenta-se a seguir um exemplo de resposta:</p><p>Observando os dois gráficos, o Pedro verificou que, na disciplina de Matemática,</p><p>as classificações estão mais concentradas em torno da média do que na</p><p>disciplina de Informática. O Pedro concluiu, assim, que os desvios padrão eram</p><p>diferentes.</p><p>1.2. .................................................................................................................. 10</p><p>Concluir que o número de casos favoráveis é ................................................ 4 (</p><p>Concluir</p>
<p>que o número de casos possíveis é ................................................4 "'</p><p>Determinar a probabilidade pedida ( )........................................2 Œ !(</p><p>"' ver nota</p><p>Nota:Se o examinando não apresentar o resultado na forma de fracção irredutível, a</p><p>cotação a atribuir à sua resposta deve ser desvalorizada em 1 ponto.</p><p>735/C/6</p><p>2.1.1. ............................................................................................................... 12</p><p>Determinar o número de páginas que a Ana leu ao fim de dias...................... 6 8</p><p>Reconhecer que a situação se traduz por uma progressão</p><p>geométrica de razão 2 e primeiro termo 1 .........................................2</p><p>Escrever a expressão que dá o número de páginas que a</p><p>Ana leu ao fim de dias ...................................... 3 8 " ‚Š ‹"##</p><p>"##</p><p>8</p><p>Simplificar a expressão anterior......................................................... 1</p><p>Determinar o número de páginas que a Fátima leu ao fim de dias .................6 8</p><p>Reconhecer que a situação se traduz por uma progressão</p><p>aritmética de razão 2 e primeiro termo 3 ........................................... 2</p><p>Escrever a expressão que dá o número de páginas que a</p><p>Fátima leu no n-ésimo dia ............................... 1 " #$ + Ð8 - "Ñ ‚ #</p><p>$ + Ð8 - "Ñ ‚ # œ " + #8 ............................................................ 1</p><p>Escrever a expressão que dá o número de páginas que a</p><p>Fátima leu ao fim de dias ......................... 1 8 ‚ 8Š ‹$&"&#8</p><p>#</p><p>Simplificar a expressão anterior......................................................... 1</p><p>2.1.2. ............................................................................................................... 10</p><p>Determinar o número de dias que a Ana demora a ler o livro ............................. 4</p><p>Equacionar o problema ( ) ....................................2 # - " œ #&&8</p><p>8 œ ) ......................................................................................... 2</p><p>Determinar o número de dias que a Fátima demora a ler o livro ......................... 4</p><p>Equacionar o problema ( ) ................................ 2 8 + #8 œ #&&#</p><p>8 œ "& ....................................................................................... 2</p><p>Resposta à questão colocada 25 de Abril ........................................................ 2 " #</p><p>V.S.F.F.735/C/7</p><p>2.2. .................................................................................................................. 10</p><p>Concluir que o número de casos favoráveis é .................... 6 "! + &' (ver nota)</p><p>Escrever a fracção ................................................................................... 2 ''</p><p>#&&</p><p>Apresentar a probabilidade pedida na forma de percentagem % .............2 " ##'</p><p>Nota:Se o examinando considerar que o número de casos favoráveis é , a* + &&</p><p>cotação a atribuir a esta etapa deve ser de 4 pontos.</p><p>3.1. .................................................................................................................. 15</p><p>Substituir, na expressão , a variável por ..............................................7 R > > !" #</p><p>R ! œ" # "#&E</p><p>E& "#&#E� � .................................................................................... 3</p><p>R ! œ" # "#&E</p><p>"#& ................................................................................................3</p><p>R ! œ E" # ......................................................................................................... 2</p><p>3.2. .................................................................................................................. 15</p><p>Equacionar o problema ................................. 8 Œ !"#&E</p><p>E& "#&#E /� � !" œ E+ #$</p><p>Resolver a equação ..............................................................................................7</p><p>Explicação do método (ver critério geral 7.1.) ....................................... 4</p><p>E ¸ #" ...............................................................................3 (ver nota)</p><p>Nota:</p><p>Se o examinando apresentar a resposta , a cotação a atribuir a estaE ¸ #!</p><p>etapa deve ser de 2 pontos.</p><p>735/C/8</p><p>4.1.1. .................................................................................................................. 5</p><p>A cotação a atribuir à resposta do examinando deve estar de acordo com os</p><p>seguintes níveis de desempenho:</p><p>Apresentar o intervalo correcto ............................................................. 5 " #Ó ! "Ò,</p><p>Apresentar um dos seguintes intervalos: ou ou .................... 3 Ò! "Ó Ò! "Ò Ó! "Ó, , ,</p><p>Apresentar o intervalo ................................................................................ 1 Ó ! #Ò,</p><p>Outras situações ....................................................................................................0</p><p>4.1.2. ............................................................................................................... 15</p><p>Volume da esfera ................................................................................1 œ B%</p><p>$ 1 $</p><p>Aresta do cubo .................................................................................. 2 œ # - #B</p><p>Volume do cubo ........................................................................... 1 œ # - # B" #$" # " # " ## - #B œ # - #B ‚ # - #B$ # ..............................................................3 " ## - #B œ % - )B + %B# #.............................................................................3 " # " #% - )B + %B ‚ # - #B œ ) - #%B + #%B - )B# # $ ........................... 3</p><p>Z ÐBÑ œ %</p><p>$ 1 B + ) - #%B + #%B - )B$ # $ ..................................................... 1</p><p>Conclusão ..............................................................................................................1</p><p>4.1.3. ............................................................................................................... 10</p><p>Determinar o valor de (raio da esfera) para o qual o volume total daB</p><p>escultura é mínimo ( ).................................................................8 " #! &), ver nota</p><p>Determinar a aresta do cubo correspondente .......................................... 2 " #! )%,</p><p>Nota:</p><p>O examinando pode encontrar o minimizante de graficamente ouZ</p><p>analiticamente.</p><p>Se o examinando utilizar a via gráfica, com recurso à calculadora, a cotação</p><p>desta etapa deve ser repartida da seguinte forma:</p><p>Explicação do método utilizado (ver critério geral 7.1.) ................5</p><p>Indicar o minimizante.......................................................................3</p><p>O intervalo admissível para o minimizante</p><p>é (ver critério geral 7.2.)Ò ! &( ! &*Ó, ; ,</p><p>Se o examinando utilizar a via analítica, a cotação desta etapa deve ser</p><p>repartida da seguinte forma:</p><p>Determinar a taxa de variação da função .......................................4</p><p>Determinar o minimizante................................................................4</p><p>V.S.F.F.735/C/9</p><p>4.2. .................................................................................................................. 13</p><p>Concluir que o raio da esfera é .....................................................................2 "</p><p>#</p><p>Concluir que a aresta do cubo é ..................................................................... 2 "</p><p>Determinar a área total da escultura, excluindo a face do</p><p>cubo que está assente no chão ............................................................................ 6</p><p>Área da superfície esférica ............................................... 2 " #1</p><p>Área de uma face do cubo ................................................ 1 " #"</p><p>Área de cinco faces ...........................................................2 " #&</p><p>Área total ...................................................................1 " #& + 1</p><p>Responder à questão colocada ............................................................................ 3</p><p>&&</p><p># &</p><p>1</p><p>,</p><p>¸ $ #&(, ......................................................................</p>
<p>1</p><p>Conclusão (4 latas) ....................................................................2</p><p>5.1. .................................................................................................................. 15</p><p>Se o examinando utilizar a via analítica para resolver esta questão, a</p><p>cotação deve ser repartida da seguinte forma:</p><p>Concluir que é maximizante de .................................................4 1 . (ver nota)</p><p>Determinar a distância máxima ..........................................................4 ˆ ‰"&# ",</p><p>Concluir que é minimizante de ................................................ 3 ! . (ver nota)</p><p>Determinar a distância mínima ............................................................4 ˆ ‰"%( ",</p><p>ou</p><p>- " Ÿ -9= B Ÿ " ..............................................................................................3</p><p>! !"'( � - ! !"'( -9= B � - ! !"'(, , , .........................................................3</p><p>" !"'( � " - ! !"'( -9= B � ! *)$$, , , ............................................................3</p><p>"&# " � "%* ' " - ! !"'( -9= B � "%( ", , , , " # ................................................3</p><p>Conclusão ........................................................................................................... 3</p><p>Se o examinando utilizar a via gráfica, com recurso à calculadora, a cotação</p><p>deve ser repartida da seguinte forma:</p><p>Explicação do método utilizado (ver critério geral 7.1.) ..................5</p><p>Indicar o máximo............................................................................. 5</p><p>O intervalo admissível é Ò "&# "&# #Ó; ,</p><p>(ver critério geral 7.2.)</p><p>Indicar o mínimo.............................................................................. 5</p><p>O intervalo admissível é Ò "%( "%( #Ó; ,</p><p>(ver critério geral 7.2.)</p><p>Nota:</p><p>Não se exige que o examinando utilize a relação entre zeros/sinal da taxa de</p><p>variação e extremos/monotonia da função. O examinando pode obter o</p><p>maximizante e o minimizante por observação directa da figura do enunciado.</p><p>735/C/10</p><p>5.2.1. ............................................................................................................... 15</p><p>Mostrar que, para , se tem .........................................................9 B œ > œ1 X</p><p>#</p><p>Substituir por .................................................................... 2 B 1</p><p>Obter a igualdade ............................................... 3 # ></p><p>X</p><p>1 œ 1</p><p>Mostrar que .................................4 # > X</p><p>X #</p><p>1 œ Í > œ1</p><p>Interpretar o resultado ..........................................................................................6</p><p>A cotação a atribuir deve estar de acordo com os</p><p>seguintes níveis de desempenho:</p><p>Interpretação correcta (exemplo: «O tempo que decorre</p><p>entre a passagem da Terra pelo periélio e o instante em</p><p>que a Terra atinge o ponto mais afastado da sua órbita,</p><p>relativamente ao Sol, é metade do tempo que a Terra</p><p>demora a descrever uma órbita completa.») ............................. 6</p><p>Interpretação mal redigida, mas onde existe evidência</p><p>de o examinando ter compreendido a situação</p><p>(exemplos: «Quando o , demora-se metade dosB œ 1</p><p>dias a fazer uma volta completa de órbita. Quando»; «</p><p>B œ 1 percorre-se metade da distância, demorando-se</p><p>metade do tempo. » ) .................................................................. 3</p><p>Outras situações (exemplo: «O tempo para se chegar</p><p>ao periélio é metade do tempo que demora a alcançar o</p><p>Sol.») .......................................................................................... 0</p><p>735/C/11</p><p>5.2.2. ............................................................................................................... 15</p><p>Determinar o número de dias que decorrem entre 4 de Janeiro e 14</p><p>de Fevereiro ( ) .........................................................................................2 ver nota</p><p>Escrever a equação ......................................4 # ‚%"</p><p>$'& #%</p><p>1</p><p>,</p><p>œ B - ! !"'( =/8B,</p><p>Resolver a equação .............................................................................................. 6</p><p>Explicação do método utilizado (ver critério geral 7.1.) ..................3</p><p>Valor de ....................................................................................... 3 B</p><p>O intervalo admissível é Ò Ó! ("'# ! ("'$, ,;</p><p>(ver critério geral 7.2.)</p><p>Determinar a distância pedida .............................................................................. 3</p><p>Substituir, na expressão ,"%* ' Ð" - ! !"'( BÑ, , cos</p><p>B pelo valor encontrado ................................................................2</p><p>Resultado correctamente arredondado .........................................1</p><p>Nota:A cotação desta etapa deve ser atribuída de acordo com o seguinte critério:</p><p>Número de dias correcto (41) ........................................................2</p><p>Número de dias igual a 40 ou a 42 ................................................1</p><p>Outras situações .............................................................................0</p><p>6. ..................................................................................................................... 25</p><p>Determinar a taxa de variação média da temperatura da água, no</p><p>intervalo ................................................................................3 Ò # $ &Ó - # $; , , " #</p><p>Determinar a taxa de variação média da temperatura da água, no</p><p>intervalo .................................................................................. 3 Ò # $ Ó - # %; , " #</p><p>Determinar a taxa de variação média da temperatura da água, no</p><p>intervalo ................................................................................3 Ò # # &Ó - # %; , , " #</p><p>X Ð # Ñ ¸ - # % w , ..................................................................................................4</p><p>Equacionar o problema ............................................ 9 " #" #- # % œ 5 )& - #&,</p><p>Determinar o valor de .....................................................................3 5 - ! !% ," #</p><p>PROPOSTA DE RESOLUÇÃO DO EXAME NACIONAL DE MATEMÁTICA B</p><p>12oAno – Prova 735 – 2aFase - 2006</p><p>(Esta proposta de correcção também pode ser consultada em www.apm.pt)</p><p>1. a</p><p>aa</p><p>1.1 a</p><p>1.1.1 a</p><p>Nº. de alunos</p><p>Classificação Matemática Informática Matemática Informática</p><p>16</p><p>17</p><p>18</p><p>19</p><p>20</p><p>6</p><p>11</p><p>17</p><p>9</p><p>7</p><p>13</p><p>11</p><p>3</p><p>9</p><p>14</p><p>Total 50 50</p><p>18x = 18x =</p><p>σ = 1,2 σ = 1,6</p><p>Confirma-se que as médias das classificações às duas disciplinas são iguais e os desvios padrão</p><p>são diferentes.</p><p>1.1.2 Em Matemática a maioria dos alunos tem classificação igual ao valor médio (18) ou próximo</p><p>deste (17 ou 19), enquanto que em Informática se verifica que a maioria das classificações são</p><p>mais afastadas do valor médio (16 ou 20). Logo, o Pedro concluiu que o desvio padrão das</p><p>classificações em Informática é maior.</p><p>1.2. Dos 14 alunos que obtiveram 20 a Informática, 7 obtiveram, também, 20 a Matemática.</p><p>Há 16 (9 + 7) alunos com classificação maior ou igual a 19 valores na disciplina de Matemática.</p><p>Escolhendo um destes alunos ao acaso, a probabilidade de ter 20 nas duas disciplinas é então</p><p>p =</p><p>7</p><p>16</p><p>.</p><p>2 aa</p><p>2.1 aa</p><p>2.1.1 Número de páginas lidas pela Ana no dia n:</p><p>n</p><p>n</p><p>n</p><p>a</p><p>1</p><p>2</p><p>4</p><p>8</p><p>16</p><p>1</p><p>2</p><p>3</p><p>4</p><p>5</p><p>… …</p><p>2</p><p>n−1</p><p>×2 A sucessão (an) é uma progressão geométrica de razão 2, pelo que a soma</p><p>dos n primeiros termos é dada pela expressão:</p><p>Sn = 1 · 1 − 2n</p><p>1 − 2</p><p>=</p><p>1 − 2n</p><p>−1</p><p>= 2n − 1.</p><p>Esta expressão representa o número de páginas que a Ana já leu ao fim de</p><p>n dias.</p><p>1</p><p>Número de páginas lidas pela Fátima no dia n:</p><p>n</p><p>n</p><p>3</p><p>5</p><p>7</p><p>9</p><p>11</p><p>1</p><p>2</p><p>3</p><p>4</p><p>5</p><p>… …</p><p>+2</p><p>fn</p><p>2n + 1</p><p>A sucessão (fn) é uma progressão aritmética de razão 2, pelo que a soma</p><p>dos n primeiros termos, número de páginas lidas pela Fátima ao fim de n</p><p>dias, é dada pela expressão:</p><p>Sn =</p><p>3 + 2n + 1</p><p>2</p><p>× n =</p><p>4 + 2n</p><p>2</p><p>× n = (2 + n)n = 2n + n2.</p><p>2.1.2 a</p><p>1</p><p>2</p><p>3</p><p>4</p><p>8</p><p>15</p><p>…</p><p>…</p><p>1</p><p>2</p><p>4</p><p>8</p><p>128</p><p>…</p><p>1</p><p>3</p><p>7</p><p>15</p><p>255</p><p>…</p><p>3</p><p>5</p><p>7</p><p>9</p>
<p>17</p><p>31</p><p>…</p><p>…</p><p>3</p><p>8</p><p>15</p><p>24</p><p>80</p><p>255</p><p>…</p><p>…</p><p>n an Σan fn</p><p>Σfn</p><p>A Ana demorou 8 dias a ler o livro; a Fátima demorou 15 dias (mais 7 dias do que a Ana).</p><p>Assim, como a Ana acabou a 18 de Abril, a Fátima terá terminado no dia 25 de Abril (18 + 7).</p><p>2.2 Número de páginas em que o número começa pelo algarismo 2:</p><p>2 0</p><p>2 1</p><p>...</p><p>...</p><p>2 9</p><p> 10 páginas</p><p>2 0 0</p><p>2 0 1</p><p>...</p><p>...</p><p>2 5 5</p><p> 56 páginas</p><p>Existem, então, 66 páginas nas condições pretendidas. A probabilidade é</p><p>p =</p><p>66</p><p>255</p><p>≈ 0, 26.</p><p>R.: A probabilidade pedida é 26%.</p><p>3 a</p><p>3.1 a</p><p>3.1.1 aa</p><p>N(0) =</p><p>125A</p><p>A + (125 − A)e−0,2×0</p><p>⇔ N(0) =</p><p>125A</p><p>A + (125 − A) × 1</p><p>⇔ N(0) =</p><p>125A</p><p>125</p><p>⇔ N(0) = A.</p><p>Verifica-se, assim, que o número de aves existente no instante inicial é A.</p><p>3.2 Ao fim de 5 anos existem mais 23 (80 − 57) aves do que no instante inicial. Assim, N(5) = A + 23,</p><p>ou seja,</p><p>125A</p><p>A + (125 − A)e−0,2×5</p><p>= A + 23 ⇔ 125A</p><p>A + (125 − A)e−1</p><p>= A + 23.</p><p>2</p><p>Inserindo no editor de funções da calculadora as funções</p><p>Y1 =</p><p>125x</p><p>x + (125 − x) × e−1</p><p>e Y2 = x + 23</p><p>e procurando as coordenadas do ponto de intersecção, obtém-se x ≈ 21:</p><p>R.: Estima-se que o número de aves existentes no instante inicial era 21.</p><p>3.2 (Outra resolução.)</p><p>80 − 57 = 23. Então, N(5) − N(0) = 23.</p><p>Como A é um número inteiro positivo e menor que 25, temos um número finito de posśıveis soluções,</p><p>pelo que poderemos resolver o problema por tentativa e erro.</p><p>Se A = 24 então,</p><p>N(t) =</p><p>125 × 24</p><p>(24 + 101)e−0,2t</p><p>e na tabela observa-se</p><p>0</p><p>5</p><p>24</p><p>49,1</p><p>t N</p><p>resultando N(5) − N(0) = 25, 1.</p><p>Para outros valores de A obtêm-se os resultados:</p><p>0</p><p>5</p><p>23</p><p>47,5</p><p>t N</p><p>0</p><p>5</p><p>22</p><p>45,9</p><p>t N</p><p>0</p><p>5</p><p>21</p><p>44,3</p><p>t N</p><p>0</p><p>5</p><p>20</p><p>42,6</p><p>t N</p><p>A = 23 A = 22 A = 21 A = 20</p><p>24,5 23,9 23,3 22,6</p><p>R.: Atendendo a que existe uma única solução (de acordo com o enunciado), A = 21 parece ser o</p><p>valor que melhor traduz esta situação.</p><p>4. a</p><p>4.1 a</p><p>4.1.1 0 < diâmetro da esfera < 2 logo,</p><p>0 < raio da esfera < 1.</p><p>R.: ]0, 1[.</p><p>4.1.2 a</p><p>Volume da esfera de raio x:</p><p>4</p><p>3</p><p>πx3.</p><p>Aresta do cubo: a = 2 − 2x</p><p>3</p><p>Volume do cubo: (2 − 2x)3</p><p>Volume da escultura:</p><p>4</p><p>3</p><p>πx3 + (2 − 2x)3 = V (x).</p><p>Falta, agora, mostrar que esta expressão é equivalente à do enunciado. Podemos fazê-lo recor-</p><p>rendo à calculadora ou analiticamente.</p><p>Introduzindo na calculadora, editor de funções, as expressões</p><p>Y1 =</p><p>4</p><p>3</p><p>πx3 + (2 − 2x)3 e Y2 =</p><p>(</p><p>4π − 24</p><p>3</p><p>)</p><p>x3 + 24x2 − 24x + 8,</p><p>verifica-se a sobreposição dos dois gráficos. Utilizando o cursor podemos confirmar a igualdade</p><p>das coordenadas de vários pontos das duas funções.</p><p>A mesma igualdade também pode ser observada recorrendo a uma tabela com alguns valores:</p><p>Duas funções cúbicas que coincidem em, pelo menos 4 pontos são idênticas. Assim, o volume da</p><p>escultura pode ser definido pela expressão dada no enunciado.</p><p>Resolução anaĺıtica.</p><p>V (x) =</p><p>4</p><p>3</p><p>πx3 + (8 − 24x + 24x2 − 8x3) ⇔ V (x) =</p><p>(</p><p>4</p><p>3</p><p>π − 8</p><p>)</p><p>x3 + 24x2 − 24x + 8 ⇔</p><p>⇔ V (x) =</p><p>(</p><p>4π − 24</p><p>3</p><p>)</p><p>x3 + 24x2 − 24x + 8.</p><p>Como se queria mostrar.</p><p>Cálculos auxiliares.</p><p>(2 − 2x)3 = (2 − 2x)2(2 − 2x) = (4 − 8x + 4x2)(2 − 2x) =</p><p>= 8 − 8x − 16x + 16x2 + 8x2 − 8x3 = 8 − 24x + 24x2 − 8x3.</p><p>4</p><p>4.1.3 Pela visualização do gráfico da função volume confirmamos que existe um mı́nimo igual a 1, 41,</p><p>para x = 0, 58. Assim, o volume da escultura é mı́nimo se o raio da esfera for = 0, 58 metros e a</p><p>aresta do cubo igual a 0, 84 metros (2 − 2 × 0, 58).</p><p>4.2 a</p><p>raio da esfera = 0, 5m;</p><p>aresta do cubo = 1m;</p><p>área da superf́ıcie esférica = 4π · (0, 5)2 ≈ 3, 142m2;</p><p>área das cinco faces do cubo = 5 × 1 = 5m2;</p><p>área total = 8, 142m2</p><p>1 lata −→ 2, 5m2;</p><p>2 latas −→ 5m2;</p><p>3 latas −→ 7, 5m2 (insuficiente);</p><p>4 latas −→ 10m2;</p><p>R.: Será necessário comprar 4 latas de tinta.</p><p>5. a</p><p>5.1 A distância mı́nima da Terra ao Sol verifica-se no periélio, para x = 0. Esta distância é igual a</p><p>d = 149, 6(1 − 0, 0167cos 0) ≈ 147, 1 milhões de quilómetros.</p><p>A distância máxima da Terra ao Sol verifica-se para x = π, por observação da figura, e é dada por</p><p>d = 149, 6(1 − 0, 0167cos π) ≈ 152, 1 milhões de quilómetros.</p><p>Podemos também obter estes valores graficamente:</p><p>5.2 a</p><p>5.2.1 Para x = π, tem-se</p><p>2πt</p><p>T</p><p>= π − 0, 0167senπ ⇔ 2πt</p><p>T</p><p>= π ⇔ 2πt = πT ⇔ 2t = T ⇔ t =</p><p>T</p><p>2</p><p>.</p><p>A Terra demora metade de um ano (365, 24/2) a descrever metade da órbita.</p><p>5</p><p>5.2.2 a</p><p>t = 0 // 4 Janeiro</p><p>��</p><p>t = 41 14 Fevereirooo</p><p>x =?</p><p>2π × 41</p><p>365, 24</p><p>= x − 0, 0167senx</p><p>Considerando</p><p>Y1 = x − 0, 0167senx e Y2 =</p><p>2π × 41</p><p>365, 24</p><p>pretende-se determinar a intersecção dos dois gráficos.</p><p>Obtém-se x ≈ 0, 71628. Logo, d = 149, 6(1 − 0, 0167 · cos(0, 71628)) ≈ 147, 7 milhões de</p><p>quilómetros.</p><p>Cálculo de d (outro processo).</p><p>Inserir a função d em Y3 e procurar a ordenada do ponto de abcissa x = 0, 71628.</p><p>6. a</p><p>T.m.v[2;3,5] =</p><p>81, 5 − 85</p><p>3, 5 − 2</p><p>= −2, 333 ◦C/min</p><p>T.m.v[2,3] =</p><p>82, 6 − 85</p><p>3 − 2</p><p>= −2, 4 ◦C/min</p><p>T.m.v[2;2,5] =</p><p>83, 8 − 85</p><p>2, 5 − 2</p><p>= −2, 4 ◦C/min</p><p>De acordo com os valores obtidos, estima-se que a taxa de variação instantânea da temperatura da água</p><p>no instante t = 2 possa ser −2, 4 ◦C/min. Tendo em conta a fórmula dada no enunciado e que T (2) = 85</p><p>e A = 25,</p><p>−2, 4 = k(85 − 25) ⇔ −2, 4 = 60k ⇔ k = −2, 4</p><p>60</p><p>⇔ k = −0, 04.</p><p>R.: k = −0.04.</p><p>6</p><p>2007</p><p>V.S.F.F.735/1</p><p>PROVA 735/11 Págs.</p><p>EXAME NACIONAL DO ENSINO SECUNDÁRIO</p><p>10.º/11.º ou 11.º/12.º Anos de Escolaridade</p><p>Decreto-Lei n.º 74/2004,de 26 de Março</p><p>Duração da prova: 150 minutos 1.ª FASE</p><p>2007</p><p>PROVA ESCRITA DE MATEMÁTICA B</p><p>735/2</p><p>Identifique claramente os itens a que responde.</p><p>Utilize apenas caneta ou esferográfica de tinta azul ou preta</p><p>(excepto nas respostas que impliquem a elaboração de</p><p>construções, desenhos ou outras representações).</p><p>É interdito o uso de «esferográfica-lápis» e de corrector.</p><p>As cotações da prova encontram-se na página 10.</p><p>A prova inclui um formulário (página 11).</p><p>V.S.F.F.735/3</p><p>Em todos os itens da prova, apresente o seu raciocínio de forma clara, indicando</p><p>todos os cálculos que tiver de efectuar e todas as justificações necessárias.</p><p>Apresente uma única resposta a cada item. Se escrever mais do que uma resposta,</p><p>deve indicar, de forma inequívoca, a que pretende que seja classificada (riscando</p><p>todas as que pretende anular).</p><p>Sempre que, na resolução de um problema, recorrer à sua calculadora, apresente</p><p>todos os elementos recolhidos na sua utilização. Mais precisamente:</p><p>• sempre que recorrer às capacidades gráficas da sua calculadora, apresente o</p><p>gráfico, ou gráficos, obtido(s), bem como coordenadas de pontos relevantes para a</p><p>resolução do problema proposto (por exemplo, coordenadas de pontos de</p><p>intersecção de gráficos, máximos, mínimos, etc.);</p><p>• sempre que recorrer a uma tabela obtida na sua calculadora, asapresente todas</p><p>linhas da tabela relevantes para a resolução do problema proposto;</p><p>• sempre que recorrer a estatísticas obtidas na sua calculadora (média, desvio</p><p>padrão, coeficiente de correlação, declive e ordenada na origem de uma recta de</p><p>regressão, etc.), as listas que introduziu na calculadora para as obter.apresente</p><p>735/4</p><p>1. Dispõe-se de dois dados perfeitos, um tetraedro e um cubo, com faces numeradas de a"</p><p>% " ' e de a , respectivamente.</p><p>Considere a experiência aleatória que consiste em lançar, simultaneamente, os dois dados</p><p>e registar a soma do número da face que fica voltada para baixo, no caso do tetraedro,</p><p>com o número da face que fica voltada para cima, no caso do cubo.</p><p>1.1. Construa o modelo de probabilidades associado à experiência aleatória</p><p>considerada.</p><p>Apresente as probabilidades na forma de fracção.</p><p>Nota: Construir um modelo de probabilidades consiste em construir uma tabela,</p><p>associando aos resultados da experiência aleatória a respectiva probabilidade.</p><p>1.2. Com base na experiência aleatória descrita, a Ana e o João decidem fazer um jogo.</p><p>A Ana lança o tetraedro e o João lança o cubo. A Ana sugere que as regras do jogo</p><p>consistam no seguinte:</p><p>• ganha o João se a soma dos números saídos for ímpar;</p><p>• ganha a Ana se a soma dos números saídos for par.</p><p>Porém, o João diz que as regras não são justas,</p>
<p>afirmando que a Ana tem</p><p>vantagem, uma vez que existem mais somas pares do que ímpares.</p><p>Num pequeno texto, comente o argumento do João, referindo se ele tem, ou não,</p><p>razão.</p><p>Deve incluir, obrigatoriamente, na sua resposta:</p><p>• uma análise do argumento do João, referindo o número de somas pares</p><p>e o número de somas ímpares;</p><p>• o valor da probabilidade de «sair soma par»;</p><p>• o valor da probabilidade de «sair soma ímpar»;</p><p>• conclusão final, referindo se o João tem, ou não, razão.</p><p>V.S.F.F.735/5</p><p>2. Uma autarquia pondera o abastecimento anual de energia eléctrica para iluminação da via</p><p>pública. Para o efeito, a rede nacional pode fornecer-lhe dois tipos de energia: energia de</p><p>origem convencional, maioritariamente resultante da combustão de , ou, emfuel</p><p>alternativa, energia eólica.</p><p>Para uma cobertura razoável de iluminação, no período nocturno, o consumo anual de</p><p>energia não poderá ser inferior a .%! Q[2</p><p>Por razões ambientais, a autarquia pretende que a quantidade de energia de origem</p><p>convencional não exceda a quantidade de energia eólica fornecida.</p><p>Relativamente à energia de origem convencional, tem-se:</p><p>• o preço por cada é de euros.Q[2 )!</p><p>Relativamente à energia eólica, tem-se:</p><p>• o preço por cada é de euros;Q[2 *!</p><p>• o fornecimento de energia, nesse ano, não poderá ultrapassar os .%!Q[2</p><p>Represente por a quantidade de energia de origem convencional e por a quantidadeB C</p><p>de energia eólica consumidas pela autarquia.</p><p>Determine que quantidade de energia de cada tipo deve ser consumida, por ano, de modo</p><p>que possam ser minimizados os custos, tendo em conta as condicionantes referidas.</p><p>Percorra, sucessivamente, as seguintes etapas:</p><p>• indique as restrições do problema;</p><p>• indique a função objectivo;</p><p>• represente graficamente a região admissível (referente ao sistema das</p><p>restrições);</p><p>• indique os valores de e para os quais é mínima a função objectivo.B C</p><p>735/6</p><p>3. Pretende-se elaborar um painel publicitário com a forma de um quadrado com metros"!</p><p>de lado. O painel deve conter três círculos luminosos, tangentes entre si, como mostra a</p><p>figura.</p><p>Relativamente ao painel, considere que:</p><p>• os diâmetros dos três círculos variam permanentemente e os seus centros estão</p><p>sempre na mesma mediana do quadrado;</p><p>• os círculos nunca saem fora do quadrado;</p><p>• os círculos inferior e superior são geometricamente iguais e são tangentes a</p><p>lados opostos do quadrado;</p><p>• quando os diâmetros dos círculos inferior e superior aumentam, diminui o</p><p>diâmetro do círculo central, e vice-versa, como sugere a figura seguinte.</p><p>Sejam o raio dos círculos inferior e superior e o raio do círculo central.= <</p><p>V.S.F.F.735/7</p><p>3.1. Mostre que = œ + <&</p><p>#</p><p>"</p><p>#</p><p>3.2. Verifique que a soma, , das áreas dos três círculos, em função de , é dada por:E <</p><p>E < œ < + & < . ! / < / & ! $ #&</p><p># #1 1 1# ,</p><p>4. O Pedro foi juntando algumas economias e, neste momento, tem euros que decide"!!!</p><p>colocar no banco, constituindo uma poupança.</p><p>Para o efeito dispõe de duas opções:</p><p>Opção A:</p><p>Por cada ano de aplicação do capital, o Pedro recebe euros de juros.%!</p><p>Opção B:</p><p>Por cada ano de aplicação do capital, o Pedro recebe juros à taxa anual de %, a incidir$ &,</p><p>sobre o capital total acumulado até à data.</p><p>4.1. Relativamente à , designe por a sucessão cujos termos são osopção B !,8</p><p>valores do capital existente decorridos anos.8</p><p>Sabendo que é uma progressão geométrica, determine a razão. !,8</p><p>Justifique a sua resposta.</p><p>4.2. Comente a seguinte afirmação:</p><p>«Comparando as duas opções apresentadas, se nos primeiros anos a opção é aA</p><p>melhor escolha, a partir de certa altura a opção torna-se mais vantajosa.»B</p><p>Sugestão: Determine o ano a partir do qual o capital acumulado de acordo com a</p><p>opção é superior ao capital acumulado caso se tivesse escolhido a opção .B A</p><p>Poderá ser útil ter em atenção que , œ "!!! ‚ " !$&8 8,</p><p>735/8</p><p>5. Sabe-se que a concentração, , em miligramas por litro, de um analgésico, na circulaçãoG</p><p>sanguínea, horas após a sua ingestão, é dada por:></p><p>G > œ "! / + / ! !"> "#></p><p>Nota: Na resolução das questões seguintes, sempre que, em cálculos intermédios,</p><p>proceder a arredondamentos, conserve duas casas decimais.</p><p>5.1. Qual é a concentração, aproximada, do analgésico uma hora e trinta minutos após</p><p>a sua ingestão?</p><p>Apresente o resultado arredondado às centésimas.</p><p>5.2. Sabe-se que o analgésico tem o efeito desejado quando a sua concentração é</p><p>superior a miligramas por litro.! &,</p><p>Considere que o analgésico foi ingerido às nove horas.</p><p>Recorrendo às potencialidades da calculadora gráficaß indique uma</p><p>aproximação do intervalo em que ele produz o efeito desejado.</p><p>Apresente os resultados em horas e minutos (com os minutos arredondados às</p><p>unidades).</p><p>V.S.F.F.735/9</p><p>6. Um farol (ponto ), situado numa ilha, encontra-se a da costa. Nesta, sobre aJ "! 57</p><p>perpendicular tirada do farol, está um observador (ponto ).E</p><p>A luz do farol descreve sucessivos círculos e tem um alcance de . Em cada"! 57</p><p>instante, o farol ilumina segundo uma trajectória rectilínea, com extremidade num ponto ,T</p><p>que percorre a circunferência representada na figura seguinte.</p><p>Sejam:</p><p>• a amplitude, em graus, do ângulo orientado cujo lado origem é a semi-rectaα</p><p>JE JTÞ Þ e cujo lado extremidade é a semi-recta</p><p>• Q o ponto médio de Ò ÓET</p><p>• TF T a distância do ponto à costa</p><p>Mostre que, para :! ! ! ")!º ºα</p><p>6.1. a distância, do observador ao ponto é dada,ET , expressa em quilómetros, T</p><p>em função de , porα</p><p>ET œ #! =/8ˆ ‰α#</p><p>6.2. a distância, , do ponto à costa é dada, em função. Texpressa em quilómetros,</p><p>de , porα</p><p>. œ #! !α =/8#Š ‹α#</p><p>Percorra, sucessivamente, as seguintes etapas:</p><p>• escreva , em função de JETs α</p><p>• escreva , em função de TEFs α</p><p>• escreva , em função de FT α</p><p>FIM</p><p>735/10</p><p>COTAÇÕES</p><p>1. 30 pontos...............................................................................</p><p>1.1. ...............................................................14 pontos</p><p>1.2. ...............................................................16 pontos</p><p>2. 22 pontos...............................................................................</p><p>3. 41 pontos...............................................................................</p><p>3.1. .............................................................. 19 pontos</p><p>3.2. ...............................................................22 pontos</p><p>4. 25 pontos...............................................................................</p><p>4.1. .............................................................. 10 pontos</p><p>4.2. ...............................................................15 pontos</p><p>5. 41 pontos...............................................................................</p><p>5.1.................................................................19 pontos</p><p>5.2. ...............................................................22 pontos</p><p>6. 41 pontos...............................................................................</p><p>6.1. ...............................................................19 pontos</p><p>6.2. ...............................................................22 pontos</p><p>TOTAL .................................................................... 200 pontos</p><p>735/11</p><p>Formulário</p><p>Comprimento de um arco de circunferência</p><p>α α< + < ( )amplitude, em radianos, do ângulo ao centro raio; $</p><p>Áreas de figuras planas</p><p>Losango: H3+198+67+39<‚H3+198+67/89<</p><p>#</p><p>Trapézio: F+=/7+39<0F+=/7/89<</p><p># ‚E6>?<+</p><p>Polígono regular: Semiperímetro Apótema‚</p><p>Sector circular: α <#</p><p># (α+ amplitude,</p><p>em radianos, do ângulo ao centro raio; < $ )</p><p>Áreas de superfícies</p><p>Área lateral de um cone: 1 < 1</p><p>( )< 1$ $raio da base geratriz;</p><p>Área de uma superfície esférica: % <1 #</p><p>( )< $ raio</p><p>Volumes</p><p>Pirâmide: "$ ‚ Área da base Altura‚</p><p>Cone: "$ ‚ Área da base Altura‚</p><p>Esfera: %$ $1 ( )< < $ raio</p><p>Progressões</p><p>Soma dos primeiros termos de uma8</p><p>Prog. Aritmética: ? &?</p><p>#</p><p>" 8 ‚ 8</p><p>Prog. Geométrica: ? ‚"</p><p>"' <</p><p>"' <</p><p>8</p><p>V.S.F.F.</p><p>735/C/1</p><p>PROVA 735/C/10 Págs.</p><p>EXAME</p>
<p>NACIONAL DO ENSINO SECUNDÁRIO</p><p>10.º/11.º ou 11.º/12.º Anos de Escolaridade</p><p>(Decreto-Lei n.º 74/2004, de 26 de Março)</p><p>Duração da prova: 150 minutos 1.ª FASE</p><p>2007</p><p>PROVA ESCRITA DE MATEMÁTICA B</p><p>COTAÇÕES</p><p>1. 30 pontos................................................................................................</p><p>1.1. .......................................................................14 pontos</p><p>1.2. .......................................................................16 pontos</p><p>2. 22 pontos................................................................................................</p><p>3. 41 pontos................................................................................................</p><p>3.1. ...................................................................... 19 pontos</p><p>3.2. .......................................................................22 pontos</p><p>4. 25 pontos................................................................................................</p><p>4.1. ...................................................................... 10 pontos</p><p>4.2. .......................................................................15 pontos</p><p>5. 41 pontos................................................................................................</p><p>5.1. .......................................................................19 pontos</p><p>5.2. .......................................................................22 pontos</p><p>6. 41 pontos................................................................................................</p><p>6.1. .......................................................................19 pontos</p><p>6.2. .......................................................................22 pontos</p><p>TOTAL ..................................................................................... 200 pontos</p><p>735/C/2</p><p>CRITÉRIOS GERAIS DE CLASSIFICAÇÃO</p><p>1. Se o examinando se enganar na identificação do item a que está a responder, ou se a omitir,</p><p>mas, pela resolução apresentada, for possível identificá-lo inequivocamente, a resposta deve</p><p>ser vista e classificada.</p><p>2. Se o examinando responder ao mesmo item mais do que uma vez, deve eliminar</p><p>inequivocamente a(s) resposta(s) que não deve(m) ser classificada(s). No caso de tal não</p><p>acontecer, será classificada a resposta que surge em primeiro lugar.</p><p>3. Num item em que a respectiva resolução exija cálculos e/ou justificações, a classificação</p><p>deve ser:</p><p>• a soma algébrica das classificações atribuídas a cada etapa, de acordo com o disposto</p><p>nos pontos 5, 6, 7, 8, 9, 10 e 11 destes critérios gerais. Se a soma for negativa, a</p><p>classificação a atribuir é de zero pontos;</p><p>• de zero pontos se o examinando se limitar a apresentar o resultado final.</p><p>4. Sempre que o examinando utilizar um processo de resolução não contemplado nos critérios</p><p>específicos, caberá ao professor classificador adoptar um critério de distribuição da cotação</p><p>que julgue adequado. Salienta-se que deve ser aceite qualquer processo cientificamente</p><p>correcto, mesmo que envolva conhecimentos ou competências não contemplados no</p><p>Programa da disciplina.</p><p>5. A cotação de cada item está subdividida pelas etapas que o examinando deve percorrer</p><p>para o resolver.</p><p>5.1. Em cada etapa, a cotação indicada é a máxima a atribuir.</p><p>5.2. O classificador não pode subdividir, em cotações parcelares, a cotação de cada etapa.</p><p>Caso uma etapa envolva um único passo, testando apenas o conhecimento de um só</p><p>conceito ou propriedade, e a sua resolução não esteja completamente correcta, deve</p><p>ser atribuída a classificação de zero pontos.</p><p>Caso uma etapa envolva mais do que um passo (por exemplo, a resolução de uma</p><p>equação, a obtenção de uma expressão em função de uma variável, etc.) e a sua</p><p>resolução esteja incompleta, ou contenha incorrecções, a classificação a atribuir deve</p><p>estar de acordo com o grau de incompletude e/ou com a gravidade dos erros</p><p>cometidos. Por exemplo:</p><p>• erros de contas ocasionais devem ser desvalorizados em um ponto;</p><p>• erros que revelem desconhecimento de conceitos, regras ou propriedades devem</p><p>ser desvalorizados em, pelo menos, metade da cotação da etapa;</p><p>• transposições erradas de dados do enunciado devem ser desvalorizadas em um</p><p>ponto, desde que o grau de dificuldade da etapa não diminua;</p><p>• transposições erradas de dados do enunciado devem ser desvalorizadas em, pelo</p><p>menos, metade da cotação da etapa, caso o grau de dificuldade da etapa diminua.</p><p>5.3. Nas etapas cuja cotação se encontra discriminada por níveis de desempenho, o</p><p>classificador deve enquadrar a resposta do examinando numa das descrições</p><p>apresentadas. O classificador não pode atribuir uma classificação diferente das</p><p>indicadas.</p><p>V.S.F.F.</p><p>735/C/3</p><p>5.4. No caso de o examinando cometer um erro numa das etapas, as etapas subsequentes</p><p>devem merecer a respectiva classificação, desde que o grau de dificuldade não tenha</p><p>diminuído, e o examinando as execute correctamente, de acordo com o erro que</p><p>cometeu.</p><p>5.5. Caso o examinando cometa, numa etapa, um erro que diminua o grau de dificuldade</p><p>das etapas subsequentes, cabe ao classificador decidir a classificação máxima a</p><p>atribuir a cada uma destas etapas. Em particular, se, devido a um erro cometido pelo</p><p>examinando, o grau de dificuldade das etapas seguintes diminuir significativamente, a</p><p>classificação máxima a atribuir em cada uma delas não deverá exceder metade da</p><p>cotação indicada.</p><p>5.6. Pode acontecer que o examinando, ao resolver um item, não percorra explicitamente</p><p>todas as etapas previstas nos critérios específicos. Todas as etapas não percorridas</p><p>explicitamente pelo examinando, mas cuja utilização e/ou conhecimento estejam</p><p>inequivocamente implícitos na resolução do item, devem receber a cotação indicada.</p><p>6. Nas etapas em que está previsto o recurso à calculadora, os critérios específicos</p><p>subdividem-se em: «Explicação do método utilizado» e «Apresentação do(s) valor(es)».</p><p>6.1. Explicação do método utilizado:</p><p>De acordo com as instruções gerais para a realização da prova, o examinando deve</p><p>apresentar todos os elementos recolhidos na utilização da calculadora. Esta</p><p>apresentação deve ser classificada de acordo com os níveis de desempenho que se</p><p>seguem. Em cada nível de desempenho, a classificação máxima prevista é a indicada</p><p>em percentagem.</p><p>Esta percentagem deve ser aplicada sobre a cotação prevista para a explicação do</p><p>método utilizado, e o valor obtido deve ser arredondado às unidades (por excesso, se</p><p>a mantissa do número a arredondar for 0,5 ou superior).</p><p>Apresentação correcta e completa de todos os elementos relevantes....................100%</p><p>Apresentação correcta, mas com ausência de alguns elementos relevantes</p><p>ou</p><p>Apresentação completa, mas com algumas incorrecções (por exemplo, não respeitar o</p><p>domínio de uma função) ........................................................................................... 70%</p><p>Apresentação incompleta e com algumas incorrecções .......................................... 40%</p><p>Ausência de explicação ou simples referências do tipo «Vi na calculadora»............ 0%</p><p>6.2. Apresentação do(s) valor(es):</p><p>Para cada valor que o examinando deve apresentar, os critérios específicos podem</p><p>indicar um intervalo admissível. O valor apresentado pelo examinando pode pertencer,</p><p>ou não, a esse intervalo.</p><p>• Se o valor pertencer ao intervalo, deve ser atribuída a cotação máxima prevista</p><p>para essa apresentação, a menos que haja lugar a qualquer desvalorização</p><p>prevista nos critérios específicos, por desrespeito do número de casas decimais</p><p>com que o resultado deve ser apresentado.</p><p>• Se o valor não pertencer ao intervalo, deve ser atribuída a classificação de zero</p><p>pontos.</p><p>735/C/4</p><p>7. Quando, num item, é pedida uma forma específica de apresentação do resultado final (por</p><p>exemplo, «em minutos», «em percentagem», etc.), este deve ser apresentado na forma</p><p>pedida. Se o resultado final apresentado pelo examinando não respeitar a forma pedida no</p><p>enunciado (por exemplo, se o enunciado pedir o resultado em minutos, e o examinando o</p><p>apresentar em horas),</p>
<p>devem ser atribuídos zero pontos na etapa correspondente ao</p><p>resultado final. No entanto, a resposta não deve ser desvalorizada se não indicar a unidade</p><p>em que é pedido o resultado (por exemplo, se o resultado final for 12 minutos, ou 12 metros,</p><p>e o examinando escrever simplesmente 12, não deve existir qualquer desvalorização).</p><p>8. O examinando deve respeitar sempre a instrução relativa à apresentação de todos os</p><p>cálculos e de todas as justificações. Se, numa etapa, o examinando não respeitar esta</p><p>instrução, apresentando algo (valor, quadro, tabela, gráfico, etc.) que não resulte de trabalho</p><p>anterior, deve ser atribuída a classificação de zero pontos a essa etapa. Todas as etapas</p><p>subsequentes que dela dependam devem ser igualmente classificadas com zero pontos.</p><p>9. O examinando deve respeitar sempre qualquer instrução relativa ao método a utilizar na</p><p>resolução de um item (por exemplo, «equacione o problema», «resolva graficamente», etc.).</p><p>Na resolução apresentada pelo examinando, deve ser inequívoco, pela apresentação de</p><p>todos os cálculos e de todas as justificações, o cumprimento da instrução. Se tal não</p><p>acontecer, considera-se que o examinando não respeitou a instrução. A etapa em que se dá</p><p>o desrespeito e todas as subsequentes que dela dependam devem ser classificadas com</p><p>zero pontos.</p><p>10. Se, na resolução de um item, o examinando utilizar simbologia, ou escrever uma expressão,</p><p>inequivocamente incorrecta do ponto de vista formal (por exemplo, se escrever o símbolo de</p><p>igualdade onde deveria estar o símbolo de equivalência), a sua resposta deve ser</p><p>desvalorizada em um ponto. Esta desvalorização não se aplica no caso em que tais</p><p>incorrecções ocorram apenas em etapas classificadas com zero pontos, nem a eventuais</p><p>utilizações do símbolo de igualdade, onde, em rigor, deveria estar o símbolo de igualdade</p><p>aproximada.</p><p>11. Existem itens em cujo enunciado é dada uma instrução relativa ao número mínimo de casas</p><p>decimais que o examinando deve conservar, sempre que, em cálculos intermédios, proceder</p><p>a arredondamentos. Indicam-se, a seguir, as desvalorizações a aplicar, na classificação total</p><p>da resposta, em caso de desrespeito dessa instrução e/ou de arredondamentos mal</p><p>efectuados.</p><p>Todos os valores intermédios estão de acordo com a instrução, mas existe, pelo menos, um</p><p>valor intermédio mal arredondado................................................................................. -1 ponto</p><p>Todos os valores intermédios estão bem arredondados, mas existe, pelo menos, um que</p><p>não está de acordo com a instrução............................................................................. -1 ponto</p><p>Existe, pelo menos, um valor intermédio mal arredondado e existe, pelo menos, um que não</p><p>está de acordo com a instrução ................................................................................. -2 pontos</p><p>12. As classificações a atribuir às respostas dos examinandos devem ser expressas,</p><p>obrigatoriamente, em números inteiros.</p><p>V.S.F.F.</p><p>735/C/5</p><p>Critérios específicos de classificação</p><p>1.1. ................................................................................................................... 14</p><p>Identificar todas as somas possíveis ....................................................................3</p><p>Determinar a probabilidade associada a cada um dos resultados ..............(1x9) 9</p><p>Construção clara e correcta da tabela................................................................... 2</p><p>Notas:</p><p>1. Se o examinando tiver elaborado uma tabela de dupla entrada, onde</p><p>identifique todas as somas possíveis (primeira etapa), pode depreender-se</p><p>que os valores encontrados para a probabilidade associada a cada um dos</p><p>resultados (segunda etapa), estando correctos, se baseiam na sua leitura.</p><p>Caso contrário, o examinando deverá apresentar as diferentes</p><p>configurações correspondentes a cada soma (por exemplo, no caso de a</p><p>soma ser sete, discriminar os resultados: . Se o ! ! ! !"ß ' à #ß & à $ß % à %ß $</p><p>examinando não apresentar uma tabela de dupla entrada para o cálculo das</p><p>probabilidades, nem apresentar qualquer justificação que a substitua, a</p><p>classificação a atribuir à segunda etapa deverá ser de 5 pontos.</p><p>2. Se o examinando não respeitar a indicação quanto à forma geométrica dos</p><p>dados (por exemplo, se considerar dois cubos), a a atribuir àclassificação</p><p>sua resposta deve ser desvalorizada em 5 pontos.</p><p>1.2. .................................................................................................................. 16</p><p>A composição deverá contemplar os seguintes tópicos:</p><p>• uma análise do argumento do João, referindo que existem cinco somas</p><p>pares e quatro somas ímpares;</p><p>• valor da probabilidade de sair soma par ;Œ #"#</p><p>• valor da probabilidade de sair soma ímpar ;Œ #"#</p><p>• conclusão final, referindo que o João não tem razão.</p><p>Na tabela seguinte, indica-se como este item deve ser classificado:</p><p>Forma</p><p>Conteúdo</p><p>Nível 3 Nível 2 Nível 1</p><p>( ) ( ) ( )</p><p>A composição contempla</p><p>os quat</p><p>‡ ‡‡ ‡‡‡</p><p>ro tópicos.</p><p>correctamente 16 15 14</p><p>A composição contempla correctamente 12 11 10</p><p>três tópicos.</p><p>A composição contempla</p><p>dois tópicos.</p><p>correctamente 8 7 6</p><p>A composição contempla correctamente 4 3 2</p><p>um tópico.</p><p>735/C/6</p><p>( ) Composição bem estruturada, sem erros de sintaxe, de‡ Nível 3</p><p>pontuação e/ou de ortografia.</p><p>( ) Composição razoavelmente estruturada, com alguns erros de‡‡ Nível 2</p><p>sintaxe, de pontuação e/ou de ortografia, cuja gravidade não</p><p>implique a perda de inteligibilidade e/ou de sentido.</p><p>( ) Composição sem estruturação aparente, com a presença de‡‡‡ Nível 1</p><p>erros graves de sintaxe, de pontuação e/ou de ortografia, com</p><p>perda frequente de inteligibilidade e/ou de sentido.</p><p>Nota: Se a tabela de probabilidades construída no item anterior estiver</p><p>incorrecta, mas o examinando agir em conformidade com os resultados nela</p><p>indicados, a classificação a atribuir não deverá ser desvalorizada. No entanto, se</p><p>a soma das duas probabilidades for diferente de um, a classificação total a</p><p>atribuir a esta resposta deve ser desvalorizada em 6 pontos.</p><p>2. ...................................................................................................................... 22</p><p>Indicar as restrições .............................................................................................. 8</p><p>B � ! .....................................................................................................1</p><p>C � ! ..................................................................................................... 1</p><p>C Ÿ %! ...................................................................................................2</p><p>B & C � %! .........................................................................................2</p><p>C � B .....................................................................................................2</p><p>Indicar a função objectivo: ........................................4 ! !G Bß C œ )!B & *! C</p><p>Representar graficamente a região admissível .....................................................6</p><p>Representar correctamente ......................................................2 B & C œ %!</p><p>Representar correctamente ................................................................ 2 C œ B</p><p>Representar correctamente .............................................................. 2 C œ %!</p><p>Indicar os valores de e para os quais é mínima a função objectivoB C</p><p>ÐB œ #! C œ #!Ñ e ........................................................................................... 4</p><p>V.S.F.F.</p><p>735/C/7</p><p>3.1. .................................................................................................................. 19</p><p>Escrever ........................................................................................ 9 %= & #< œ "!</p><p>Escrever ........................................................................................10 = œ 2 <& "</p><p># #</p><p>3.2. ..................................................................................................................</p>
<p>22</p><p>Constatar que ........................................................................7 E œ # = & <1 1# #</p><p>Substituir por .................................................................................................. 7 = <</p><p>Calcular e simplificar ................................................................................... 8 E < !</p><p>Nota: Se o examinando substituir por um valor aproximado, em qualquer das1</p><p>etapas, a classificação a atribuir à sua resposta deve ser desvalorizada em 4</p><p>pontos.</p><p>735/C/8</p><p>4.1. .................................................................................................................. 10</p><p>Determinar e e .................................. (3+3) 6 , , , œ "!$& , œ "!(" ##&" # " # , !</p><p>Dividir por e concluir que ........................................................... 4 , , < œ " !$&# " ,</p><p>Notas:</p><p>1. O examinando pode explicitar que o factor actualiza, por multiplicação," !$&,</p><p>o capital do ano anterior. Nesse caso, deverá ser-lhe atribuída a cotação total do</p><p>item (10 pontos).</p><p>2. Se o examinando indicar o valor da razão sem qualquer justificação, a sua</p><p>resposta deve ser classificada com 3 pontos.</p><p>4.2. .................................................................................................................. 15</p><p>Concluir que, no nono ano, o capital acumulado de acordo com a opção</p><p>B A é superior ao capital acumulado caso se tivesse escolhido a opção .......... 10</p><p>Justificar que, nos primeiros anos, a opção é a melhor escolha, mas, aA</p><p>partir do nono ano, a opção torna-se mais vantajosa ......................5 B (ver nota)</p><p>Nota: Se o examinando tornar compreensível o seu raciocínio, mas não o fizer</p><p>de um modo totalmente claro, a classificação total a atribuir a esta etapa deverá</p><p>ser desvalorizada em 1 ponto.</p><p>V.S.F.F.</p><p>735/C/9</p><p>5.1. .................................................................................................................. 19</p><p>Reconhecer que ....................................................................................... 6 > œ " &,</p><p>G " & ¸ " ($71Î6 !, , ....................................................................................... 13</p><p>5.2. .................................................................................................................. 22</p><p>Equacionar o problema: .................................................. 6 ! & œ "! / 2 /, !"> "#></p><p>Resolução do problema .......................................................................................16</p><p>Explicação do método utilizado (ver critério geral 6.1.).......................... 5</p><p>> ¸ ! !& 2, ( ) .......................................................................2 ver nota 1</p><p>> ¸ $738 .............................................................................................. 2</p><p>> ¸ # *% 2, ( )..........................................................................2 ver nota 2</p><p>> ¸ #2 &'738 ...................................................................................... 2</p><p>Concluir que o intervalo de tempo está compreendido entre</p><p>*2 $738 "" 2 &'738e ......................................................................3</p><p>Notas:</p><p>1. Se o examinando indicar > ¸ ! !% > ¸ ! !', , ou , deve ser-lhe</p><p>atribuída a cotação total da etapa (2 pontos).</p><p>2. Se o examinando indicar > ¸ # *$ > ¸ # *&, , ou , deve ser-lhe</p><p>atribuída a cotação total da etapa (2 pontos).</p><p>735/C/10</p><p>6.1. .................................................................................................................. 19</p><p>Reconhecer que ou .................................................. 5 TJQ œ EJQ œs sα α</p><p># #</p><p>Escrever ............................................................................ 7 =/8 œŠ ‹α# "!</p><p>QT</p><p>Concluir que ET œ #! =/8ˆ ‰α# .........................................................................7</p><p>6.2. .................................................................................................................. 22</p><p>Reconhecer que ......................................................................6 JET œ *! 2s α</p><p>#</p><p>Reconhecer que .............................................................. 6 TEF œs α</p><p># (ver nota)</p><p>Escrever .............................................................................. 4 =/8ˆ ‰α# œ .</p><p>ET</p><p>Concluir que . œ #! !α =/8#Š ‹α# .....................................................................6</p><p>Nota: Se o examinando concluir directamente que por qualquerTEF œs α</p><p># ,método correcto, deve ser-lhe atribuída a classificação total correspondente às</p><p>duas primeiras etapas (6 + 6).</p><p>ASSOCIAÇÃO DE PROFESSORES DE MATEMÁTICA</p><p>PROPOSTA DE RESOLUÇÃO DA PROVA</p><p>MATEMÁTICA B (735 – 21 de Junho)</p><p>1.1.</p><p>No lançamento dos dois dados os resultados para a soma são os constantes na tabela</p><p>seguinte:</p><p>+ 1 2 3 4 5 6</p><p>1 2 3 4 5 6 7</p><p>2 3 4 5 6 7 8</p><p>3 4 5 6 7 8 9</p><p>4 5 6 7 8 9 10</p><p>Sendo o número de casos possíveis de 24 a distribuição de probabilidade de acordo com</p><p>a tabela anterior é:</p><p>ix 2 3 4 5 6 7 8 9 10</p><p>( )ixP 24</p><p>1</p><p>24</p><p>2</p><p>24</p><p>3</p><p>24</p><p>4</p><p>24</p><p>4</p><p>24</p><p>4</p><p>24</p><p>3</p><p>24</p><p>2</p><p>24</p><p>1</p><p>1.2.</p><p>Na tabela elaborada na alínea anterior existem 24 somas possíveis, das quais 12 são</p><p>pares e 12 são ímpares.</p><p>Pela distribuição de probabilidade temos que o valor da probabilidade de sair “soma</p><p>par” é:</p><p>P(“soma par”)=P(2)+P(4)+P(6)+P(8)+P(10)= .</p><p>2</p><p>1</p><p>24</p><p>1</p><p>24</p><p>3</p><p>24</p><p>4</p><p>24</p><p>3</p><p>24</p><p>1</p><p>=++++</p><p>P(“soma ímpar”)=1-P(“soma par”)=</p><p>2</p><p>1</p><p>2</p><p>1</p><p>1 =− .</p><p>Assim, o João não tem razão visto as somas pares e as somas ímpares serem</p><p>equiprováveis.</p><p>2.</p><p>Sejam então:</p><p>x energia convencional ao preço de 80 euros.</p><p>y energia eólica ao preço de 90 euros.</p><p>Como o consumo da energia convencional não pode exceder o da eólica uma das</p><p>condições é:</p><p>xy ≥</p><p>Os consumos são valores não negativos logo,</p><p>0</p><p>0</p><p>≥</p><p>≥</p><p>y</p><p>x</p><p>Como o total do consumo não pode ser inferior ao 40 MWh temos como condição:</p><p>40≥+ yx</p><p>Atendendo a que o consumo da energia eólica não pode ultrapassar os 40MWh ainda</p><p>temos:</p><p>40≤y</p><p>O custo será dado por 80x+90y valor que queremos minimizado</p><p>Inserindo no editor de funções as condições obtemos o seguinte gráfico na janela de</p><p>visualização [0,50]x[0,50]:</p><p>O triângulo da figura com a trama é a região admissível.</p><p>Os vértices da região admissível são: (0,40); (20,20) e (40,40)</p><p>Estes pontos são aqueles onde é possível que a função objectivo atinja um valor</p><p>mínimo.</p><p>A função é mínima em (20,20) isto é: 80x20+90x20=1710 pois em (0,40) vale:</p><p>80x0+90x40=3600 e em (40,40) vale: 80x40+90x40=6800</p><p>3.1.</p><p>Sendo s o raio de cada circunferência tangente aos lados do quadrado o seu diâmetro é</p><p>2s e o diâmetro da circunferência que está no meio é 2r. A soma destes diâmetros tem</p><p>de ser igual a 10, lado do quadrado, pelo que:</p><p>rsrsrss</p><p>2</p><p>1</p><p>2</p><p>5</p><p>5210222 −=⇔=+⇔=++</p><p>3.2.</p><p>Área do círculo superior é</p><p>2</p><p>2</p><p>1</p><p>2</p><p>5</p><p></p><p></p><p></p><p></p><p></p><p></p><p>− rπ</p><p>Área dos círculos superior e inferior é</p><p>2</p><p>5</p><p>2</p><p>25</p><p>4</p><p>1</p><p>2</p><p>5</p><p>4</p><p>25</p><p>2</p><p>2</p><p>1</p><p>2</p><p>5</p><p>2</p><p>2</p><p>2</p><p>2</p><p>r</p><p>rrrr</p><p>π</p><p>πππ +−=</p><p></p><p></p><p></p><p></p><p></p><p>+−=</p><p></p><p></p><p></p><p></p><p></p><p>−</p><p>Área do círculo central: 2</p><p>rπ</p><p>Área dos três círculos:</p><p>2</p><p>3</p><p>5</p><p>2</p><p>25</p><p>2</p><p>5</p><p>2</p><p>25 2</p><p>2</p><p>2</p><p>r</p><p>rr</p><p>r</p><p>r</p><p>π</p><p>ππ</p><p>π</p><p>π +−=++−</p><p>Aqui o raio (r) da circunferência central não pode superior a 5 senão o diâmetro era</p><p>maior que o lado do quadrado, e deve ser maior que 0 de modo a garantir a existência de</p><p>3 círculos.</p><p>4.1.</p><p>Sendo o valor inicial de 1000, no final do 1º ano terá 1000+1000 x 0,035=1035</p><p>No final do 2º ano terá 1035+1035 x 0,035=1071,225</p><p>Ou seja, 225,10711035 21 == beb</p><p>Como é afirmado que a sucessão é uma progressão geométrica o quociente de dois</p><p>termos consecutivos é igual à razão da progressão.</p><p>Assim a razão será dada por 035,1</p><p>1035</p><p>225,1071</p><p>1</p><p>2 ==</p><p>b</p><p>b</p><p>4.2. Seja na a sucessão associada à opção A. A sucessão é uma progressão aritmética de</p><p>razão 40 em que o 1º termo é igual a 1040.</p><p>( ) nana nn 4010004011040 +=⇔×−+=</p><p>Como nb é</p>
<p>uma progressão geométrica de razão 1,035 e 1º termo 1035 temos que:</p><p>n</p><p>n</p><p>n</p><p>n bb 035,11000035,11035 1</p><p>×=⇔×=</p><p>−</p><p>Usando a calculadora inserimos:</p><p>n</p><p>y</p><p>ny</p><p>035,11000</p><p>401000</p><p>2</p><p>1</p><p>×=</p><p>×=</p><p>Consultando a tabela da qual se apresenta um excerto:</p><p>podemos concluir que a partir do 8ºano a opção B é mais vantajosa e que até ao 8º ano</p><p>inclusive a opção A é a mais vantajosa.</p><p>5. 1.</p><p>Sabendo que um hora e tinta minutos é 1,5h temos:</p><p>( ) ( ) ...73343,1105,1 35,1</p><p>=−=</p><p>−−</p><p>eeC</p><p>A concentração é, aproximadamente, de 1,73 miligramas por litro.</p><p>5.2.</p><p>Teremos de ver durante quanto tempo é superior a 0,5 miligramas por litro ou seja</p><p>resolver a inequação:</p><p>10 5,0)( 2</p><p>>−</p><p>−− tt ee</p><p>Inserindo no editor de função em Y1 o 1ºmembro da inequação e em Y2 o 2º membro</p><p>da inequação, obtemos na janela [0,5]x[0,5] o gráfico seguinte:</p><p>O resultado vai estar no intervalo entre as duas intersecções. Estas obtêm-se com o</p><p>comando ‘intersect’ e são:</p><p>1ª intersecção (0,05;0,5) e 2ª intersecção (2,94;0,5)</p><p>Assim, o medicamento começa a fazer efeito 0,05x60=3 minutos depois de ser tomado e</p><p>deixa de fazer efeito 2,94h depois de ser tomado ou seja 2h e 56 minutos.</p><p>Como foi tomado às 9h00, vai começar a fazer efeito às 9h e 3m e deixa de fazer efeito</p><p>às 11h e 58m.</p><p>6.1.</p><p>O [ ]FAP∆ é um triângulo isósceles pois FPFA = pois são raios de uma mesma</p><p>circunferência.</p><p>Sendo M o ponto médio de [ ]AP , então pelo facto do triângulo ser isósceles a</p><p>altura [ ]FM bissecta o AFP∠ .</p><p>Assim,</p><p>2</p><p>ˆ α</p><p>=PFM .</p><p>Sendo [ ]FMP∆ um triângulo rectângulo temos:</p><p>2</p><p>10</p><p>1022</p><p>ααα</p><p>senPM</p><p>PM</p><p>sen</p><p>FP</p><p>PM</p><p>sen =⇔=⇔= .</p><p>Logo, </p><p></p><p></p><p></p><p></p><p></p><p>×=⇔</p><p></p><p></p><p></p><p></p><p></p><p>×=⇔=</p><p>2</p><p>20</p><p>2</p><p>1022</p><p>αα</p><p>senAPsenAPPMAP</p><p>6.2.</p><p>Sendo [ ]FAP∆ um triângulo isósceles temos .FPAFAP ∠≅∠</p><p>Assim,</p><p>2</p><p>ˆ απ −</p><p>=PAF .</p><p>Sabendo que</p><p>2</p><p>ˆ π</p><p>=BAF , pois FA é perpendicular à linha de costa, concluímos:</p><p>222</p><p>ˆ ααππ</p><p>=</p><p>−</p><p>−=BAP</p><p>Logo, no triângulo rectângulo [ ]PBA temos</p><p></p><p></p><p></p><p></p><p></p><p></p><p>=⇔</p><p></p><p></p><p></p><p></p><p></p><p></p><p>=⇔=</p><p>2</p><p>20</p><p>2</p><p>20</p><p>22</p><p>2 α</p><p>α</p><p>αα</p><p>senBP</p><p>sen</p><p>BP</p><p>sen</p><p>AP</p><p>BP</p><p>sen .</p><p>Portanto, ( ) </p><p></p><p></p><p></p><p></p><p></p><p>=</p><p>2</p><p>20 2 α</p><p>α send</p><p>V.S.F.F.735/1</p><p>PROVA 735/12 Págs.</p><p>EXAME NACIONAL DO ENSINO SECUNDÁRIO</p><p>10.º/11.º ou 11.º/12º Anos de Escolaridade</p><p>Decreto-Lei n.º 74/2004, de 26 de Março</p><p>Duração da prova: 150 minutos 2.ª FASE</p><p>2007</p><p>PROVA ESCRITA DE MATEMÁTICA B</p><p>735/2</p><p>Identifique claramente os itens a que responde.</p><p>Utilize apenas caneta ou esferográfica de tinta azul ou preta</p><p>(excepto nas respostas que impliquem a elaboração de</p><p>construções, desenhos ou outras representações).</p><p>É interdito o uso de «esferográfica-lápis» e de corrector.</p><p>As cotações da prova encontram-se na página 11.</p><p>A prova inclui um formulário (página 12).</p><p>V.S.F.F.735/3</p><p>Em todos os itens da prova, apresente o seu raciocínio de forma clara, indicando</p><p>todos os cálculos que tiver de efectuar e todas as justificações necessárias.</p><p>Apresente uma única resposta a cada item. Se escrever mais do que uma resposta,</p><p>deve indicar, de forma inequívoca, a que pretende que seja classificada (riscando</p><p>todas as que pretende anular).</p><p>Sempre que, na resolução de um problema, recorrer à sua calculadora, apresente</p><p>todos os elementos recolhidos na sua utilização. Mais precisamente:</p><p>• sempre que recorrer às capacidades gráficas da sua calculadora, apresente o</p><p>gráfico, ou gráficos, obtido(s), bem como coordenadas de pontos relevantes para a</p><p>resolução do problema proposto (por exemplo, coordenadas de pontos de</p><p>intersecção de gráficos, máximos, mínimos, etc.);</p><p>• sempre que recorrer a uma tabela obtida na sua calculadora, asapresente todas</p><p>linhas da tabela relevantes para a resolução do problema proposto;</p><p>• sempre que recorrer a estatísticas obtidas na sua calculadora (média, desvio</p><p>padrão, coeficiente de correlação, declive e ordenada na origem de uma recta de</p><p>regressão, etc.), as listas que introduziu na calculadora para as obter.apresente</p><p>735/4</p><p>1. A evolução da massa salarial de um conjunto de trabalhadores é, por vezes, explicável</p><p>através de modelos matemáticos.</p><p>Numa dada empresa, fez-se um estudo comparativo da evolução dos vencimentos (em</p><p>euros) de dois trabalhadores, e , entre 1998 e 2006.A B</p><p>• Relativamente ao trabalhador , o valor do vencimento mensal em cada ano, noA</p><p>período compreendido entre 1998 e 2006, é apresentado na tabela seguinte e</p><p>reproduzido num diagrama de dispersão.</p><p>Anos</p><p>Salário</p><p>1998 1999 2000 2001 2002 2003 2004 2005 2006</p><p>*!! *") *%# *&$ *&& *() "!!! "!"& "!%$</p><p>Evolução do salário do trabalhador A</p><p>• Relativamente ao trabalhador , sabe-se que, em 1998, recebia mensalmenteB</p><p>'&# euros e que, nos anos seguintes, referentes ao período em estudo, o valor</p><p>do seu vencimento mensal pode ser obtido através do modelo</p><p>@ œ '&# ‚ " !&!#8 , 8!"</p><p>Nota: a variável está associada aos anos relativos ao período em estudo,8</p><p>concretamente, corresponde a 1998, corresponde a 1999, etc.8 œ " 8 œ #</p><p>V.S.F.F.735/5</p><p>1.1. , indiqueUtilizando a sua calculadora um valor aproximado do coeficiente de</p><p>correlação linear entre as variáveis descritas na tabela (anos/salário) referente ao</p><p>trabalhador . Apresente o resultado com duas casas decimais.A</p><p>Interprete esse valor, tendo em conta o diagrama de dispersão correspondente.</p><p>1.2. Tome em atenção que o modelo que traduz a evolução do salário do trabalhador B</p><p>é uma progressão geométrica.</p><p>1.2.1. Indique o primeiro termo e a razão da progressão geométrica em questão.</p><p>1.2.2. Um trabalhador aufere, por ano, 12 ordenados mensais mais o subsídio</p><p>de férias e o décimo terceiro mês, ambos com valor igual ao do ordenado</p><p>mensal.</p><p>Utilizando a fórmula apropriada (que faz parte do formulário), calcule,</p><p>aproximadamente, o valor da totalidade dos vencimentos auferidos pelo</p><p>trabalhador entre 1998 e 2006, inclusive.B</p><p>Apresente o resultado arredondado às unidades.</p><p>Nota: Sempre que, em cálculos intermédios, proceder a</p><p>arredondamentos, conserve, no mínimo, duas casas decimais.</p><p>735/6</p><p>2. O campo de futebol de um dado clube tem uma bancada destinada a não sócios, que leva</p><p>% !!! "! espectadores. Se o preço de cada bilhete for euros, prevê-se que a lotação</p><p>dessa bancada fique esgotada.</p><p>Com base em experiências anteriores, verifica-se que, se o preço de cada bilhete for</p><p>aumentado numa certa percentagem, , sobre o valor base ( euros), o número deB "!</p><p>espectadores baixa metade dessa percentagem. Por exemplo, se o preço dos bilhetes</p><p>aumentar % , , o número de espectadores sofre um decréscimo de %."! B œ ! " &,</p><p>Admitindo a exactidão do modelo descrito e considerando sempre o aumento percentual,</p><p>B "!, sobre o preço base ( euros), responda às questões que se seguem.</p><p>2.1. Mostre que, se for o aumento percentual do preço de cada bilhete para aquelaB</p><p>bancada, num dado jogo, então a receita de bilheteira, , é dada por:V</p><p>V B œ 0 #! !!! B 1 #! !!! B 1 %! !!! ! Ÿ B Ÿ # ! # , com</p><p>Tenha em atenção que:</p><p>• o preço de cada bilhete, , em função do aumento percentual, , é dado: B</p><p>por :ÐBÑ œ "! Ð" 1 BÑ</p><p>• o número de espectadores, , em função do aumento percentual, é8 B,</p><p>dado por 8ÐBÑ œ % !!! 0 # !!! B</p><p>V.S.F.F.735/7</p><p>2.2. Um dos elementos da direcção do clube sugere que o preço de cada bilhete seja</p><p>de euros, para serem maximizadas as receitas de bilheteira. Porém, um#!</p><p>segundo elemento da direcção opõe-se, dizendo que o ideal é manter o preço de</p><p>cada bilhete a euros, uma vez que as receitas de bilheteira são superiores se"!</p><p>assim for.</p><p>Num pequeno texto, comente o argumento de cada um dos elementos da</p><p>direcção do clube, tendo em conta o objectivo de maximizar as receitas de</p><p>bilheteira.</p><p>Deve incluir, obrigatoriamente, na sua resposta:</p><p>• o valor da percentagem, , que a direcção do clube deve aplicar sobre oB</p><p>preço base ( euros), para que se maximizem as receitas de bilheteira,"!</p><p>e o respectivo valor da receita (no caso de discordar da opinião de cada</p><p>um dos elementos da direcção);</p><p>• um argumento, fundamentado, referente às propostas de cada um dos</p><p>elementos da direcção, dizendo se concorda, ou não, com elas;</p><p>• todos os elementos recolhidos na utilização</p>
<p>da sua calculadora gráfica</p><p>que se tenham mostrado relevantes.</p><p>2.3. À entrada para o recinto do jogo, cada espectador, sócio ou não sócio, recebeu um</p><p>cartão numerado para se habilitar a um sorteio. Estavam presentes ')#&</p><p>espectadores, dos quais % eram não sócios. Foram sorteados,%!</p><p>simultaneamente, dois números. Qual a probabilidade de ambos os contemplados</p><p>serem sócios?</p><p>Apresente o resultado final com aproximação às centésimas.</p><p>735/8</p><p>3. Numa determinada localidade, o responsável pelo planeamento urbanístico apresentou</p><p>uma proposta para a construção de uma rotunda com metros de diâmetro. No centro"!</p><p>da rotunda, pretende-se construir um jardim em forma de losango, com metros de#!</p><p>perímetro, como sugere a figura. À volta do jardim, serão colocados calçada e outros</p><p>elementos decorativos.</p><p>Relativamente à figura, considere que:</p><p>• os pontos e são os vértices do losango;Eß Fß G H</p><p>• o ponto é o centro da circunferência;S</p><p>• o ângulo tem de amplitude , EHS ! < <α α</p><p>1</p><p>#</p><p>3.1. Mostre que a área, em , da zona destinada ao jardim é dada, em função de ,7# α</p><p>por:</p><p>E œ &! -9= Þ =/8 ! < < !α α α α , 1</p><p>#</p><p>3.2. Determine .EŠ ‹1%</p><p>Interprete geometricamente o resultado obtido, indicando qual a forma particular do</p><p>losango, para α œ 1</p><p>%</p><p>V.S.F.F.735/9</p><p>4. No período de testes que antecedeu a entrada em funcionamento de um gasómetro, com</p><p>capacidade de toneladas, procedeu-se ao seu enchimento, continuamente, durante"!!</p><p>#% horas.</p><p>Por razões de segurança, o gasómetro foi lastrado com toneladas de gás, após o que# &,</p><p>se iniciou a operação de enchimento. A partir daí, o seu enchimento foi feito de acordo</p><p>com o modelo:</p><p>, sendo Q > œ ! "!!</p><p>"$$* / !ß%*> ! Ÿ > Ÿ #%</p><p>( representa a massa total, expressa em toneladas, existente no gasómetro horasQ ></p><p>desde o início do seu enchimento.)</p><p>Nota: Na resolução das questões seguintes, sempre que, em cálculos intermédios,</p><p>proceder a arredondamentos, conserve duas casas decimais.</p><p>4.1. Qual era a massa total, aproximada, existente no gasómetro horas após o início$</p><p>do seu enchimento?</p><p>Apresente o resultado arredondado às centésimas.</p><p>4.2. Durante o período em que decorre o enchimento do gasómetro, fará sentido afirmar</p><p>que existe um dado intervalo de tempo em que a taxa de variação média do modelo</p><p>assume um valor negativo?</p><p>Justifique devidamente a sua resposta.</p><p>735/10</p><p>5. Para vedar três canteiros circulares, com metros de raio cada, um agricultor decidiu%</p><p>colocar uma rede em forma de triângulo equilátero, como a figura sugere.Ò ÓEFG ,</p><p>Relativamente à figura, considere que:</p><p>• as circunferências são tangentes entre si;</p><p>• os lados do triângulo são tangentes às circunferências;</p><p>• os pontos e são os centros das circunferências;Lß M N</p><p>• é o ponto médio de ;K FG Ò Ó</p><p>• é ponto do lado ;H EG L Ò Ó tangente à circunferência de centro</p><p>• é ponto de tangência das circunferências de centros e respectivamente;P M N ,</p><p>• é a amplitude do ângulo .α HEL</p><p>Quantos metros da rede mencionada necessita, aproximadamente, o agricultor para vedar</p><p>os três canteiros?</p><p>Apresente o resultado final arredondado às unidades.</p><p>Nota: Sempre que, em cálculos intermédios, proceder a arredondamentos, conserve três</p><p>casas decimais.</p><p>Sugere-se que:</p><p>• determine a altura do triângulo ;Ò ÓLMN</p><p>• determine a altura do triângulo ;Ò ÓEFG</p><p>• determine o lado do triângulo .Ò ÓEFG</p><p>FIM</p><p>V.S.F.F.735/11</p><p>COTAÇÕES</p><p>1. 32 pontos...............................................................................</p><p>1.1 ................................................................12 pontos</p><p>1.2. ...............................................................20 pontos</p><p>1.2.1. .............................................. 8 pontos</p><p>1.2.2. ............................................ 12 pontos</p><p>2. 60 pontos...............................................................................</p><p>2.1. ...............................................................16 pontos</p><p>2.2. ...............................................................24 pontos</p><p>2.3. ...............................................................20 pontos</p><p>3. 44 pontos...............................................................................</p><p>3.1. .............................................................. 22 pontos</p><p>3.2. ...............................................................22 pontos</p><p>4. 40 pontos...............................................................................</p><p>4.1. .............................................................. 18 pontos</p><p>4.2. ...............................................................22 pontos</p><p>5. 24 pontos...............................................................................</p><p>TOTAL .................................................................... 200 pontos</p><p>735/12</p><p>Formulário</p><p>Comprimento de um arco de circunferência</p><p>α α< 0 < ( )amplitude, em radianos, do ângulo ao centro raio;</p><p>Áreas de figuras planas</p><p>Losango: H3+198+67+39<‚H3+198+67/89<</p><p>#</p><p>Trapézio: F+=/7+39<0F+=/7/89<</p><p># ‚E6>?<+</p><p>Polígono regular: Semiperímetro Apótema‚</p><p>Sector circular: α <</p><p>#</p><p># (α0 amplitude,</p><p>em radianos, do ângulo ao centro raio; < )</p><p>Áreas de superfícies</p><p>Área lateral de um cone: 1 < 1</p><p>( )< 1 raio da base geratriz;</p><p>Área de uma superfície esférica: % <1 #</p><p>( )< raio</p><p>Volumes</p><p>Pirâmide: "$ ‚ Área da base Altura‚</p><p>Cone: "$ ‚ Área da base Altura‚</p><p>Esfera: %$ $1 ( )< < raio</p><p>Progressões</p><p>Soma dos primeiros termos de uma8</p><p>Prog. Aritmética: ? $?</p><p>#</p><p>" 8 ‚ 8</p><p>Prog. Geométrica: ? ‚" "* <</p><p>"* <</p><p>8</p><p>V.S.F.F.</p><p>735/C/1</p><p>PROVA 735/C/8 Págs.</p><p>EXAME NACIONAL DO ENSINO SECUNDÁRIO</p><p>10.º/11.º ou 11.º/12.º Anos de Escolaridade</p><p>Decreto-Lei n.º 74/2004, de 26 de Março</p><p>Duração da prova: 150 minutos 2.ª FASE</p><p>2007</p><p>PROVA ESCRITA DE MATEMÁTICA B</p><p>COTAÇÕES</p><p>1. 32 pontos................................................................................................</p><p>1.1 ........................................................................12 pontos</p><p>1.2. .......................................................................20 pontos</p><p>1.2.1. ......................................................8 pontos</p><p>1.2.2. ....................................................12 pontos</p><p>2. 60 pontos................................................................................................</p><p>2.1. .......................................................................16 pontos</p><p>2.2. .......................................................................24 pontos</p><p>2.3. .......................................................................20 pontos</p><p>3. 44 pontos................................................................................................</p><p>3.1. ...................................................................... 22 pontos</p><p>3.2. .......................................................................22 pontos</p><p>4. 40 pontos................................................................................................</p><p>4.1. ...................................................................... 18 pontos</p><p>4.2. .......................................................................22 pontos</p><p>5. 24 pontos................................................................................................</p><p>TOTAL ..................................................................................... 200 pontos</p><p>735/C/2</p><p>CRITÉRIOS GERAIS DE CLASSIFICAÇÃO</p><p>1. Se o examinando se enganar na identificação do item a que está a responder, ou se a omitir,</p><p>mas, pela resolução apresentada, for possível identificá-lo inequivocamente, a resposta deve</p><p>ser vista e classificada.</p><p>2. Se o examinando responder ao mesmo item mais do que uma vez, deve eliminar</p><p>inequivocamente a(s) resposta(s) que não deve(m) ser classificada(s). No caso de tal não</p><p>acontecer, será classificada a resposta que surge em primeiro lugar.</p><p>3. Num item em que a respectiva resolução exija cálculos e/ou justificações, a classificação</p><p>deve ser:</p><p>• a soma algébrica das classificações atribuídas a cada</p>
<p>http://www.toomates.net/biblioteca/CompendiumAMC12.pdf</p><p>http://www.toomates.net/biblioteca/CompendiumAIME.pdf</p><p>http://www.toomates.net/biblioteca/CompendiumUSAJMO.pdf</p><p>http://www.toomates.net/biblioteca/CompendiumUSAMO.pdf</p><p>http://www.toomates.net/biblioteca/CompendiumTSTST.pdf</p><p>http://www.toomates.net/biblioteca/CompendiumTST.pdf</p><p>http://www.toomates.net/biblioteca/CompendiumELMO.pdf</p><p>http://www.toomates.net/biblioteca/CompendiumPutnam.pdf</p><p>http://www.toomates.net/biblioteca/CompendiumOME.pdf</p><p>http://www.toomates.net/biblioteca/CompendiumOMEFL.pdf</p><p>http://www.toomates.net/biblioteca/CompendiumOMEC.pdf</p><p>http://www.toomates.net/biblioteca/CompendiumOMEA.pdf</p><p>http://www.toomates.net/biblioteca/CompendiumOMEM.pdf</p><p>http://www.toomates.net/biblioteca/CompendiumCDP.pdf</p><p>http://www.toomates.net/biblioteca/CompendiumOMI.pdf</p><p>http://www.toomates.net/biblioteca/CompendiumArchimede.pdf</p><p>http://www.toomates.net/biblioteca/CompendiumHMMT.pdf</p><p>http://www.toomates.net/biblioteca/CompendiumBMO.pdf</p><p>http://www.toomates.net/biblioteca/CompendiumBalkanMO.pdf</p><p>http://www.toomates.net/biblioteca/CompendiumJBMO.pdf</p><p>http://www.toomates.net/biblioteca/CompendiumIMO.pdf</p><p>http://www.toomates.net/biblioteca/CompendiumIGO.pdf</p><p>http://www.toomates.net/biblioteca/CompendiumSMT.pdf</p><p>http://www.toomates.net/biblioteca/CompendiumINMO.pdf</p><p>http://www.toomates.net/biblioteca/CompendiumCMO.pdf</p><p>http://www.toomates.net/biblioteca/CompendiumHMMT.pdf</p><p>http://www.toomates.net/biblioteca2/contestproblembooks/TheContestProblemBook1.pdf</p><p>http://www.toomates.net/biblioteca2/contestproblembooks/TheContestProblemBook2.pdf</p><p>http://www.toomates.net/biblioteca2/contestproblembooks/TheContestProblemBook3.pdf</p><p>http://www.toomates.net/biblioteca2/contestproblembooks/TheContestProblemBook4.pdf</p><p>http://www.toomates.net/biblioteca2/contestproblembooks/TheContestProblemBook5.pdf</p><p>http://www.toomates.net/biblioteca2/contestproblembooks/TheContestProblemBook6.pdf</p><p>http://www.toomates.net/biblioteca2/contestproblembooks/TheContestProblemBook7.pdf</p><p>http://www.toomates.net/biblioteca2/contestproblembooks/TheContestProblemBook8.pdf</p><p>http://www.toomates.net/biblioteca2/contestproblembooks/TheContestProblemBook9.pdf</p><p>http://www.toomates.net/biblioteca2/PizzazzBookA.pdf</p><p>http://www.toomates.net/biblioteca2/PizzazzBookB.pdf</p><p>http://www.toomates.net/biblioteca2/PizzazzBookC.pdf</p><p>http://www.toomates.net/biblioteca2/PizzazzBookD.pdf</p><p>http://www.toomates.net/biblioteca2/PizzazzBookE.pdf</p><p>http://www.toomates.net/biblioteca2/Pizzazz_pre_Algebra.pdf</p><p>http://www.toomates.net/biblioteca2/pizzazz_algebra.pdf</p><p>http://www.toomates.net/biblioteca/CompendiumREOIM.pdf</p><p>http://www.toomates.net/</p><p>https://mega.nz/folder/l9RmRIya#VdMjbhXE7IOzKXCZOQD0QQ</p><p>https://www.youtube.com/c/GerardRomo</p><p>https://toomatesbloc.blogspot.com/</p><p>Este documento forma parte del siguiente bloque:</p><p>Información general:</p><p>http://www.toomates.net/biblioteca/Portugal.pdf</p><p>Enunciados y soluciones en español del 2024:</p><p>http://www.toomates.net/biblioteca/Portugal2024.pdf</p><p>A (635) 1997-2020:</p><p>http://www.toomates.net/biblioteca/Portugal635.pdf</p><p>A (635) 2021-2024:</p><p>http://www.toomates.net/biblioteca/Portugal635b.pdf</p><p>B (735) 2006-2020:</p><p>http://www.toomates.net/biblioteca/Portugal735.pdf</p><p>B (735) 2021-2024:</p><p>http://www.toomates.net/biblioteca/Portugal735b.pdf</p><p>MACS (835) 2021-2024:</p><p>http://www.toomates.net/biblioteca/Portugal835.pdf</p><p>Compendiums de pruebas PAU internacionales y privadas:</p><p>Portugal: http://www.toomates.net/biblioteca/Portugal.pdf</p><p>Italia: http://www.toomates.net/biblioteca/Italia.pdf</p><p>Francia: http://www.toomates.net/biblioteca/Francia.pdf</p><p>Pearson Edexcel International A Level: http://www.toomates.net/biblioteca/Edexcel.pdf</p><p>Cambridge International A Level: http://www.toomates.net/biblioteca/Cambridge.pdf</p><p>International Baccalaureate (IB): http://www.toomates.net/biblioteca/IB.pdf</p><p>Compendiums de pruebas PAU españolas:</p><p>Cataluña (TEC): http://www.toomates.net/biblioteca/Pautec.pdf</p><p>Cataluña (CCSS): http://www.toomates.net/biblioteca/Pauccss.pdf</p><p>Valencia: http://www.toomates.net/biblioteca/Valencia.pdf</p><p>Galicia: http://www.toomates.net/biblioteca/Galiciapau.pdf</p><p>País Vasco: http://www.toomates.net/biblioteca/Paisvascopau.pdf</p><p>Baleares: http://www.toomates.net/biblioteca/Balears.pdf</p><p>http://www.toomates.net/biblioteca/Portugal.pdf</p><p>http://www.toomates.net/biblioteca/Portugal2024.pdf</p><p>http://www.toomates.net/biblioteca/Portugal635.pdf</p><p>http://www.toomates.net/biblioteca/Portugal635b.pdf</p><p>http://www.toomates.net/biblioteca/Portugal735.pdf</p><p>http://www.toomates.net/biblioteca/Portugal735b.pdf</p><p>http://www.toomates.net/biblioteca/Portugal835.pdf</p><p>http://www.toomates.net/biblioteca/Portugal.pdf</p><p>http://www.toomates.net/biblioteca/Italia.pdf</p><p>http://www.toomates.net/biblioteca/Francia.pdf</p><p>http://www.toomates.net/biblioteca/Edexcel.pdf</p><p>http://www.toomates.net/biblioteca/CompendiumALevel1.pdf</p><p>http://www.toomates.net/biblioteca/IB.pdf</p><p>http://www.toomates.net/biblioteca/Pautec.pdf</p><p>http://www.toomates.net/biblioteca/Pauccss.pdf</p><p>http://www.toomates.net/biblioteca/Valencia.pdf</p><p>http://www.toomates.net/biblioteca/Galiciapau.pdf</p><p>http://www.toomates.net/biblioteca/Paisvascopau.pdf</p><p>http://www.toomates.net/biblioteca/Balears.pdf</p><p>Índice.</p><p>Año Página</p><p>2006 5</p><p>2007 61</p><p>2008 111</p><p>2009 153</p><p>2010 207</p><p>2011 270</p><p>2012 352</p><p>2013 449</p><p>2014 550</p><p>2015 617</p><p>2016 714</p><p>2017 810</p><p>2018 895</p><p>2019 975</p><p>2020 1041</p><p>2021 1090</p><p>2022 1138</p><p>Fuente principal:</p><p>https://mat.absolutamente.net/joomla/index.php/recursos/exames-e-testes-intermedios/matematica-a#1997</p><p>https://mat.absolutamente.net/joomla/index.php/recursos/exames-e-testes-intermedios/matematica-a#1997</p><p>2006</p><p>V.S.F.F.735/1</p><p>PROVA 735/11 Págs.</p><p>EXAME NACIONAL DO ENSINO SECUNDÁRIO</p><p>11.º Ano de Escolaridade (Decreto-Lei n.º 74/2004, de 26 de Março)</p><p>Curso Científico-Humanístico</p><p>de Artes Visuais</p><p>Duração da prova: 150 minutos 1.ª FASE</p><p>2006</p><p>PROVA ESCRITA DE MATEMÁTICA - B</p><p>735/2</p><p>Identifique claramente os grupos e os itens a que responde.</p><p>Utilize apenas caneta ou esferográfica de tinta azul ou preta</p><p>(excepto nas respostas que impliquem a elaboração de</p><p>construções, desenhos ou outras representações).</p><p>É interdito o uso de «esferográfica-lápis» e de corrector.</p><p>As cotações da prova encontram-se na página 10.</p><p>A prova inclui um formulário (pág. 11).</p><p>V.S.F.F.735/3</p><p>Em todas as questões da prova, apresente o seu raciocínio de forma clara,</p><p>indicando todos os cálculos que tiver de efectuar e todas as justificações</p><p>necessárias.</p><p>Apresente uma única resposta a cada item. Se escrever mais do que uma</p><p>resposta, deve indicar de forma inequívoca a que pretende que seja classificada</p><p>(riscando todas as que pretende anular).</p><p>Sempre que, na resolução de um problema, recorrer à sua calculadora, apresente</p><p>todos os elementos recolhidos na sua utilização. Mais precisamente:</p><p>• sempre que recorrer às capacidades gráficas da sua calculadora, apresente o</p><p>gráfico, ou gráficos, obtido(s), bem como coordenadas de pontos relevantes para</p><p>a resolução do problema proposto (por exemplo, coordenadas de pontos de</p><p>intersecção de gráficos, máximos, mínimos, etc.);</p><p>• sempre que recorrer a uma tabela obtida na sua calculadora, asapresente todas</p><p>linhas da tabela relevantes para a resolução do problema proposto;</p><p>• sempre que recorrer a estatísticas obtidas na sua calculadora (média, desvio</p><p>padrão, coeficiente de correlação, declive e ordenada na origem de uma recta de</p><p>regressão, etc.), as listas que introduziu na calculadora para as obter.apresente</p><p>735/4</p><p>V.S.F.F.735/5</p><p>1. A turma da Isabel decidiu fazer arranjos florais, utilizando flores do horto da escola, para</p><p>vender no Dia dos Namorados.</p><p>Idealizaram arranjos formados por margaridas, rosas e violetas.</p><p>Dispõem de: 192 margaridas, 88 rosas e 112 violetas.</p><p>Pensaram formar dois tipos de arranjos: A e B.</p><p>Cada arranjo do tipo A:</p><p>• será composto por 16 margaridas, 4 rosas e 8 violetas;</p><p>• dará um lucro de 3 euros.</p><p>Cada arranjo do tipo B:</p><p>• será</p>
<p>etapa, de acordo com o disposto</p><p>nos pontos 5, 6, 7, 8, 9, 10 e 11 destes critérios gerais. Se a soma for negativa, a</p><p>classificação a atribuir é de zero pontos;</p><p>• de zero pontos se o examinando se limitar a apresentar o resultado final.</p><p>4. Sempre que o examinando utilizar um processo de resolução não contemplado nos critérios</p><p>específicos, caberá ao professor classificador adoptar um critério de distribuição da cotação</p><p>que julgue adequado. Salienta-se que deve ser aceite qualquer processo cientificamente</p><p>correcto, mesmo que envolva conhecimentos ou competências não contemplados no</p><p>Programa da disciplina.</p><p>5. A cotação de cada item está subdividida pelas etapas que o examinando deve percorrer</p><p>para o resolver.</p><p>5.1. Em cada etapa, a cotação indicada é a máxima a atribuir.</p><p>5.2. O classificador não pode subdividir, em cotações parcelares, a cotação de cada etapa.</p><p>Caso uma etapa envolva um único passo, testando apenas o conhecimento de um só</p><p>conceito ou propriedade, e a sua resolução não esteja completamente correcta, deve</p><p>ser atribuída a classificação de zero pontos.</p><p>Caso uma etapa envolva mais do que um passo (por exemplo, a resolução de uma</p><p>equação, a obtenção de uma expressão em função de uma variável, etc.) e a sua</p><p>resolução esteja incompleta, ou contenha incorrecções, a classificação a atribuir deve</p><p>estar de acordo com o grau de incompletude e/ou com a gravidade dos erros</p><p>cometidos. Por exemplo:</p><p>• erros de contas ocasionais devem ser desvalorizados em um ponto;</p><p>• erros que revelem desconhecimento de conceitos, regras ou propriedades devem</p><p>ser desvalorizados em, pelo menos, metade da cotação da etapa;</p><p>• transposições erradas de dados do enunciado devem ser desvalorizadas em um</p><p>ponto, desde que o grau de dificuldade da etapa não diminua;</p><p>• transposições erradas de dados do enunciado devem ser desvalorizadas em, pelo</p><p>menos, metade da cotação da etapa, caso o grau de dificuldade da etapa diminua.</p><p>5.3. Nas etapas cuja cotação se encontra discriminada por níveis de desempenho, o</p><p>classificador deve enquadrar a resposta do examinando numa das descrições</p><p>apresentadas. O classificador não pode atribuir uma classificação diferente das</p><p>indicadas.</p><p>V.S.F.F.</p><p>735/C/3</p><p>5.4. No caso de o examinando cometer um erro numa das etapas, as etapas subsequentes</p><p>devem merecer a respectiva classificação, desde que o grau de dificuldade não tenha</p><p>diminuído, e o examinando as execute correctamente, de acordo com o erro que</p><p>cometeu.</p><p>5.5. Caso o examinando cometa, numa etapa, um erro que diminua o grau de dificuldade</p><p>das etapas subsequentes, cabe ao classificador decidir a classificação máxima a</p><p>atribuir a cada uma destas etapas. Em particular, se, devido a um erro cometido pelo</p><p>examinando, o grau de dificuldade das etapas seguintes diminuir significativamente, a</p><p>classificação máxima a atribuir em cada uma delas não deverá exceder metade da</p><p>cotação indicada.</p><p>5.6. Pode acontecer que o examinando, ao resolver um item, não percorra explicitamente</p><p>todas as etapas previstas nos critérios específicos. Todas as etapas não percorridas</p><p>explicitamente pelo examinando, mas cuja utilização e/ou conhecimento estejam</p><p>inequivocamente implícitos na resolução do item, devem receber a cotação indicada.</p><p>6. Nas etapas em que está previsto o recurso à calculadora, os critérios específicos</p><p>subdividem-se em: «Explicação do método utilizado» e «Apresentação do(s) valor(es)».</p><p>6.1. Explicação do método utilizado:</p><p>De acordo com as instruções gerais para a realização da prova, o examinando deve</p><p>apresentar todos os elementos recolhidos na utilização da calculadora. Esta</p><p>apresentação deve ser classificada de acordo com os níveis de desempenho que se</p><p>seguem. Em cada nível de desempenho, a classificação máxima prevista é a indicada</p><p>em percentagem.</p><p>Esta percentagem deve ser aplicada sobre a cotação prevista para a explicação do</p><p>método utilizado, e o valor obtido deve ser arredondado às unidades (por excesso, se</p><p>a mantissa do número a arredondar for 0,5 ou superior).</p><p>Apresentação correcta e completa de todos os elementos relevantes....................100%</p><p>Apresentação correcta, mas com ausência de alguns elementos relevantes</p><p>ou</p><p>Apresentação completa, mas com algumas incorrecções (por exemplo, não respeitar o</p><p>domínio de uma função) ........................................................................................... 70%</p><p>Apresentação incompleta e com algumas incorrecções .......................................... 40%</p><p>Ausência de explicação ou simples referências do tipo «Vi na calculadora»............ 0%</p><p>6.2. Apresentação do(s) valor(es):</p><p>Para cada valor que o examinando deve apresentar, os critérios específicos podem</p><p>indicar um intervalo admissível. O valor apresentado pelo examinando pode pertencer,</p><p>ou não, a esse intervalo.</p><p>• Se o valor pertencer ao intervalo, deve ser atribuída a cotação máxima prevista</p><p>para essa apresentação, a menos que haja lugar a qualquer desvalorização</p><p>prevista nos critérios específicos, por desrespeito do número de casas decimais</p><p>com que o resultado deve ser apresentado.</p><p>• Se o valor não pertencer ao intervalo, deve ser atribuída a classificação de zero</p><p>pontos.</p><p>735/C/4</p><p>7. Quando, num item, é pedida uma forma específica de apresentação do resultado final (por</p><p>exemplo, «em minutos», «em percentagem», etc.), este deve ser apresentado na forma</p><p>pedida. Se o resultado final apresentado pelo examinando não respeitar a forma pedida no</p><p>enunciado (por exemplo, se o enunciado pedir o resultado em minutos, e o examinando o</p><p>apresentar em horas), devem ser atribuídos zero pontos na etapa correspondente ao</p><p>resultado final. No entanto, a resposta não deve ser desvalorizada se não indicar a unidade</p><p>em que é pedido o resultado (por exemplo, se o resultado final for 12 minutos, ou 12 metros,</p><p>e o examinando escrever simplesmente 12, não deve existir qualquer desvalorização).</p><p>8. O examinando deve respeitar sempre a instrução relativa à apresentação de todos os</p><p>cálculos e de todas as justificações. Se, numa etapa, o examinando não respeitar esta</p><p>instrução, apresentando algo (valor, quadro, tabela, gráfico, etc.) que não resulte de trabalho</p><p>anterior, deve ser atribuída a classificação de zero pontos a essa etapa. Todas as etapas</p><p>subsequentes que dela dependam devem ser igualmente classificadas com zero pontos.</p><p>9. O examinando deve respeitar sempre qualquer instrução relativa ao método a utilizar na</p><p>resolução de um item (por exemplo, «equacione o problema», «resolva graficamente», etc.).</p><p>Na resolução apresentada pelo examinando, deve ser inequívoco, pela apresentação de</p><p>todos os cálculos e de todas as justificações, o cumprimento da instrução. Se tal não</p><p>acontecer, considera-se que o examinando não respeitou a instrução. A etapa em que se dá</p><p>o desrespeito e todas as subsequentes que dela dependam devem ser classificadas com</p><p>zero pontos.</p><p>10. Se, na resolução de um item, o examinando utilizar simbologia, ou escrever uma expressão,</p><p>inequivocamente incorrecta do ponto de vista formal (por exemplo, se escrever o símbolo de</p><p>igualdade onde deveria estar o símbolo de equivalência), a sua resposta deve ser</p><p>desvalorizada em um ponto. Esta desvalorização não se aplica no caso em que tais</p><p>incorrecções ocorram apenas em etapas classificadas com zero pontos, nem a eventuais</p><p>utilizações do símbolo de igualdade, onde, em rigor, deveria estar o símbolo de igualdade</p><p>aproximada.</p><p>11. Existem itens em cujo enunciado é dada uma instrução relativa ao número mínimo de casas</p><p>decimais que o examinando deve conservar, sempre que, em cálculos intermédios, proceder</p><p>a arredondamentos. Indicam-se, a seguir, as desvalorizações a aplicar, na classificação total</p><p>da resposta, em caso de desrespeito dessa instrução e/ou de arredondamentos mal</p><p>efectuados.</p><p>Todos os valores intermédios estão de acordo com a instrução, mas existe, pelo menos, um</p><p>valor intermédio mal arredondado................................................................................. -1 ponto</p><p>Todos os valores intermédios estão bem arredondados, mas existe, pelo menos, um que</p><p>não está</p>
<p>de acordo com a instrução............................................................................. -1 ponto</p><p>Existe, pelo menos, um valor intermédio mal arredondado e existe, pelo menos, um que não</p><p>está de acordo com a instrução ................................................................................. -2 pontos</p><p>12. As classificações a atribuir às respostas dos examinandos devem ser expressas,</p><p>obrigatoriamente, em números inteiros.</p><p>V.S.F.F.</p><p>735/C/5</p><p>Critérios específicos de classificação</p><p>1.1. ................................................................................................................... 12</p><p>Indicar o valor do coeficiente de correlação linear !< ¸ ! **,</p><p>(ver nota)............................................................................................................... 5</p><p>Justificar a adequabilidade do modelo ..................................................................7</p><p>Nota:</p><p>Como a tabela é apresentada no enunciado, não se exige que o</p><p>examinando transcreva para a folha de prova as listas que introduziu na sua</p><p>calculadora.</p><p>Se o examinando não respeitar o número de casas decimais, a classificação</p><p>da etapa deve ser desvalorizada em 1 ponto.</p><p>Se o examinando apresentar o valor , a classificação a atribuir nesta! *),</p><p>etapa deve ser de 4 pontos.</p><p>1.2.1. ................................................................................................................. 8</p><p>Indicar o primeiro termo ..............................................................................4 !'&#</p><p>Indicar a razão ....................................................................................... 4 !" !&!#,</p><p>1.2.2. ................................................................................................................ 12</p><p>Utilização correcta da fórmula Ð'&# ‚ ¸ ("*& #%"!"ß!&!#</p><p>"!"ß!&!#</p><p>*</p><p>, Ñ (ver</p><p>nota 1).................................................................................................................... 9</p><p>("*& #% ‚ "% ¸ "!!($$, ..............................................................3 (ver nota 2)</p><p>Notas:</p><p>1. Todos os resultados coerentes com uma resolução que respeite a</p><p>indicação, dada no enunciado, relativa ao número de casas decimais a</p><p>conservar nos cálculos intermédios, devem ser classificados com a cotação</p><p>total da etapa.</p><p>2. Se o examinando multiplicar por o valor referente à soma dos termos"#</p><p>da progressão geométrica, a classificação da etapa deve ser desvalorizada</p><p>em 1 ponto.</p><p>2.1. .................................................................................................................. 16</p><p>Escrever a igualdade ............................12 V B œ %!!! 1 #!!!B "! 2 "!B ! ! !</p><p>Obter a igualdade ...........................4 V B œ 1 #!!!!B 2 #!!!!B 2 %!!!! ! #</p><p>735/C/6</p><p>2.2. .................................................................................................................. 24</p><p>A composição deverá conter os seguintes tópicos:</p><p>• identificação do máximo e do maximizante da função ;(ver nota)</p><p>• comentário à proposta do primeiro elemento da direcção;</p><p>• comentário à proposta do segundo elemento da direcção;</p><p>• apresentação dos elementos recolhidos na utilização da calculadora (ver</p><p>nota).</p><p>Na tabela seguinte, indica-se como este item deve ser classificado:</p><p>Forma</p><p>Conteúdo</p><p>Nível 3 Nível 2 Nível 1</p><p>( ) ( ) ( )</p><p>A composição contempla corre</p><p>‡ ‡‡ ‡‡‡</p><p>ctamente 24 23 22</p><p>os quatro tópicos.</p><p>A composição contempla correctamente 18 17 16</p><p>três tópicos.</p><p>A composição contempla correctamente 12 11 10</p><p>dois tópicos.</p><p>A composição contempla correctamente 6 5 4</p><p>um tópico.</p><p>( ) Composição bem estruturada, sem erros de sintaxe, de‡ Nível 3</p><p>pontuação e/ou de ortografia.</p><p>( ) Composição razoavelmente estruturada, com alguns erros de‡‡ Nível 2</p><p>sintaxe, de pontuação e/ou de ortografia, cuja gravidade não</p><p>implique a perda de inteligibilidade e/ou de sentido.</p><p>( ) Composição sem estruturação aparente, com a presença de‡‡‡ Nível 1</p><p>erros graves de sintaxe, de pontuação e/ou de ortografia, com</p><p>perda frequente de inteligibilidade e/ou de sentido.</p><p>Nota: Se, na apresentação dos elementos recolhidos na utilização da</p><p>calculadora, o examinando não apresentar todos os elementos necessários, ou</p><p>se os apresentar com incorrecções, considera-se que a composição contempla</p><p>o tópico em questão, cabendo ao professor classificador considerar a</p><p>composição como sendo do ou do , de acordo com o grau deNível 2 Nível 1</p><p>incompletude e/ou a gravidade dos erros cometidos.</p><p>V.S.F.F.</p><p>735/C/7</p><p>2.3. .................................................................................................................. 20</p><p>Indicar que existem não sócios.................................................................... 3 #($!</p><p>Indicar que existem sócios........................................................................... 2 %!*&</p><p>Encontrar a probabilidade pedidaŒ #TÐ Ñ ¸«ambos serem sócios» ..................... 15 œ ! $'%!*&‚%!*%</p><p>')#&‚')#% , (ver nota)</p><p>Nota:</p><p>Se o examinando escrever «ambos serem sócios» ,TÐ Ñ œ %!*&</p><p>')#&‚')#%</p><p>#</p><p>a classificação a atribuir nesta etapa deve ser de 11 pontos.</p><p>Se o examinando escrever «ambos serem sócios» ,TÐ Ñ œ %!*&‚%!*%</p><p>')#&#a classificação a atribuir nesta etapa deve ser de 11 pontos.</p><p>Se o examinando escrever «ambos serem sócios» ,TÐ Ñ œ Š ‹%!*&</p><p>')#&</p><p>#</p><p>a classificação a atribuir nesta etapa deve ser de 8 pontos.</p><p>Se o examinando escrever «ambos serem sócios» ,TÐ Ñ œ %!*&</p><p>')#&a classificação a atribuir nesta etapa deve ser de 4 pontos.</p><p>3.1. .................................................................................................................. 22</p><p>Escrever .................................................................................4 -9= œ !α SH</p><p>&</p><p>Concluir que SH œ & -9=α ...............................................................................2</p><p>Concluir que FH œ "! -9=α ............................................................................ 2</p><p>Escrever .................................................................................4 =/8 œ !α SE</p><p>&</p><p>Concluir que SE œ &=/8α ...............................................................................2</p><p>Concluir que EG œ "! =/8α ............................................................................ 2</p><p>Concluir que E !α α αœ &! =/8 Þ -9= ..............................................................6</p><p>3.2. .................................................................................................................. 22</p><p>Escrever ................................................................ 4 E œŠ ‹1% &! =/8 Þ -9=1 1</p><p>% %</p><p>Determinar ............................................................................... 6 E œ #&7Š ‹1% #</p><p>Interpretar o resultado ........................................................................................ 12</p><p>735/C/8</p><p>4.1. ................................................................................................................... 18</p><p>Constatar que ............................................................................................. 3 > œ $</p><p>Escrever ...................................................................... 5 Q $ œ ! "!!</p><p>"+$* /"!ß%*‚$</p><p>Q $ ¸ "! !$ ! , (ver nota).................................................................................. 10</p><p>Nota: Se o examinando não respeitar o número de casas decimais, ou</p><p>indicar um valor mal arredondado a classificação da etapa deve ser</p><p>desvalorizada em 1 ponto.</p><p>4.2. .................................................................................................................. 22</p><p>Referir que a função é sempre crescente ...............................11 Q > ! (ver nota)</p><p>Para tal, o examinando pode invocar uma de duas alternativas:</p><p>• abordagem gráfica, via calculadora;</p><p>• caracterização do processo de enchimento.</p><p>Concluir que é sempre positiva ..................................11</p>
<p>X Z Q. . .Ò Ó+ ,; (ver nota)</p><p>Nota: Se o examinando tornar compreensível o seu raciocínio, mas não o fizer</p><p>de um modo totalmente claro, a classificação total a atribuir na etapa deverá</p><p>ser desvalorizada em 2 pontos.</p><p>5. ..................................................................................................................... 24</p><p>Determinar a altura do triângulo .............................................................. 6 ÒLMN Ó</p><p>Constatar que é um triângulo equilátero de lado ...................3 ÒLMN Ó )</p><p>Determinar ........................................................ 3 LP œ % $È ¸ ' *#),</p><p>Determinar a altura do triângulo ............................................................. 7 Ò EFGÓ</p><p>Determinar ..............................................................................4 EL œ )</p><p>Determinar ............................................3 EK œ "# 2 % $È ¸ ") *#),</p><p>Determinar o lado do triângulo ........................................ 6 Ò EFGÓ 6 ¸ #" )&' , !</p><p>Conclusão final (são necessários metros de rede)...........................................5 ''</p><p>Proposta de Resolução do Exame de Matemática B</p><p>Cod 735 – 2ª Fase 2007</p><p>1.1.</p><p>Introduzidos em duas listas da calculadora os valores de 1 a 9 correspondentes aos anos</p><p>e os valores dos salários e calculada a regressão linear obteve-se como coeficiente de</p><p>correlação r ≈ 0,99.</p><p>Verifica-se uma forte correlação entre a variação dos anos e o correspondente aumento</p><p>de salários.</p><p>1.2.1.</p><p>1.2.2.</p><p>Número de anos: 2006-1998+1=9</p><p>Total de vencimentos para um mês em cada ano:</p><p>9</p><p>9 9</p><p>1 1,0502</p><p>652 7195, 2442...</p><p>1 1,0502</p><p>S S</p><p>−</p><p>= × ⇔ =</p><p>−</p><p>Como em cada ano recebe 14 meses o valor total é 914 100733,4192S× = …</p><p>O valor total dos vencimentos durante os 9 anos é de cerca de 100733 euros.</p><p>2.1.</p><p>A receita é igual ao produto do número de bilhetes vendidos pelo preço de cada bilhete:</p><p>( ) ( ) ( ) ( ) .40000200002000020004000110 2</p><p>++−=⇔−×+= xxxRxxxR</p><p>2. 2.</p><p>Para que se maximizem as receitas de bilheteira o aumento deve ser de 50%, ou seja</p><p>x≈0, 5, obtendo-se uma receita de 45000 euros.</p><p>Esta conclusão pode tirar-se da observação do gráfico da função R(x) obtido na</p><p>calculadora com uma janela de visualização: [0,2]×[0,50000] e calculando o seu</p><p>máximo.</p><p>0502,1r</p><p>U</p><p>U</p><p>r</p><p>7304,648U</p><p>652U</p><p>1</p><p>2</p><p>2</p><p>1</p><p>=⇔=</p><p>=</p><p>=</p><p>Ora a opção de um dos directores de passar o preço de cada bilhete para 20 euros, sendo</p><p>o preço base de 10 euros, correspondia a um aumento de 100% ou seja x=1. Assim</p><p>obtinha-se uma receita de 40000 euros. Quanto ao outro elemento da direcção ao manter</p><p>o preço de 10 euros também obtinha uma receita de 40000 euros. Ou seja as propostas</p><p>são equivalentes mas não maximizam as receitas de bilheteira.</p><p>2.3.</p><p>Percentagem de sócios: 60%</p><p>Número de sócios: 6825×0,6 = 4095</p><p>4095 4094</p><p>( ) 0,3599...</p><p>6825 6824</p><p>P ambos sócios</p><p>×</p><p>= ≈</p><p>×</p><p>A probabilidade é de 0,36</p><p>3.1.</p><p>Sendo [ ]ABCD um losango então [ ] [ ]BDeAC são perpendiculares e bissectam-se.</p><p>Assim,</p><p>αsenOA 5= e αcos5=OD</p><p>αα cos1010 == BDesenAC</p><p>Logo,</p><p>( ) ( ) ααα</p><p>αα</p><p>α cos50</p><p>2</p><p>cos1010</p><p>senA</p><p>sen</p><p>A =⇔</p><p>×</p><p>= .</p><p>3.2.</p><p>225</p><p>4</p><p>cos</p><p>4</p><p>50</p><p>4</p><p>msenA ==</p><p></p><p></p><p></p><p></p><p> πππ</p><p>.</p><p>No editor de função introduzi a função )(αA e na janela de visualização [0,</p><p>2</p><p>π</p><p>]×[0,30]</p><p>obtive o seguinte gráfico:</p><p>Determinado o máximo desta função temos o ponto de coordenadas ( ≈ 0,7854;25)</p><p>sendo a primeira coordenada um valor aproximado de</p><p>4</p><p>π</p><p>.</p><p>A forma particular do losango é um quadrado.</p><p>Resolução alternativa:</p><p>Para</p><p>4</p><p>π</p><p>α = , vem</p><p>2</p><p>2</p><p>π</p><p>α = , e o losango é, em particular, um quadrado. Obtemos a área</p><p>do quadrado fazendo 2555 =×=× ll . Assim, o valor particular de ( ) 225mA =α obtido</p><p>para</p><p>4</p><p>π</p><p>α = , representa a área do quadrado de m5 de lado.</p><p>4.1.</p><p>( ) 03,10</p><p>391</p><p>100</p><p>3</p><p>349,0</p><p>≈</p><p>+</p><p>=</p><p>×−</p><p>e</p><p>M</p><p>4.2.</p><p>Após introduzir a função ( )tM obtemos o seguinte gráfico na janela de visualização</p><p>[0,24]×[0,150]:</p><p>Não pode existir um intervalo onde a taxa de variação média seja negativa porque a</p><p>função é crescente no seu domínio. Assim, em qualquer intervalo, a taxa de variação é</p><p>sempre positiva.</p><p>5.</p><p>Apresentamos duas resoluções possíveis.</p><p>1ª resolução</p><p>Se o triângulo [ABC] é equilátero, cada um dos</p><p>ângulos internos tem de amplitude 60º. Como a</p><p>altura [AG] divide o triângulo dado em dois</p><p>triângulos congruentes, º30=</p><p>∧</p><p>CAG . Considerando</p><p>o ponto auxiliar D’, do lado [AC] tangente à</p><p>circunferência de centro J, obtemos o rectângulo</p><p>D’</p><p>[DD’JH], pois os raios da circunferência são perpendiculares à tangente ([AC]) nos</p><p>pontos de tangência. Como 8=HJ , temos 8' =DD .</p><p>( )</p><p>928.6AD</p><p>º30tg</p><p>4</p><p>AD =⇔=</p><p>CDAD '= , logo 856.21856.138928.628AC =+=×+=</p><p>Para vedar os canteiros, o agricultor precisa de 66856.213 ≈× metros de rede.</p><p>2ª resolução</p><p>O [ ]HIJ∆ é equilátero pois cada um dos lados é constituído por dois raios de</p><p>circunferências iguais.</p><p>[HL] é altura deste triângulo pois une o vértice H ao ponto médio da base [IJ].</p><p>O [ ]HLJ∆ é rectângulo em L pois [HL] é uma altura.</p><p>48 == LJeHJ</p><p>4848 222</p><p>=⇔−= HLHL</p><p>O [ ]AHD∆ é rectângulo pois D é ponto do lado [ ]AC tangente à circunferência de</p><p>centro H.</p><p>Assim, 8</p><p>6</p><p>4</p><p>=⇔</p><p></p><p></p><p></p><p></p><p></p><p></p><p>= AH</p><p>sen</p><p>AH</p><p>π</p><p>.</p><p>Sendo 4=LG , por ser igual a um raio.</p><p>Temos que:</p><p>4812 +=⇔++= AGLGHLAHAG</p><p>O [ ]AGC∆ é rectângulo pois [AG] é altura do [ ]ABC∆ .</p><p>3</p><p>ˆ π</p><p>=GCA pois é um ângulo interno do [ ]ABC∆ .</p><p>Assim,</p><p>3</p><p>48224 +</p><p>=AC .</p><p>Perímetro do [ ]ABC∆ é igual a</p><p>72 6 48</p><p>3 65,569...</p><p>3</p><p>AC</p><p>+</p><p>× = =</p><p>O agricultor necessita de 66 metros de rede para vedar os três canteiros.</p><p>FIM</p><p>2008</p><p>Prova 735 • Página 1/ 8</p><p>EXAME NACIONAL DO ENSINO SECUNDÁRIO</p><p>Decreto-Lei n.º 74/2004, de 26 de Março</p><p>Prova Escrita de Matemática B</p><p>10.º/11.º anos ou 11.º/12.º anos de Escolaridade</p><p>Prova 735/1.ª Fase 8 Páginas</p><p>Duração da Prova: 150 minutos. Tolerância: 30 minutos</p><p>2008</p><p>Utilize apenas caneta ou esferográfica de tinta indelével azul ou preta, excepto nas respostas</p><p>que impliquem a elaboração de construções, desenhos ou outras representações, que podem</p><p>ser primeiramente elaboradas a lápis, sendo, a seguir, passadas a tinta.</p><p>Utilize a régua, o compasso, o esquadro, o transferidor e a calculadora gráfica sempre que</p><p>necessário.</p><p>Não é permitido o uso de corrector. Em caso de engano, deve riscar, de forma inequívoca,</p><p>aquilo que pretende que não seja classificado.</p><p>Escreva de forma legível a numeração dos grupos e/ou dos itens, bem como as respectivas</p><p>respostas.</p><p>Para cada item, apresente apenas uma resposta. Se escrever mais do que uma resposta a um</p><p>mesmo item, apenas é classificada a resposta apresentada em primeiro lugar.</p><p>Em todas as respostas, indique todos os cálculos que tiver de efectuar e todas as justificações necessárias.</p><p>Sempre que, na resolução de um problema, recorrer à sua calculadora, apresente todos os elementos</p><p>recolhidos na sua utilização. Mais precisamente:</p><p>• sempre que recorrer às capacidades gráficas da sua calculadora, apresente o gráfico, ou gráficos,</p><p>obtido(s), bem como as coordenadas de pontos relevantes para a resolução do problema proposto (por</p><p>exemplo, coordenadas de pontos de intersecção de gráficos, máximos, mínimos, etc.);</p><p>• sempre que recorrer a uma tabela obtida na sua calculadora, apresente todas as linhas da tabela</p><p>relevantes para a resolução do problema proposto;</p><p>• sempre que recorrer a estatísticas obtidas na sua calculadora (média, desvio-padrão, coeficiente de</p><p>correlação, declive e ordenada na origem de uma recta de regressão, etc.), apresente as listas que</p><p>introduziu na calculadora para as obter.</p><p>As cotações dos itens encontram-se na página 7.</p><p>A prova inclui um Formulário na página 8.</p><p>Prova 735 • Página 2/ 8</p><p>1. Pretende-se fazer um canteiro, no jardim de uma escola, com a forma de um quadrado de 7 metros de</p><p>lado.</p>
<p>Fig. 1</p><p>A figura 1 representa um projecto desse canteiro, designado por [ABCD ], em que a região sombreada</p><p>representa a zona que se pretende relvar, e o quadrado [EFGH] representa o local destinado a plantar</p><p>roseiras.</p><p>Tem-se, em metros:</p><p>1.1. Admita que x = 3. Pretende-se plantar 700 roseiras na zona reservada para esse efeito. Cada</p><p>roseira necessita de uma área quadrangular com 20 centímetros de lado.</p><p>Será possível plantar as 700 roseiras nessa zona? Justifique.</p><p>1.2. Mostre que a área, a, da região relvada, em metros quadrados, é dada, em função de x, por</p><p>Calcule a(0) e interprete o valor obtido no contexto da situação descrita.</p><p>2. Nos itens seguintes, considere a função , definida no intervalo [0, 7], no contexto</p><p>descrito no grupo de itens anterior.</p><p>2.1. Mostre que a taxa de variação média da função a, no intervalo [3, 4], é zero.</p><p>2.2. Do facto de a taxa de variação média da função a, no intervalo [3, 4], ser zero, podemos concluir</p><p>que a função a é constante no intervalo [3, 4]?</p><p>Justifique a sua resposta.</p><p>2.3. Atendendo ao orçamento existente, pretende-se que a zona relvada tenha a maior área possível.</p><p>Determine o valor de x para que tal aconteça.</p><p>( )</p><p>2</p><p>214 xxxa −=</p><p>( )</p><p>2</p><p>214 xxxa −=</p><p>AE FB GC HD x= = = =</p><p>� �</p><p>�</p><p>���</p><p>�</p><p>�</p><p>�</p><p>Prova 735 • Página 3/ 8</p><p>3. O «jogo da moedinha» consiste no seguinte: cada jogador (num conjunto de dois ou mais) esconde zero,</p><p>uma, duas ou três moedas, numa das suas mãos. Seguidamente, cada um dos jogadores tenta adivinhar</p><p>o número total de moedas «escondidas».</p><p>O David e o Pedro jogam com frequência o «jogo da moedinha». Admita que cada um deles escolhe,</p><p>aleatoriamente e com igual probabilidade, o número de moedas, entre zero e três, que vai esconder na</p><p>sua mão.</p><p>3.1. Seja Y a variável aleatória «número total de moedas escondidas pelo David e pelo Pedro».</p><p>Construa a tabela de distribuição de probabilidade da variável aleatória Y.</p><p>Indique se é mais provável que o número total de moedas escondidas pelo David e pelo Pedro seja</p><p>menor do que dois ou maior do que três.</p><p>3.2. Considere X a variável aleatória «número de vezes por semana que os dois amigos se encontram</p><p>para realizar o referido jogo».</p><p>Admita que a seguinte tabela corresponde à distribuição de probabilidade da variável X.</p><p>Determine o valor de a e calcule o valor médio da variável aleatória X.</p><p>4. Thomas Malthus, pensador do século XVIII, elaborou um modelo para prever a evolução da população</p><p>mundial. De acordo com este modelo, a população mundial duplicaria, de 25 anos em 25 anos.</p><p>Considerando que, no ano de 1900, a população mundial era de 1,65 mil milhões de pessoas, estime, de</p><p>acordo com o Modelo de Malthus, qual teria sido o valor da população mundial em 2000.</p><p>Apresente o resultado em milhares de milhões, arredondado às unidades.</p><p>X = x i 0 1 2 3 4</p><p>P(X = x i) 0,10 0,20 a 0,25 0,15</p><p>Prova 735 • Página 4/ 8</p><p>5. A população mundial, desde 1900, evoluiu de acordo com a tabela abaixo:</p><p>Admita que a evolução da população mundial desde 1900 é bem modelada por uma função exponencial</p><p>do número de pessoas, em que a variável independente designa o número de anos após 1900.</p><p>Estime a população mundial para 2010.</p><p>Recorra à calculadora e utilize a regressão exponencial para determinar a expressão de uma função</p><p>que se ajuste aos dados da tabela, percorrendo as seguintes etapas:</p><p>• considere o ano de 1900 como o ano zero (0), o ano de 1910 como o ano dez (10), e assim</p><p>sucessivamente até ao ano de 2000 como o ano cem (100);</p><p>• escreva essa expressão (apresente os valores numéricos envolvidos na expressão e fornecidos pela</p><p>calculadora, com quatro casas decimais);</p><p>• usando essa expressão, estime a população mundial para 2010 (apresente o resultado em milhares de</p><p>milhões de habitantes, arredondado às centésimas).</p><p>Ano</p><p>Número de pessoas</p><p>(em milhares de milhões)</p><p>1900 1,65</p><p>1910 1,75</p><p>1920 1,86</p><p>1930 2,07</p><p>1940 2,30</p><p>1950 2,56</p><p>1960 3,04</p><p>1970 3,71</p><p>1980 4,45</p><p>1990 5,28</p><p>2000 6,08</p><p>Prova 735 • Página 5/ 8</p><p>6. Na figura 2 está representado um pêndulo simples, E, oscilando no</p><p>plano DAB.</p><p>Quando um pêndulo oscila à superfície da Terra, o plano de oscilação</p><p>não se mantém fixo, vai rodando ao longo do tempo, em torno de um</p><p>eixo vertical, representado na figura por CD, devido ao movimento de</p><p>rotação da Terra. O tempo que decorre entre o início da oscilação do</p><p>pêndulo e o momento em que o plano de oscilação do pêndulo</p><p>completa uma rotação de 360º designa-se por período. Este período</p><p>não é o mesmo em todos os lugares da Terra, pois depende da latitude</p><p>do lugar em que se realiza a experiência. Vamos considerar apenas</p><p>lugares do hemisfério norte.</p><p>A relação entre o período, T, medido em horas, e a latitude do lugar, q,</p><p>medida em graus, estabelecida por Jean Foucault (1819-1868), em</p><p>1851, é:</p><p>(Lei do seno de Foucault)</p><p>6.1. Mostre que, no Pólo Norte, o pêndulo tem um período de 24 horas. Recorde que a latitude no Pólo</p><p>Norte é de 90º.</p><p>6.2. A latitude de Paris, onde Foucault realizou a experiência que confirmou a referida lei, é,</p><p>aproximadamente, de 49º.</p><p>O João declarou ter feito uma experiência semelhante à de Foucault, nas mesmas condições, tendo</p><p>obtido o valor de 48 horas para o período do pêndulo.</p><p>Num pequeno texto e usando apenas a lei do seno de Foucault:</p><p>• indique o período que Foucault terá registado na sua experiência de 1851;</p><p>• indique a latitude do local em que o João terá feito a sua experiência;</p><p>• comente, fundamentadamente, a possibilidade de a experiência do João poder ter sido realizada</p><p>em Portugal Continental, sabendo que Portugal Continental está compreendido entre,</p><p>aproximadamente, as latitudes 36º e 42º.</p><p>FIM</p><p>( )</p><p>T</p><p>sen q</p><p>24=</p><p>Prova 735 • Página 6/ 8</p><p>�</p><p>�</p><p>�</p><p>�</p><p>�</p><p>�</p><p>�</p><p>Fig. 2</p><p>COTAÇÕES</p><p>1. .................................................................................................................................................. 30 pontos</p><p>1.1. ...................................................................................................................... 10 pontos</p><p>1.2. ...................................................................................................................... 20 pontos</p><p>2. .................................................................................................................................................. 60 pontos</p><p>2.1. ...................................................................................................................... 20 pontos</p><p>2.2. ...................................................................................................................... 20 pontos</p><p>2.3. ...................................................................................................................... 20 pontos</p><p>3. .................................................................................................................................................. 40 pontos</p><p>3.1. ...................................................................................................................... 20 pontos</p><p>3.2. ...................................................................................................................... 20 pontos</p><p>4. .................................................................................................................................................. 20 pontos</p><p>5. .................................................................................................................................................. 20 pontos</p><p>6. .................................................................................................................................................. 30 pontos</p><p>6.1. ...................................................................................................................... 10 pontos</p><p>6.2. ...................................................................................................................... 20 pontos</p><p>______________</p><p>TOTAL .............................................................. 200 pontos</p><p>Prova 735</p>
<p>• Página 7/ 8</p><p>Formulário</p><p>Comprimento de um arco de circunferência</p><p>α r (α – amplitude, em radianos, do ângulo ao centro; r – raio)</p><p>Áreas de figuras planas</p><p>Losango:</p><p>Trapézio: × Altura</p><p>Polígono regular: Semiperímetro × Apótema</p><p>Sector circular: (α – amplitude, em radianos, do ângulo ao centro; r – raio)</p><p>Áreas de superfícies</p><p>Área lateral de um cone: π r g (r – raio da base; g – geratriz)</p><p>Área de uma superfície esférica: 4 π r2 (r – raio)</p><p>Volumes</p><p>Pirâmide: × Área da base × Altura</p><p>Cone: × Área da base × Altura</p><p>Esfera: π r3 (r – raio)</p><p>Progressões</p><p>Soma dos n primeiros termos de uma</p><p>Progressão aritmética: × n</p><p>Progressão geométrica: u1 ×</p><p>1 – rn</p><p>———–</p><p>1 – r</p><p>u1 + un</p><p>———–</p><p>2</p><p>4—</p><p>3</p><p>1—</p><p>3</p><p>1—</p><p>3</p><p>α r2</p><p>——</p><p>2</p><p>Base maior + Base menor————————————</p><p>2</p><p>Diagonal maior × Diagonal menor</p><p>————————————————</p><p>2</p><p>Prova 735 • Página 8/ 8</p><p>Prova 735 • Página C/1/ 9</p><p>EXAME NACIONAL DO ENSINO SECUNDÁRIO</p><p>Decreto-Lei n.º 74/2004, de 26 de Março</p><p>Prova Escrita de Matemática B</p><p>10.º/11.º anos ou 11.º/12.º anos de Escolaridade</p><p>Prova 735/1.ª Fase 9 Páginas</p><p>Duração da Prova: 150 minutos. Tolerância: 30 minutos</p><p>2008</p><p>COTAÇÕES</p><p>1. ................................................................................................................................ 30 pontos</p><p>1.1. ..................................................................................................... 10 pontos</p><p>1.2. ..................................................................................................... 20 pontos</p><p>2. ................................................................................................................................ 60 pontos</p><p>2.1. ..................................................................................................... 20 pontos</p><p>2.2. ..................................................................................................... 20 pontos</p><p>2.3. ..................................................................................................... 20 pontos</p><p>3. ................................................................................................................................ 40 pontos</p><p>3.1. ..................................................................................................... 20 pontos</p><p>3.2. ..................................................................................................... 20 pontos</p><p>4. ................................................................................................................................ 20 pontos</p><p>5. ................................................................................................................................ 20 pontos</p><p>6. ................................................................................................................................ 30 pontos</p><p>6.1. ..................................................................................................... 10 pontos</p><p>6.2. ..................................................................................................... 20 pontos</p><p>__________________________</p><p>TOTAL ......................................... 200 pontos</p><p>CRITÉRIOS GERAIS DE CLASSIFICAÇÃO DA PROVA</p><p>As classificações a atribuir às respostas são expressas em números inteiros não negativos.</p><p>Os critérios de classificação dos itens apresentam-se organizados por etapas e/ou por níveis de</p><p>desempenho. A cada nível de desempenho e a cada etapa corresponde uma dada pontuação.</p><p>Os critérios de classificação dos itens de resposta aberta extensa orientada apresentam-se organizados por</p><p>níveis de desempenho. Nesses itens, desde que os mesmos tenham cotação igual ou superior a quinze</p><p>pontos e impliquem a produção de um texto, a classificação a atribuir traduz a avaliação simultânea das</p><p>competências específicas da disciciplina e das competências de comunicação escrita em língua portuguesa.</p><p>A avaliação das competências de comunicação escrita em língua portuguesa pode contribuir para valorizar a</p><p>classificação atribuída ao desempenho do aluno, ao nível das competências específicas da disciplina. Esta</p><p>valorização pode atingir cerca de 10% da cotação do item e faz-se de acordo com os níveis de desempenho</p><p>a seguir descritos.</p><p>Nível Descritor</p><p>3</p><p>Composição bem estruturada, sem erros de sintaxe, de pontuação e/ou de ortografia, ou</p><p>com erros esporádicos cuja gravidade não implique a perda de inteligibilidade e/ou de rigor</p><p>de sentido.</p><p>2</p><p>Composição razoavelmente estruturada, com alguns erros de sintaxe, de pontuação e/ou de</p><p>ortografia cuja gravidade não implique a perda de inteligibilidade e/ou de sentido.</p><p>1</p><p>Composição sem estruturação aparente, com a presença de erros graves de sintaxe, de pon-</p><p>tuação e/ou de ortografia cuja gravidade implique a perda frequente de inteligibilidade e/ou</p><p>de sentido.</p><p>Prova 735 • Página C/2/ 9</p><p>No quadro seguinte, apresentam-se critérios de classificação a aplicar às respostas aos itens em situações</p><p>não consideradas anteriormente.</p><p>Situação Classificação</p><p>1. Engano na identificação do item a que o examinando</p><p>está a responder.</p><p>Deve ser vista e classificada a resposta se, pela resolução</p><p>apresentada, for possível identificar inequivocamente o</p><p>item.</p><p>2. Omissão da identificação do item a que o examinando</p><p>está a responder.</p><p>3. É apresentada mais do que uma resposta ao mesmo</p><p>item, sem o examinando indicar, de forma inequívoca,</p><p>aquela que pretende que seja classificada.</p><p>Deve ser vista e classificada apenas a resposta que surge</p><p>em primeiro lugar, na folha de respostas.</p><p>4. É apresentado apenas o resultado final, embora a</p><p>resolução do item exija cálculos e/ou justificações.</p><p>Deve ser atribuída a classificação de zero pontos.</p><p>5. Ilegibilidade da resposta. Deve ser atribuída a classificação de zero pontos.</p><p>6. Item com etapas. A cotação indicada em cada etapa é a classificação máxima</p><p>atribuível à resposta.</p><p>A classificação da resposta ao item resulta da soma das</p><p>classificações das diferentes etapas, à qual, eventualmente,</p><p>se subtraem um ou dois pontos, de acordo com o previsto</p><p>nas situações 15 e 19.</p><p>7. Etapa com passos. A cotação indicada em cada passo é a classificação máxima</p><p>atribuível à resposta.</p><p>A classificação na etapa resulta da soma das classificações</p><p>dos diferentes passos.</p><p>8. Item ou etapa cuja cotação se encontra discriminada por</p><p>níveis de desempenho.</p><p>O classificador deve enquadrar a resposta do examinando</p><p>numa das descrições apresentadas, não podendo atribuir</p><p>uma classificação diferente das cotações indicadas.</p><p>9. Utilização de processos de resolução do item não</p><p>previstos nos critérios específicos.</p><p>Deve ser aceite qualquer processo de resolução</p><p>cientificamente correcto, ainda que não esteja previsto nos</p><p>critérios específicos de classificação ou no programa.</p><p>O critério específico deve ser adaptado ao processo de</p><p>resolução apresentado, mediante distribuição da cotação</p><p>do item pelas etapas percorridas pelo examinando. Esta</p><p>adaptação do critério deve ser utilizada em todos os</p><p>processos de resolução análogos.</p><p>10. Não são apresentadas, explicitamente, todas as etapas,</p><p>mas a resolução apresentada permite perceber,</p><p>inequivocamente, que elas foram percorridas.</p><p>A(s) etapa(s) implícita(s) é(são) classificada(s) com a</p><p>cotação total para ela(s) prevista.</p><p>11. Transposição incorrecta do enunciado. Se o grau de dificuldade de resolução da etapa não diminuir,</p><p>deve subtrair-se um ponto à classificação da etapa.</p><p>Se o grau de dificuldade de resolução da etapa diminuir, a</p><p>classificação máxima a atribuir nessa etapa não deve ser</p><p>superior a 50% da cotação prevista.</p><p>12. Erro ocasional num cálculo. Deve subtrair-se um ponto à classificação na etapa em que</p><p>ocorre o erro.</p><p>13. Erro que revela desconhecimento de conceitos, de</p><p>regras ou de propriedades.</p><p>A classificação máxima a atribuir a essa etapa não deve ser</p><p>superior a 50% da cotação prevista para a mesma.</p><p>Prova 735 • Página C/3/ 9</p><p>Situação Classificação</p><p>14. Erro na resolução de uma etapa. A resolução dessa etapa deve ser classificada de acordo</p><p>com o erro cometido.</p>
<p>Se o erro não diminuir o grau de dificuldade de resolução</p><p>das etapas subsequentes, estas devem ser classificadas de</p><p>acordo com os critérios de classificação.</p><p>Se o erro diminuir o grau de dificuldade de resolução das</p><p>etapas subsequentes, a classificação máxima a atribuir a</p><p>essas etapas não deve ser superior a 50% da cotação</p><p>prevista.</p><p>15. Em cálculos intermédios, é pedida uma aproximação a</p><p>um número de casas decimais. O examinando não</p><p>respeita o pedido, e/ou os arredondamentos estão</p><p>incorrectos.</p><p>Deve subtrair-se um ponto à classificação total da</p><p>resposta.</p><p>16. A apresentação do resultado final não respeita a forma</p><p>solicitada. [Exemplo: é pedido o resultado em centíme-</p><p>tros e o examinando apresenta-o em metros.]</p><p>Subtrair um ponto à classificação da resposta na etapa</p><p>correspondente ao resultado final.</p><p>17. Na apresentação do resultado final, não está expressa</p><p>a unidade de medida. [Exemplo: «15» em vez de «15</p><p>metros».]</p><p>Na etapa relativa ao resultado final, a resposta é</p><p>classificada tal como se a unidade de medida estivesse</p><p>indicada.</p><p>18. O resultado final apresenta um número de casas</p><p>decimais diferente do solicitado e/ou está</p><p>incorrectamente arredondado.</p><p>Deve subtrair-se um ponto à classificação da resposta na</p><p>etapa correspondente ao resultado final.</p><p>19. Utilização de simbologias ou de expressões</p><p>inequivocamente incorrectas, do ponto de vista formal.</p><p>Deve subtrair-se um ponto à classificação total da</p><p>resposta, excepto:</p><p>– se as incorrecções ocorrerem nas etapas já classificadas</p><p>com zero pontos;</p><p>– no caso de uso do símbolo de igualdade onde, em rigor,</p><p>deveria ter sido usado o símbolo de igualdade</p><p>aproximada.</p><p>Prova 735 • Página C/4/ 9</p><p>CRITÉRIOS ESPECÍFICOS DE CLASSIFICAÇÃO</p><p>1.1. ................................................................................................................................................. 10 pontos</p><p>Este item pode ser resolvido por, pelo menos, dois processos:</p><p>1.º processo</p><p>Determinar o lado do quadrado [EFGH ] .... (5 m) ....................................................... 4</p><p>Determinar a área do quadrado [EFGH ] .... (25 m2) ................................................... 1</p><p>Determinar a área do quadrado necessário para plantar cada roseira .... (0,04 m2) .... 2</p><p>Determinar a área dos 700 quadrados (28 m2)............................................................... 2</p><p>Conclusão .... (impossibilidade) ........................................................................................ 1</p><p>2.º processo</p><p>Determinar da área do quadrado [EFGH ] .... (5 m) ..................................................... 4</p><p>Determinar a área do quadrado [EFGH ] .... (25 m2) ................................................... 1</p><p>Determinar a área do quadrado necessário para plantar cada roseira .... (0,04 m2) .... 2</p><p>Determinar o quociente .... .... (625) .................................................................... 2</p><p>Conclusão .... (impossibilidade) ........................................................................................ 1</p><p>1.2. ................................................................................................................................................. 20 pontos</p><p>Este item pode ser resolvido por, pelo menos, dois processos:</p><p>1.º processo</p><p>Determinar uma expressão que dá a área do quadrado [EFGH ] .... (x 2 + (7 – x )2).... 5</p><p>Simplificar a expressão anterior .... (2x 2 – 14x + 49) ...................................................... 4</p><p>Determinar a expressão que dá a área da região relvada</p><p>(49 – (2x 2 – 14x + 49) ou 14x – 2x 2)...................................................................... 5</p><p>Calcular a (0) ................................................................................................................... 2</p><p>Interpretar o valor obtido ................................................................................................... 4</p><p>(«Quando x é zero, todo o canteiro passa a ser destinado a plantar roseiras, pelo que deixa</p><p>de existir zona destinada a ser relvada».)</p><p>2.º processo</p><p>Determinar uma expressão que dá a área de cada triângulo rectângulo .. .... 7</p><p>Determinar a expressão que dá a área da região relvada 7</p><p>Calcular a (0) ................................................................................................................... 2</p><p>Interpretar o valor obtido ................................................................................................... 4</p><p>( )x x x x2</p><p> 7 − 4× = 14 −2   2</p><p>( )x x 7 −    2</p><p>,</p><p>25</p><p>0 04</p><p>Prova 735 • Página C/5/ 9</p><p>2.1. ................................................................................................................................................. 20 pontos</p><p>Escrever a expressão que dá a taxa de variação média no intervalo [3, 4] .................... 8</p><p>Calcular a (3) ................................................................................................................... 5</p><p>Calcular a (4) ................................................................................................................... 5</p><p>Calcular a taxa de variação média ................................................................................... 2</p><p>2.2. ................................................................................................................................................. 20 pontos</p><p>Este item pode ser resolvido por, pelo menos, três processos:</p><p>1.º processo</p><p>Calcular a (x0) para algum x0 ∈ ]3, 4[ ........................................................................... 10</p><p>Referir que a função não é constante em [3, 4] ............................................................... 8</p><p>Referir que a conclusão não pode ser retirada.................................................................. 2</p><p>2.º processo</p><p>Esboçar o gráfico da função a ......................................................................................... 10</p><p>Referir que a função não é constante em [3, 4] ............................................................... 8</p><p>Referir que a conclusão não pode ser retirada ................................................................. 2</p><p>3.º processo</p><p>Exibir um contra-exemplo: função real de variável real, definida pelo examinando, com</p><p>taxa de variação média nula num intervalo do seu domínio, e não constante ................. 20</p><p>2.3. ................................................................................................................................................. 20 pontos</p><p>Este item pode ser resolvido por, pelo menos, três processos:</p><p>1.º processo</p><p>Recorrendo à calculadora gráfica:</p><p>Apresentar o gráfico de a (ver nota 1) ............................................................................. 15</p><p>Indicar o maximizante da função (ver nota 2) .................................................................. 5</p><p>Notas:</p><p>1. Caso o gráfico apresentado não respeite o domínio da função a , a classificação nesta etapa</p><p>deverá ser, no máximo, dez pontos.</p><p>2. Não deve ser desvalorizada uma resposta que difira, no máximo, numa décima de 3,5. Qualquer</p><p>outra resposta deve ser classificada com zero pontos.</p><p>2.º processo</p><p>Determinar a taxa de variação da função ......................................................................... 10</p><p>Determinar maximizante ................................................................................................ 10</p><p>Prova 735 • Página C/6/ 9</p><p>3.º processo</p><p>Referir que o gráfico de a é uma parábola com a concavidade voltada para baixo ......... 5</p><p>Concluir que o vértice da parábola tem abcissa 3,5 ......................................................... 10</p><p>Concluir que 3,5 é o maximizante da função .................................................................... 5</p><p>3.1. ................................................................................................................................................. 20 pontos</p><p>Construir a tabela de distribuição de probabilidade (ver nota)</p>
<p>......................................... 14</p><p>Estudar a comparação de probabilidades ........................................................................ 6</p><p>Indicar a probabilidade de o número total de moedas “escondidas”</p><p>ser menor do que dois .... ............................................................ 2</p><p>Indicar a probabilidade de o número total de moedas “escondidas”</p><p>ser maior do que três .... ............................................................. 2</p><p>Concluir ................................................................................................. 2</p><p>Nota: Cada coluna é classificada com 2 pontos (1 ponto para cada linha).</p><p>No caso de o examinando construir uma tabela de dupla entrada para melhor calcular as</p><p>probabilidades, esta tabela deve ser classificada com 7 pontos e cada coluna da tabela de</p><p>distribuição deve ser classificada com 1 ponto.</p><p>3.2. ................................................................................................................................................. 20 pontos</p><p>Calcular o valor de a .... (0,30) ........................................................................................ 10</p><p>Calculo de valor médio da variável aleatória .... (2,15) .................................................... 10</p><p>4. .................................................................................................................................................... 20 pontos</p><p>Este item pode ser resolvido por, pelo menos, dois processos:</p><p>1.º processo</p><p>Estabelecer a progressão geométrica (an) (cujos termos, para além do primeiro termo,</p><p>correspondem ao valor previsto para a população mundial de 25 em 25 anos a partir de</p><p>1900):</p><p>Primeiro termo (a1 = 1,65) ................................................................................................ 2</p><p>Razão (r = 2) .................................................................................................................... 2</p><p>Termo geral (an = 1,65 × 2n–1 ou equivalente) ............................................................... 8</p><p>Termo correspondente ao ano 2000 .................................................................................... 8</p><p>n = 5 ................................................................................................... 4</p><p>a5 = 26,4 ............................................................................................ 3</p><p>a5 ≈ 26 ................................................................................................ 1</p><p>Nota: Se o examinando não explicitar o termo geral, mas calcular o quinto termo correctamente,</p><p>devem ser atribuídos os 8 pontos desta etapa.</p><p>6    16</p><p>3    16</p><p>Prova 735 • Página C/7/ 9</p><p>2.º processo</p><p>Construir uma tabela com os resultados sucessivos ........................................................... 19</p><p>Concluir que a população prevista para 2000 era aproximadamente de 26 mil milhões .... 1</p><p>5. .................................................................................................................................................... 20 pontos</p><p>Determinar uma expressão (ver nota) ................................................................................ 15</p><p>Valor previsto para 2010 .... (valor no intervalo [6,46; 6,49]) ......................................... 5</p><p>Nota: De acordo com o modelo de calculadora, a regressão exponencial pode ser expressa, por</p><p>exemplo, por a × ebt , ou por a × bt .</p><p>Para a expressão a × ebt , obtemos a = 1,4471 e b = 0,0136.</p><p>Para a expressão a × bt , obtemos a = 1,4471 e b = 1,0137.</p><p>6.1. ................................................................................................................................................. 10 pontos</p><p>Substituir q por 90º .......................................................................................................... 5</p><p>Calcular o período ............................................................................................................ 5</p><p>6.2. ................................................................................................................................................. 20 pontos</p><p>Apresenta-se, a seguir, um exemplo de resposta:</p><p>«O valor do período na experiência de Foucault é aproximadamente de 32 horas,</p><p>porque se tem . O valor da latitude na experiência do João é de</p><p>30º, porque . Como o território de</p><p>Portugal Continental se situa entre 36º e 42º de latitude, resulta que o João não realizou</p><p>a sua experiência nesse terrritório.»</p><p>Tal como o exemplo ilustra, a composição deverá contemplar os seguintes tópicos:</p><p>• valor do período na experiência de Foucault;</p><p>• valor da latitude na experiência do João;</p><p>• impossibilidade de um período de 48 horas para o território de Portugal Continental.</p><p>ºT senq q</p><p>sen q</p><p>24 1= 48 ⇔ = 48 ⇔ = ⇔ = 30</p><p>2</p><p>º</p><p>T</p><p>sen</p><p>24= ≈ 32</p><p>49</p><p>ANO</p><p>POPULAÇÃO PREVISTA</p><p>(em mil milhões)</p><p>1900 1,65</p><p>1925 3,3</p><p>1950 6,6</p><p>1975 13,2</p><p>2000 26,4</p><p>Prova 735 • Página C/8/ 9</p><p>Na tabela seguinte, indica-se como deve ser classificado este item, de acordo com os</p><p>níveis de desempenho no domínio da comunicação escrita em língua portuguesa descritos</p><p>nos critérios gerais e os níveis de desempenho no domínio específico da disciplina:</p><p>Notas:</p><p>1. Apenas podem ser atribuídas classificações correspondentes a um dos valores constantes do</p><p>quadro. Não há lugar a classificações intermédias. No caso de a resposta não atingir o nível 1 de</p><p>desempenho no domínio específico da disciplina, a classificação a atribuir é zero pontos. Neste</p><p>caso, não é classificado o desempenho no domínio da comunicação escrita em língua portuguesa.</p><p>2. O examinando pode apresentar o valor de arredondado ou não às unidades.</p><p>3. O examinando pode resolver a equação T = 48, graficamente.</p><p>4. O examinando pode calcular os valores do período para as latitudes de 36º e 42º para concluir a</p><p>impossibilidade da realização da experiência do João em Portugal Continental.</p><p>5. Se o examinando se enganar no cálculo da latitude na experiência do João, deve-se considerar</p><p>que a composição não contempla o segundo tópico. O tópico seguinte deve ser analisado de</p><p>acordo com o erro cometido. Em particular, o erro poderá levar o examinando a concluir pela</p><p>possibilidade da realização da experiência do João no território de Portugal Continental. No</p><p>entanto, em caso algum se pode concluir pela certeza dessa realização. Se o examinando</p><p>expressar tal conclusão, deve-se considerar que a composição não contempla o terceiro tópico.</p><p>ºsen</p><p>24</p><p>49</p><p>Descritores do nível de desempenho no domínio</p><p>da comunicação escrita em língua portuguesa</p><p>Descritores do nível de desempenho</p><p>no domínio específico da disciplina</p><p>Níveis</p><p>1 2 3</p><p>N</p><p>ív</p><p>e</p><p>is</p><p>3 A composição contempla, correctamente, os três tópicos. 18 19 20</p><p>2 A composição contempla, correctamente, dois tópicos. 12 13 14</p><p>1 A composição contempla, correctamente, um tópico. 6 7 8</p><p>Prova 735 • Página C/9/ 9</p><p>PROPOSTA DE RESOLUÇÃO DA PROVA</p><p>MATEMÁTICA B (735 – 23 de Junho)</p><p>1.</p><p>1.1.</p><p>Se 3== FBAE então 4=AF</p><p>Logo o lado do quadrado que designamos por l é:</p><p>52543 2222</p><p>==+=+= AFAEl</p><p>Assim a área A do quadrado é:</p><p>225cmA =</p><p>A área total ocupada pelas roseiras é:</p><p>2</p><p>2</p><p>2870004,0</p><p>04,02,02,0</p><p>m</p><p>m</p><p>=×</p><p>=×</p><p>Donde, a área necessária para plantar as roseiras é superior à área disponível pelo que</p><p>não é possível plantar as 700 roseiras na zona referida.</p><p>1.2.</p><p>Se xFBAE == e 7=AB temos que xAF −= 7</p><p>Logo, o lado do quadrado é:</p><p>( ) 4914214497 22222 +−=+−+=−+ xxxxxxx</p><p>Pelo que para a área A do quadrado temos:</p><p>49142 2 +−= xxA</p><p>A área do quadrado [ABCD] menos a área do quadrado [EFGH] dá a área relvada.</p><p>Área relvada = xxxx 1424914249 22 +−=−+−</p><p>Calculemos :)0(a</p><p>( ) 0020140 2 =×−×=a</p><p>Sendo o valor igual a 0 significa que não existe área relvada.</p><p>2.</p><p>2.1.</p><p>Calculemos a taxa de variação no intervalo indicado:</p><p>( ) ( )</p><p>0</p><p>1</p><p>3231442414</p><p>34</p><p>34 22</p><p>=×+×−×−×=</p><p>−</p><p>− aa</p><p>c.q.d.</p><p>2.2.</p><p>Não, pois teria de ser zero em cada intervalo</p>
<p>contido em [ ]4,3 .</p><p>2.3.</p><p>Como a função tem um máximo para o valor da abcissa do vértice da parábola, vamos</p><p>calcular esse valor.</p><p>Calculemos os zeros da função quadrática:</p><p>700214 2 =∨=⇔=− xxxx</p><p>A abcissa do vértice é o ponto médio dos zeros:</p><p>Abcissa do vértice:</p><p>2</p><p>7</p><p>2</p><p>70 =+</p><p>Logo, a zona a relvar tem o maior valor possível para</p><p>2</p><p>7=x .</p><p>3.</p><p>3.1.</p><p>Na tabela seguinte estão representados os casos possíveis:</p><p>Pedro</p><p>David</p><p>0</p><p>1</p><p>2</p><p>3</p><p>0 0 1 2 3</p><p>1 1 2 3 4</p><p>2 2 3 4 5</p><p>3 3 4 5 6</p><p>Designando por iy o número total de moedas obtemos a seguinte distribuição de</p><p>probabilidade:</p><p>iy 0 1 2 3 4 5 6</p><p>( )iyP 16</p><p>1</p><p>8</p><p>1</p><p>16</p><p>3</p><p>4</p><p>1</p><p>16</p><p>3</p><p>8</p><p>1</p><p>16</p><p>1</p><p>Estudemos as probabilidades indicadas:</p><p>P(“sair menor que 2”)=P(0)+P(1)= .</p><p>16</p><p>3</p><p>8</p><p>1</p><p>16</p><p>1 =+</p><p>P(“sair maior que 3”)=</p><p>8</p><p>3</p><p>16</p><p>1</p><p>8</p><p>1</p><p>16</p><p>3 =++ .</p><p>Como</p><p>16</p><p>3</p><p>8</p><p>3 > , pois numa fracção com o mesmo numerador é maior a que tiver menor</p><p>denominador. Podemos concluir que é maior provável que o número de moedas seja</p><p>superior a 3.</p><p>3.2.</p><p>Como a massa de probabilidade é 1 temos que:</p><p>0,10+0,20+a+0,25+0.15=1⇔ a=0,30</p><p>O valor médio da variável aleatória é:</p><p>15,215,0425,0330,0220,0110,00 =×+×+×+×+×</p><p>4.</p><p>Se a população duplica de 25 em 25 anos estamos perante uma progressão geométrica</p><p>de razão 2.</p><p>2=r</p><p>Designemos por nU o seu termo geral, assim:</p><p>1265,1 −×= n</p><p>nU</p><p>Para saber a ordem do termo correspondente a 100 anos:</p><p>100:25=4</p><p>Como 1900 corresponde ao 1º termo o termo correspondente ao ano 2000 é o 5º termo:</p><p>O valor de 5U é:</p><p>40,26265,1 5</p><p>4</p><p>5 =⇔×= UU</p><p>Portanto, o número de habitantes estimado pelo Modelo de Malthus é de 26,40 mil</p><p>milhões de pessoas.</p><p>Ou então:</p><p>1900 temos 1,65 mil milhões de habitantes</p><p>1925 temos 3,3 mil milhões de habitantes</p><p>1950 temos 6,6 mil milhões de habitantes</p><p>1975 temos 13,2 mil milhões de habitantes</p><p>2000 temos 26,4 mil milhões de habitantes</p><p>5.</p><p>Introduzindo os dados nas listas da calculadora como indicado no enunciado obtemos:</p><p>0 1,65</p><p>10 1,75</p><p>20 1,86</p><p>30 2,07</p><p>40 2,30</p><p>50 2,56</p><p>60 3,04</p><p>70 3,71</p><p>80 4,45</p><p>90 5,28</p><p>100 6,08</p><p>Utilizando a regressão exponencial obtive para xba × os valores:</p><p>a = 1,447144298</p><p>b = 1,013730948</p><p>donde a expressão pedida é:</p><p>x0137,14471,1 ×</p><p>O valor estimado da população mundial será em 2010 obtido para o valor x=110 é de</p><p>6,4645 milhares de milhões</p><p>6.1.</p><p>Sabendo que a latitude do Pólo Norte é de 90º, temos que º90=q .</p><p>Donde, pela Lei dos senos de Foucault:</p><p>horasT</p><p>sen</p><p>T 24</p><p>º90</p><p>24 =⇔=</p><p>O que prova que o período é de 24 horas.</p><p>6.2.</p><p>O período registado pela experiência de Foucault para uma latitude de</p><p>aproximadamente 49º é de:</p><p>8,31</p><p>7547,0</p><p>24</p><p>º49</p><p>24 =⇔=⇔= TT</p><p>sen</p><p>T horas</p><p>Para o João obter um período de 48 horas a latitude foi, segundo a Lei do seno de</p><p>Foucault, de:</p><p>º30</p><p>2</p><p>1</p><p>48</p><p>2424</p><p>48 =⇔=⇔=⇔= qqsenqsen</p><p>psen</p><p>.</p><p>Como a latitude de Portugal Continental está compreendida entre 36º e 42º, o João não</p><p>poderia ter realizado a experiência em Portugal Continental pois a latitude em que a</p><p>realizou é de 30º.</p><p>Prova 735 • Página 1/ 8</p><p>EXAME NACIONAL DO ENSINO SECUNDÁRIO</p><p>Decreto-Lei n.º 74/2004, de 26 de Março</p><p>Prova Escrita de Matemática B</p><p>10.º/11.º anos ou 11.º/12.º anos de Escolaridade</p><p>Prova 735/2.ª Fase 8 Páginas</p><p>Duração da Prova: 150 minutos. Tolerância: 30 minutos</p><p>2008</p><p>Utilize apenas caneta ou esferográfica de tinta indelével azul ou preta, excepto nas respostas</p><p>que impliquem a elaboração de construções, desenhos ou outras representações, que podem</p><p>ser primeiramente elaboradas a lápis, sendo, a seguir, passadas a tinta.</p><p>Utilize a régua, o compasso, o esquadro, o transferidor e a calculadora gráfica sempre que</p><p>necessário.</p><p>Não é permitido o uso de corrector. Em caso de engano, deve riscar, de forma inequívoca,</p><p>aquilo que pretende que não seja classificado.</p><p>Escreva de forma legível a numeração dos grupos e/ou dos itens, bem como as respectivas</p><p>respostas.</p><p>Para cada item, apresente apenas uma resposta. Se escrever mais do que uma resposta a um</p><p>mesmo item, apenas é classificada a resposta apresentada em primeiro lugar.</p><p>Em todas as respostas, indique todos os cálculos que tiver de efectuar e todas as justificações necessárias.</p><p>Sempre que, na resolução de um problema, recorrer à sua calculadora, apresente todos os elementos</p><p>recolhidos na sua utilização. Mais precisamente:</p><p>• sempre que recorrer às capacidades gráficas da sua calculadora, apresente o gráfico, ou gráficos,</p><p>obtido(s), bem como as coordenadas de pontos relevantes para a resolução do problema proposto (por</p><p>exemplo, coordenadas de pontos de intersecção de gráficos, máximos, mínimos, etc.);</p><p>• sempre que recorrer a uma tabela obtida na sua calculadora, apresente todas as linhas da tabela</p><p>relevantes para a resolução do problema proposto;</p><p>• sempre que recorrer a estatísticas obtidas na sua calculadora (média, desvio-padrão, coeficiente de</p><p>correlação, declive e ordenada na origem de uma recta de regressão, etc.), apresente as listas que</p><p>introduziu na calculadora para as obter.</p><p>As cotações dos itens encontram-se na página 7.</p><p>A prova inclui um Formulário na página 8.</p><p>Prova 735 • Página 2/ 8</p><p>1. Numa região montanhosa, pretendia-se abrir um túnel em linha recta, unindo dois locais à mesma</p><p>altitude. Devido à escassez de meios, seguiu-se um processo que era usado na Grécia Antiga.</p><p>No esquema da figura 1, que não está à escala, a</p><p>região sombreada representa a montanha, e o</p><p>segmento [AF ] o túnel. Este esquema ilustra o</p><p>processo utilizado: sempre à mesma altitude, uma</p><p>equipa técnica deslocou-se 750 metros para leste</p><p>do ponto A, até ao ponto B ; do ponto B,</p><p>deslocou-se 450 metros para norte, até ao ponto</p><p>C, e assim sucessivamente, até ao ponto F, tal</p><p>como está indicado na figura.</p><p>No fim deste processo, a equipa decidiu-se a usar coordenadas cartesianas, para saber que direcção</p><p>deveriam tomar as escavações.</p><p>Para esse efeito, imaginou o referencial com origem</p><p>em A, indicado na figura 2. A unidade usada nos</p><p>eixos foi o metro.</p><p>Tendo em conta este referencial, responda aos</p><p>seguintes itens.</p><p>1.1. Indique as coordenadas dos pontos assina-</p><p>lados na figura (A, B, C, D, E, F).</p><p>1.2. Determine a equação reduzida da recta AF.</p><p>Prova 735 • Página 3/ 8</p><p>� ��� �</p><p>���</p><p>���</p><p>���</p><p>��</p><p>� ����</p><p>�</p><p>�</p><p>��</p><p>� � ��� �</p><p>���</p><p>���</p><p>���</p><p>��</p><p>� ���� �</p><p>Fig. 1</p><p>Fig. 2</p><p>2. Numa piscicultura, existe um tanque que tem actualmente 300 robalos. Ao serem introduzidas x trutas</p><p>no tanque, a proporção P(x ) do número de trutas, relativamente ao número total de peixes que</p><p>passam a existir no tanque, é tal que .</p><p>2.1. A equação P(x) = 1 é impossível.</p><p>Interprete esta impossibilidade no contexto da situação descrita.</p><p>2.2. Pretende-se que a percentagem de trutas, relativamente ao número total de peixes, seja de 25%.</p><p>Qual é o número de trutas a introduzir no tanque?</p><p>3. Admita agora que, no tanque, existem 300 robalos e 200 trutas.</p><p>3.1. Vai ser pescado, ao acaso, um peixe do tanque. Admita que cada peixe tem igual probabilidade de</p><p>ser pescado.</p><p>Qual é a probabilidade de se pescar um robalo?</p><p>3.2. Foram retirados do tanque doze robalos. Os valores dos respectivos comprimentos e pesos são os</p><p>que constam da seguinte tabela.</p><p>Recorrendo à calculadora, determine o coeficiente de correlação linear entre as variáveis a e p,</p><p>arredondado às centésimas.</p><p>Interprete o valor obtido, tendo em conta a nuvem de pontos que pode visualizar na calculadora.</p><p>4. Numa pequena cidade foi colocado, em lugar de destaque,</p><p>um painel publicitário alusivo às ofertas turísticas da região.</p><p>4.1. O painel tem um mecanismo que faz accionar um ponto</p><p>luminoso (ponto P ), que descreve uma circunferência de</p><p>centro O, com cinco metros de raio, tal como a figura 3</p><p>sugere.</p><p>Sejam:</p><p>• θ a amplitude, em graus, do ângulo orientado cujo lado</p><p>origem é a semi-recta O</p><p>•</p><p>A e cujo lado extremidade é a</p><p>semi-recta O</p><p>•</p><p>P ;</p><p>• ;</p><p>• h a distância do ponto luminoso à base do painel.</p><p>OB=7</p><p>Comprimento</p><p>a (em mm)</p>
<p>157 165 168 159 172 165 166 163 159 169 171 168</p><p>Peso p (em g) 52 61 67 60 70 65 66 62 58 72 72 68</p><p>( )</p><p>x</p><p>P x</p><p>x</p><p>=</p><p>300 +</p><p>Prova 735 • Página 4/ 8</p><p>�</p><p>�</p><p>�</p><p>�</p><p>�</p><p>�</p><p>�</p><p>�</p><p>�</p><p>�</p><p>Fig. 3</p><p>Comece por completar a tabela seguinte, relativa a várias posições do ponto P, ao longo de uma volta.</p><p>De seguida, mostre que, para 0º < θ < 90º, a distância, h, expressa em metros, do ponto luminoso à</p><p>base do painel, é dada, em função de θ , por</p><p>4.2. Um gabinete de publicidade turística está a projectar um painel no qual figuram dez circunferências</p><p>com o mesmo centro.</p><p>Conforme o projecto, a primeira circunferência terá 3 metros de raio, a segunda terá 3,10 metros de</p><p>raio e assim sucessivamente, de acordo com uma progressão aritmética de razão 0,10 metros. Com</p><p>o objectivo de fazer realçar o painel, à noite, pretende-se que cada uma destas dez circunferências</p><p>fique coberta com fio luminoso.</p><p>Quantos metros de fio luminoso serão necessários para executar o projecto?</p><p>Apresente o resultado arredondado às centésimas. Nos valores intermédios, use sempre, pelo</p><p>menos, três casas decimais.</p><p>5. Numa determinada região do interior, as chuvas torrenciais causaram inundações, e a região foi</p><p>considerada zona de catástrofe. Os prejuízos acentuaram-se muito nas actividades agrícolas. Para</p><p>enfrentar esta situação, os organismos ligados aos serviços agro-pecuários decidiram adquirir rações</p><p>para animais. Foram pedidos, com urgência, dois tipos de ração: FarX e FarY.</p><p>A FARJO é uma fábrica especializada na produção destes tipos de ração. Estas rações contêm três</p><p>aditivos: vitaminas, sabores e conservantes.</p><p>Por cada tonelada de ração do tipo FarX, são necessários dois quilogramas de vitaminas, um quilograma</p><p>de sabores e um quilograma de conservantes.</p><p>Por cada tonelada de ração do tipo FarY, são necessários um quilograma de vitaminas, dois quilogramas</p><p>de sabores e três quilogramas de conservantes.</p><p>A FARJO dispõe, diariamente, de 16 quilogramas de vitaminas, 11 quilogramas de sabores e 15</p><p>quilogramas de conservantes. Estas são as únicas restrições na produção destas rações.</p><p>Represente por x a quantidade de ração FarX produzida diariamente, expressa em toneladas, e por y a</p><p>quantidade de ração FarY produzida diariamente, expressa em toneladas.</p><p>5.1. É possível a FARJO fabricar, num só dia, 4 toneladas de FarX e 3 toneladas de FarY?</p><p>Justifique.</p><p>5.2. Quais são as quantidades de ração de cada tipo que devem ser produzidas, de modo que a</p><p>quantidade total de ração produzida diariamente seja máxima?</p><p>Percorra, sucessivamente, as seguintes etapas:</p><p>• indique as restrições do problema;</p><p>• indique a função objectivo;</p><p>• represente graficamente a região admissível, referente ao sistema de restrições;</p><p>• indique os valores das variáveis para os quais é máxima a função objectivo.</p><p>( )h senθ θ= 7 + 5</p><p>θ 0º 90º 180º 270º 360º</p><p>h</p><p>Prova 735 • Página 5/ 8</p><p>6. Sabe-se que Leonardo da Vinci (1456-1519) também se interessava por Matemática. Numa melancólica</p><p>nota sobre a noite de 30 de Novembro de 1504, escreveu o seguinte, numa caligrafia regular e da direita</p><p>para a esquerda (como costumava):</p><p>«Na noite de Santo André, encontrei a solução para a quadratura do círculo, quando se acabavam a</p><p>candeia, a noite e o papel em que estava a escrever. Terminei-a de manhã».</p><p>Durante anos e anos, procuraram-se, entre os infindáveis cadernos que nos deixou, os manuscritos</p><p>contendo as reflexões feitas naquela noite. Em vão: nunca foram encontrados.</p><p>Nas férias da Páscoa de 2008, o Manuel foi passar uns dias a casa dos avós. Vasculhando coisas velhas</p><p>no sótão, encontrou uns papéis corroídos pelo tempo, escritos em italiano antigo e também numa</p><p>caligrafia regular, da direita para a esquerda: pareciam ser o tão procurado caderno de Leonardo sobre a</p><p>quadratura do círculo. Ficou espantado.</p><p>Imagine que o Manuel lhe pede a si que estude a possibilidade de a autoria dos papéis ser de Leonardo</p><p>da Vinci.</p><p>Admita que, numa aula de Matemática B, aprendeu que a massa de carbono 14 (C14), presente num</p><p>artefacto desde a sua produção, é dada pela fórmula</p><p>y(t) = c e–0,000121 t</p><p>em que c é a massa original de C14, em gramas, e t é o tempo, em anos, decorrido desde o momento</p><p>da produção do artefacto.</p><p>Decidiu, por isso, recorrer a um laboratório científico especializado em análises de C14, que o informou do</p><p>seguinte: o manuscrito contém 96% da massa de C14 original, ou seja, designando por c a massa de</p><p>C14 original, a massa de C14 que o manuscrito contém é de 0,96 c.</p><p>Com base nesta informação, redija um pequeno texto para o Manuel, no qual constem:</p><p>• a idade do papel (em número inteiro de anos);</p><p>• a data (em anos) em que terá sido fabricado;</p><p>• a conclusão quanto à possibilidade de Leonardo da Vinci ser o autor do manuscrito.</p><p>FIM</p><p>Prova 735 • Página 6/ 8</p><p>COTAÇÕES</p><p>1. .................................................................................................................................................. 30 pontos</p><p>1.1. ...................................................................................................................... 12 pontos</p><p>1.2. ...................................................................................................................... 18 pontos</p><p>2. .................................................................................................................................................. 40 pontos</p><p>2.1. ...................................................................................................................... 20 pontos</p><p>2.2. ...................................................................................................................... 20 pontos</p><p>3. .................................................................................................................................................. 40 pontos</p><p>3.1. ...................................................................................................................... 20 pontos</p><p>3.2. ...................................................................................................................... 20 pontos</p><p>4. .................................................................................................................................................. 40 pontos</p><p>4.1. ...................................................................................................................... 20 pontos</p><p>4.2. ...................................................................................................................... 20 pontos</p><p>5. .................................................................................................................................................. 30 pontos</p><p>5.1. ...................................................................................................................... 10 pontos</p><p>5.2. ...................................................................................................................... 20 pontos</p><p>6. .................................................................................................................................................. 20 pontos</p><p>______________</p><p>TOTAL .............................................................. 200 pontos</p><p>Prova 735 • Página 7/ 8</p><p>Formulário</p><p>Comprimento de um arco de circunferência</p><p>α r (α – amplitude, em radianos, do ângulo ao centro; r – raio)</p><p>Áreas de figuras planas</p><p>Losango:</p><p>Trapézio: × Altura</p><p>Polígono regular: Semiperímetro × Apótema</p><p>Sector circular: (α – amplitude, em radianos, do ângulo ao centro; r – raio)</p><p>Áreas de superfícies</p><p>Área lateral de um cone: π r g (r – raio da base; g – geratriz)</p><p>Área de uma superfície esférica: 4 π r2 (r – raio)</p><p>Volumes</p><p>Pirâmide: × Área da base × Altura</p><p>Cone: × Área da base × Altura</p><p>Esfera: π r3 (r – raio)</p><p>Progressões</p><p>Soma dos n primeiros termos de uma</p><p>Progressão aritmética: × n</p><p>Progressão geométrica: u1 ×</p><p>1 – rn</p><p>———–</p><p>1 – r</p><p>u1 + un</p><p>———–</p><p>2</p><p>4—</p><p>3</p><p>1—</p><p>3</p><p>1—</p><p>3</p><p>α r2</p><p>——</p><p>2</p><p>Base maior + Base menor————————————</p><p>2</p><p>Diagonal maior × Diagonal menor</p><p>————————————————</p>
<p>2</p><p>Prova 735 • Página 8/ 8</p><p>Prova 735 • Página C/1/ 9</p><p>EXAME NACIONAL DO ENSINO SECUNDÁRIO</p><p>Decreto-Lei n.º 74/2004, de 26 de Março</p><p>Prova Escrita de Matemática B</p><p>10.º/11.º anos ou 11.º/12.º anos de Escolaridade</p><p>Prova 735/2.ª Fase 9 Páginas</p><p>Duração da Prova: 150 minutos. Tolerância: 30 minutos</p><p>2008</p><p>COTAÇÕES</p><p>1. ................................................................................................................................ 30 pontos</p><p>1.1. ..................................................................................................... 12 pontos</p><p>1.2. ..................................................................................................... 18 pontos</p><p>2. ................................................................................................................................ 40 pontos</p><p>2.1. ..................................................................................................... 20 pontos</p><p>2.2. ..................................................................................................... 20 pontos</p><p>3. ................................................................................................................................ 40 pontos</p><p>3.1. ..................................................................................................... 20 pontos</p><p>3.2. ..................................................................................................... 20 pontos</p><p>4. ................................................................................................................................ 40 pontos</p><p>4.1. ..................................................................................................... 20 pontos</p><p>4.2. ..................................................................................................... 20 pontos</p><p>5. ................................................................................................................................ 30 pontos</p><p>5.1. ..................................................................................................... 10 pontos</p><p>5.2. ..................................................................................................... 20 pontos</p><p>6. ................................................................................................................................ 20 pontos</p><p>__________________________</p><p>TOTAL ......................................... 200 pontos</p><p>CRITÉRIOS GERAIS DE CLASSIFICAÇÃO DA PROVA</p><p>As classificações a atribuir às respostas são expressas em números inteiros não negativos.</p><p>Os critérios de classificação dos itens apresentam-se organizados por etapas e/ou por níveis de</p><p>desempenho. A cada nível de desempenho e a cada etapa corresponde uma dada pontuação.</p><p>Os critérios de classificação dos itens de resposta aberta extensa orientada apresentam-se organizados por</p><p>níveis de desempenho. Nestes itens, desde que tenham cotação igual ou superior a quinze pontos e</p><p>impliquem a produção de um texto, a classificação a atribuir traduz a avaliação simultânea das competências</p><p>específicas da disciciplina e das competências de comunicação escrita em língua portuguesa.</p><p>A avaliação das competências de comunicação escrita em língua portuguesa pode contribuir para valorizar a</p><p>classificação atribuída ao desempenho ao nível das competências específicas da disciplina. Esta valorização</p><p>pode atingir cerca de 10% da cotação do item e faz-se de acordo com os níveis de desempenho a seguir</p><p>descritos:</p><p>Nível Descritor</p><p>3</p><p>Composição bem estruturada, sem erros de sintaxe, de pontuação e/ou de ortografia, ou</p><p>com erros esporádicos cuja gravidade não implique a perda de inteligibilidade e/ou de rigor</p><p>de sentido.</p><p>2</p><p>Composição razoavelmente estruturada, com alguns erros de sintaxe, de pontuação e/ou de</p><p>ortografia cuja gravidade não implique a perda de inteligibilidade e/ou de sentido.</p><p>1</p><p>Composição sem estruturação aparente, com a presença de erros graves de sintaxe, de pon-</p><p>tuação e/ou de ortografia cuja gravidade implique a perda frequente de inteligibilidade e/ou</p><p>de sentido.</p><p>Prova 735 • Página C/2/ 9</p><p>No quadro seguinte, apresentam-se critérios de classificação a aplicar às respostas aos itens em situações</p><p>não consideradas anteriormente.</p><p>Situação Classificação</p><p>1. Engano na identificação do item a que o aluno está a</p><p>responder.</p><p>2. Omissão da identificação do item a que o aluno está a</p><p>responder.</p><p>Deve ser vista e classificada a resposta se, pela resolução</p><p>apresentada, for possível identificar inequivocamente o</p><p>item.</p><p>3. É apresentada mais do que uma resposta ao mesmo item</p><p>e o aluno não indica, de forma inequívoca, aquela que</p><p>pretende que seja classificada.</p><p>Deve ser vista e classificada apenas a resposta que surge</p><p>em primeiro lugar, na folha de respostas.</p><p>4. É apresentado apenas o resultado final, embora a</p><p>resolução do item exija cálculos e/ou justificações.</p><p>A resposta deve ser classificada com zero pontos.</p><p>5. Ilegibilidade da resposta. A resposta deve ser classificada com zero pontos.</p><p>6. Item com etapas. A cotação indicada para cada etapa é a classificação</p><p>máxima que lhe é atribuível.</p><p>A classificação da resposta ao item resulta da soma das</p><p>classificações das diferentes etapas, à qual, eventualmente,</p><p>se subtrai um ou dois pontos, de acordo com o previsto nas</p><p>situações 15 e 19.</p><p>7. Etapa com passos. A cotação indicada para cada passo é a classificação</p><p>máxima que lhe é atribuível.</p><p>A classificação da etapa resulta da soma das classificações</p><p>dos diferentes passos.</p><p>8. Item ou etapa cuja cotação se encontra discriminada por</p><p>níveis de desempenho.</p><p>O classificador deve enquadrar a resposta do aluno numa</p><p>das descrições apresentadas, não podendo atribuir uma</p><p>classificação diferente das cotações indicadas.</p><p>9. Utilização de processos de resolução do item não</p><p>previstos nos critérios específicos.</p><p>O critério específico deve ser adaptado ao processo de</p><p>resolução apresentado, mediante distribuição da cotação</p><p>do item pelas etapas percorridas pelo aluno. Esta adaptação</p><p>do critério deve ser utilizada em todos os processos de</p><p>resolução análogos.</p><p>Deve ser aceite qualquer processo de resolução</p><p>cientificamente correcto, ainda que não esteja previsto nos</p><p>critérios específicos de classificação ou no programa.</p><p>10. Não são apresentadas, explicitamente, todas as etapas,</p><p>mas a resolução apresentada permite perceber, inequi-</p><p>vocamente, que elas foram percorridas.</p><p>A(s) etapa(s) implícita(s) é(são) classificada(s) com a</p><p>cotação total para ela(s) prevista.</p><p>11. Transposição incorrecta do enunciado. Se o grau de dificuldade de resolução da etapa não</p><p>diminuir, subtrair um ponto na cotação da etapa.</p><p>Se o grau de dificuldade de resolução da etapa diminuir, a</p><p>classificação máxima a atribuir a essa etapa não deve ser</p><p>superior a 50% da cotação prevista.</p><p>12. Erro ocasional num cálculo. Subtrair um ponto à classificação da etapa em que ocorre o</p><p>erro.</p><p>13. Erro que revela desconhecimento de conceitos, de</p><p>regras ou de propriedades.</p><p>A classificação máxima a atribuir a essa etapa não deve ser</p><p>superior a 50% da cotação prevista para a mesma.</p><p>Prova 735 • Página C/3/ 9</p><p>Situação Classificação</p><p>14. Erro na resolução de uma etapa. A resolução desta etapa é classificada de acordo com o</p><p>erro cometido.</p><p>Se o erro não diminuir o grau de dificuldade de resolução</p><p>das etapas subsequentes, estas são classificadas de acordo</p><p>com os critérios de classificação.</p><p>Se o erro diminuir o grau de dificuldade de resolução das</p><p>etapas subsequentes, a classificação máxima a atribuir a</p><p>essas etapas não deve ser superior a 50% da cotação</p><p>prevista.</p><p>15. Em cálculos intermédios, é pedida uma aproximação a</p><p>um número de casas decimais. O aluno não respeita o</p><p>pedido, e/ou os arredondamentos estão incorrectos.</p><p>Deve subtrair-se um ponto à classificação total do item.</p><p>16. A apresentação do resultado final não respeita a forma</p><p>solicitada. [Exemplo: é pedido o resultado em centíme-</p><p>tros e o aluno apresenta-o em metros].</p><p>Deve subtrair-se um ponto à classificação da etapa</p>
<p>composto por 8 margaridas, 8 rosas e 8 violetas;</p><p>• dará um lucro de 2 euros.</p><p>1.1. A Isabel sugeriu que se fizessem 7 arranjos de cada tipo.</p><p>O Dinis sugeriu que se fizessem 10 arranjos do tipo A e 5 do tipo B.</p><p>Averigúe se cada uma destas propostas é, ou não, viável, tendo em conta as flores</p><p>disponíveis.</p><p>1.2. Determine o número de arranjos de cada tipo que os alunos devem produzir, para</p><p>obterem o maior lucro possível (admitindo que vendem todos os arranjos).</p><p>735/6</p><p>2. Numa festa de aldeia, foi montado um palco para a realização de um espectáculo.</p><p>Em frente deste, colocou-se uma plateia, com um total de 465 cadeiras, dispostas em filas.</p><p>Em cada fila, as cadeiras foram encostadas umas às outras, sem intervalos entre elas.</p><p>A primeira fila tem 10 cadeiras e a última fila tem 52 cadeiras.</p><p>A segunda fila tem mais cadeiras do que a primeira. A terceira fila tem também mais 5 5</p><p>cadeiras do que a segunda, e assim sucessivamente. Cada fila tem, portanto, mais 5</p><p>cadeiras do que a anterior.</p><p>2.1. Mostre que a plateia tem 15 filas.</p><p>2.2. Determine o valor de .5</p><p>2.3. A organização decidiu distribuirdo espectáculo , ao acaso, os 465 bilhetes para os</p><p>lugares sentados. A Nazaré recebeu um bilhete. Ela sabe que, em cada fila, os dois</p><p>lugares situados nas extremidades (um em cada ponta) têm má visibilidade para o</p><p>palco, pelo que gostaria que não lhe calhasse um lugar desses.</p><p>Qual é a probabilidade de a Nazaré ver satisfeita a sua pretensão? Apresente o</p><p>resultado na forma de fracção irredutível.</p><p>3. A Margarida, aluna do curso de Artes Visuais, pretende fazer uma composição artística</p><p>num pedaço de tecido. Para isso, começou por entornar um frasco de tinta azul no tecido.</p><p>Admita que a mancha produzida pela tinta sobre o tecido é um círculo cujo raio vai</p><p>aumentando com o decorrer do tempo.</p><p>Sabe-se que, segundos após o frasco ter sido completamente entornado, a > área (em</p><p>-7#) de tecido ocupada pela mancha é dada, para um certo valor de , por5</p><p>EÐ>Ñ œ > � !"!!</p><p>""% /5> , sendo</p><p>3.1. Supondo que, ao fim de cinco segundos, o raio da mancha circular é de ,% -7</p><p>determine o valor de . Apresente o resultado arredondado às centésimas.5</p><p>3.2. Admita agora que .5 œ + ! #&,</p><p>Calcule a taxa de variação média da função no intervalo , apresentando oE Ò!ß %Ó</p><p>resultado arredondado às unidades.</p><p>Interprete o valor obtido, no contexto do problema.</p><p>V.S.F.F.735/7</p><p>4. Para analisar o som produzido pela vibração de um diapasão, recolheram-se alguns dados</p><p>com um sensor ligado a uma calculadora gráfica.</p><p>O sensor mede a variação de uma certa grandeza (que designaremos por ), ao longoC</p><p>do tempo (que designaremos por ).B</p><p>A partir dos dados, recolhidos em intervalos de tempo iguais, obteve-se, na calculadora, o</p><p>diagrama de dispersão que se pode observar nas figuras 1 e 2 (o eixo das abcissas</p><p>corresponde à variável e o das ordenadas à variável ). B C</p><p>Figura 1 Figura 2</p><p>Em cada uma das figuras, está representada a posição do cursor no visor da calculadora.</p><p>Na figura 1, o cursor encontra-se num ponto cuja ordenada é o máximo de .C</p><p>Na figura 2, o cursor encontra-se num ponto cuja ordenada é o mínimo de .C</p><p>Admita que o fenómeno é bem modelado por uma função definida por uma expressão do</p><p>tipo onde , e são constantes reais positivas.C œ + 4 , Ð- BÑ + , -cos ,</p><p>4.1. Relativamente a qualquer função definida por uma expressão do tipo indicado,</p><p>justifique que:</p><p>4.1.1. O contradomínio é o intervalo Ò + + ,ß + 4 ,Ó</p><p>4.1.2. #</p><p>-</p><p>1 é período da função.</p><p>4.2. Determine os valores dos parâmetros , e , tendo em conta:+ , -</p><p>• os dados contidos nas figuras 1 e 2</p><p>• a alínea 4.1.1.</p><p>• a alínea 4.1.2. e o facto de não existir nenhum período positivo inferior a #-</p><p>1</p><p>Apresente o valor de arredondado às unidades.-</p><p>735/8</p><p>5. A empresa de telecomunicações TLV efectuou um estudo estatístico relativo a todos os</p><p>modelos de telemóveis já vendidos pela empresa.</p><p>Este estudo revelou que o número , em , de vendidas, depende do8 milhares unidades</p><p>preço (em euros) de cada telemóvel, de acordo com o seguinte diagrama de dispersão.:</p><p>5.1. Admita que a empresa possui um ficheiro com os nomes de todos os clientes e,</p><p>para cada um deles, o preço do telemóvel adquirido (cada cliente adquiriu apenas</p><p>um telemóvel). Para assinalar o seu aniversário, a resolveu sortear uma TLV</p><p>viagem entre os seus clientes.</p><p>Qual é a probabilidade de a viagem sair a um cliente que tenha comprado um</p><p>telemóvel por um preço inferior a 180 euros? Apresente o resultado na forma de</p><p>fracção irredutível.</p><p>5.2. entreRecorrendo à sua calculadora, determine o coeficiente de correlação linear</p><p>as variáveis e . Apresente o valor pedido arredondado às centésimas.: 8</p><p>Explique como procedeu, reproduzindo na sua folha de prova as listas que</p><p>introduziu na calculadora.</p><p>Tendo em conta o diagrama de dispersão apresentado na figura acima, interprete o</p><p>valor obtido.</p><p>5.3. A vai lançar um novo modelo de telemóvel. Com base no estudo efectuado, TLV</p><p>bem como noutros indicadores, esta empresa prevê, relativamente ao modelo que</p><p>vai ser lançado, que a relação entre (número, em , de telemóveis que8 milhares</p><p>serão vendidos) e (preço de cada telemóvel do novo modelo) estará de acordo:</p><p>com a expressão</p><p>8 œ + ! !$ : 4 "!,</p><p>Seja a quantia (em euros) que a empresa prevê vir a receber pela venda dos;</p><p>telemóveis do novo modelo.</p><p>Escreva uma expressão que dê a quantia , em função do preço de cada; :</p><p>telemóvel. Apresente essa expressão na forma de um polinómio reduzido.</p><p>V.S.F.F.735/9</p><p>6. Pretende-se construir um filtro de forma cónica, com uma capacidade superior a meio litro.</p><p>Para o efeito, dispõe-se de uma folha de papel de filtro, de forma rectangular, de 32 cm de</p><p>comprimento e 18 cm de largura.</p><p>Na figura, está representado um esquema de uma possível planificação do filtro. Como se</p><p>pode observar, essa planificação é um sector circular, de raio igual à largura da folha de</p><p>papel.</p><p>Averigúe se o filtro construído de acordo com esta planificação tem, ou não, uma</p><p>capacidade superior a meio litro.</p><p>Nota: sempre que, nos cálculos intermédios, proceder a arredondamentos, conserve, no</p><p>mínimo, quatro casas decimais.</p><p>Percorra sucessivamente as seguin-</p><p>tes etapas:</p><p>• Determine a amplitude, em</p><p>radianos, do ângulo , represen-α</p><p>tado na figura junta.</p><p>• Determine o perímetro da base do</p><p>cone.</p><p>• Determine o raio da base do</p><p>cone.</p><p>• Determine a altura do cone.</p><p>• Determine o volume do cone e</p><p>responda à questão colocada.</p><p>(recorde que )" 63><9 œ "!!! -7$</p><p>FIM</p><p>735/10</p><p>COTAÇÕES</p><p>1. ............................................................................................. 30</p><p>1.1. .......................................................................... 10</p><p>1.2. .......................................................................... 20</p><p>2. ............................................................................................. 30</p><p>2.1. .......................................................................... 10</p><p>2.2. .......................................................................... 10</p><p>2.3. .......................................................................... 10</p><p>3. ............................................................................................. 30</p><p>3.1. ......................................................................... 15</p><p>3.2. .......................................................................... 15</p><p>4. ............................................................................................. 45</p><p>4.1. ......................................................................... 30</p><p>4.1.1. .................................................. 15</p><p>4.1.2. .................................................. 15</p><p>4.2. .......................................................................... 15</p><p>5. .............................................................................................</p>
<p>correspondente ao resultado final.</p><p>17. Na apresentação do resultado final, não está expressa</p><p>a unidade de medida. [Exemplo: «15» em vez de «15</p><p>metros»].</p><p>A etapa relativa ao resultado final é classificada tal como se</p><p>a unidade de medida estivesse indicada.</p><p>18. O resultado final apresenta um número de casas</p><p>decimais diferente do solicitado e/ou está incorrecta-</p><p>mente arredondado.</p><p>Deve subtrair-se um ponto à classificação da etapa</p><p>correspondente ao resultado final.</p><p>19. Utilização de simbologias ou de expressões inequi-</p><p>vocamente incorrectas do ponto de vista formal.</p><p>Deve subtrair-se um ponto à classificação total do item,</p><p>excepto:</p><p>– se as incorrecções ocorrerem apenas em etapas já</p><p>classificadas com zero pontos;</p><p>– no caso de uso do símbolo de igualdade onde, em rigor,</p><p>deveria ter sido usado o símbolo de igualdade</p><p>aproximada.</p><p>Prova 735 • Página C/4/ 9</p><p>CRITÉRIOS ESPECÍFICOS DE CLASSIFICAÇÃO</p><p>1.1. ................................................................................................................................................. 12 pontos</p><p>Indicação correcta das 6 coordenadas (ver notas) ....................... (6 × 2) ....................... 12</p><p>Notas:</p><p>1. Se o examinando, tendo calculado bem os valores das coordenadas de um ponto, ao indicar</p><p>as coordenadas como pares ordenados, trocar a abcissa com a ordenada, a classificação a</p><p>atribuir à indicação das coordenadas desse ponto deverá ser de 1 ponto.</p><p>2. Se o examinando, tendo calculado bem os valores das coordenadas de um ponto, não indicar</p><p>as coordenadas sob a forma de par ordenado, a classificação a atribuir à indicação das</p><p>coordenadas desse ponto deverá ser de 1 ponto.</p><p>3. Se o examinando se enganar no cálculo de alguma das coordenadas de um ponto, e utilizar</p><p>esse resultado no cálculo das coordenadas dos pontos seguintes, estas devem ser</p><p>classificadas de acordo com o erro cometido.</p><p>1.2. .................................................................................................................................................. 18 pontos</p><p>Escrever a equação y = mx + b ................................................................................... 4</p><p>Determinar o valor de m .... (ver nota) .................................................................... 7</p><p>Indicar ou calcular o valor de b .... (0) ............................................................................ 5</p><p>Escrever a equação reduzida da recta AF .... ( ) .............................................. 2</p><p>Nota: Caso o examinando se tenha enganado no cálculo das coordenadas de F, a classificação</p><p>a atribuir a esta etapa deve estar de acordo com o erro cometido.</p><p>2.1. .................................................................................................................................................. 20 pontos</p><p>A classificação a atribuir à resposta deve estar de acordo com os seguintes níveis de</p><p>desempenho:</p><p>Interpretação correcta (por exemplo: «o número de trutas não é igual ao número</p><p>total de peixes.») ........................................................................................................ 20</p><p>Interpretação correcta do ponto de vista matemático, mas desligada do contexto da</p><p>situação descrita (por exemplo: «o denominador da fracção nunca é igual ao</p><p>numerador.») .............................................................................................................. 10</p><p>Outras situações ........................................................................................................ 0</p><p>Nota: Qualquer tentativa de mostrar a impossibilidade da equação não deve ser valorizada.</p><p>y x3=</p><p>2</p><p>3    2</p><p>Prova 735 • Página C/5/ 9</p><p>2.2. .................................................................................................................................................. 20 pontos</p><p>Equacionar o problema ( ou equivalente) ............................................. 8</p><p>Resolver a equação (ver nota) ........................................................................................ 12</p><p>Nota: O examinando pode resolver a equação gráfica ou analiticamente.</p><p>Se o examinando resolver a equação graficamente, a cotação desta etapa deve ser</p><p>repartida da seguinte forma:</p><p>Apresentação do(s) gráfico(s) ...................................................................................... 8</p><p>• Apresentação do referencial, da curva representativa da função P e da recta</p><p>de equação y = 0,25 .................................................................................. 2</p><p>• Ausência de pontos de abcissa negativa ...................................................... 2</p><p>• Representação do ponto de intersecção ...................................................... 2</p><p>• Representação da abcissa do ponto de intersecção .................................... 2</p><p>Solução ......................................................................................................................... 4</p><p>Observação: Caso o examinando não apresente, na etapa anterior, a abcissa do ponto</p><p>de intersecção, mas conclua que a solução do problema é 100, devem</p><p>ser atribuídos os dois pontos relativos à representação da referida</p><p>abcissa.</p><p>ou</p><p>Se o examinando resolver a equação analiticamente, a cotação desta etapa deve ser</p><p>repartida da seguinte forma:</p><p>Desembaraçar de denominadores ............................................................................... 6</p><p>Isolar os termos em x num dos membros ................................................................... 4</p><p>Obter o valor de x ........................................................................................................ 2</p><p>3.1. .................................................................................................................................................. 20 pontos</p><p>Indicar o número de casos possíveis .... (500) ................................................................ 9</p><p>Indicar o número de casos favoráveis .... (300) .............................................................. 9</p><p>Indicar a probabilidade .... .................................................................................... 2</p><p>Nota: Na escrita da fracção estão incluídas implicitamente as indicações do número de</p><p>casos possíveis e do número de casos favoráveis, pelo que devem ser atribuídos 20</p><p>pontos. Se o examinando se limitar a escrever a fracção , a resposta deve ser</p><p>classificada com 16 pontos.</p><p>3.2. .................................................................................................................................................. 20 pontos</p><p>Indicar o coeficiente de correlação linear .... (0,94) ........................................................ 10</p><p>Interpretar o valor obtido ................................................................................................... 10</p><p>Interpretação relativa ao sinal ......................................................................... 5</p><p>Interpretação relativa ao valor absoluto .......................................................... 5</p><p>3</p><p>5</p><p>300</p><p>500</p><p>300    500</p><p>x</p><p>x</p><p>25=</p><p>300 + 100</p><p>Prova 735 • Página C/6/ 9</p><p>4.1. .................................................................................................................................................. 20 pontos</p><p>Completar a tabela ............................................... (5 × 2) ................................................ 10</p><p>Mostrar que h = 7 + 5 senθ ......................................................................................... 10</p><p>............................................................................................. 2</p><p>................................................................................................ 3</p><p>.............................................................................................. 3</p><p>........................................................................................................ 2</p><p>4.2. ..................................................................................................................................................</p>
<p>20 pontos</p><p>Este item pode ser resolvido por, pelo menos, três processos:</p><p>1.º Processo:</p><p>Determinar o perímetro da primeira circunferência (18,849) .......................................... 3</p><p>Determinar o perímetro da décima circunferência (24,504) ........................................... 7</p><p>Calcular o resultado pedido com a fórmula que dá a soma de n primeiros</p><p>termos da progressão aritmética (216,77) .................................................................. 10</p><p>Substituir correctamente os valores na fórmula .............................................. 7</p><p>Restantes cálculos .......................................................................................... 3</p><p>2.º Processo:</p><p>Calcular os dez perímetros ......................................... (10 × 1)........................................ 10</p><p>Calcular a soma dos perímetros (216,77) ..................................................................... 10</p><p>3.º Processo:</p><p>Calcular a soma dos raios ................................................................................................. 8</p><p>Calcular o décimo termo da progressão aritmética dos raios (3,900) .......... 4</p><p>Calcular a soma dos dez primeiros termos da progressão aritmética</p><p>dos raios (34,500) ......................................................................................... 4</p><p>Substituir correctamente os valores na fórmula .................................. 3</p><p>Restantes cálculos ............................................................................... 1</p><p>Multiplicar o valor obtido por 2π .... (216,77) ................................................................ 12</p><p>5.1. .................................................................................................................................................. 10 pontos</p><p>Este item pode ser resolvido por, pelo menos, dois processos:</p><p>1.º Processo:</p><p>Determinar o número de quilogramas de vitaminas necessário para 4 toneladas de</p><p>FarX e para 3 toneladas de FarY (2 × 4 + 3 = 11) ................................................... 2</p><p>Determinar o número de quilogramas de sabores necessário para 4 toneladas de FarX</p><p>e para 3 toneladas de FarY (4 + 2 × 3 = 10) ............................................................ 2</p><p>Determinar o número de quilogramas de conservantes necessário para 4 toneladas de</p><p>FarX e para 3 toneladas de FarY (4 + 3 × 3 = 13) ................................................... 2</p><p>Concluir que é possível .................................................................................................... 4</p><p>DC = 7</p><p>senPD θ= 5</p><p>senPD</p><p>OP</p><p>θ=</p><p>h PD DC= +</p><p>Prova 735 • Página C/7/ 9</p><p>2.º Processo:</p><p>Indicar as restrições da região admissível ........................................................................ 8</p><p>Restrições:</p><p>2x + y ≤	16 ............................................................................................. 2</p><p>x + 2y ≤	11 ............................................................................................. 2</p><p>x + 3y ≤	15 ............................................................................................. 2</p><p>x ≥ 0 ......................................................................................................... 1</p><p>y ≥ 0 ......................................................................................................... 1</p><p>Verificar, gráfica ou analiticamente, que (4,3) pertence à região admissível .................. 2</p><p>5.2. .................................................................................................................................................. 20 pontos</p><p>Indicar as restrições da região admissível ........................................................................ 8</p><p>Restrições:</p><p>2x + y ≤	16 ............................................................................................. 2</p><p>x + 2y ≤	11 ............................................................................................. 2</p><p>x + 3y ≤	15 ............................................................................................. 2</p><p>x ≥ 0 ......................................................................................................... 1</p><p>y ≥ 0 ......................................................................................................... 1</p><p>Indicar a função objectivo .... (C(x, y)= x + y) .......................................................... 2</p><p>Representar graficamente a região admissível ................................................................ 6</p><p>Representar correctamente 2x + y =	16 ..................................................... 1</p><p>Representar correctamente x + 2y =	11 ..................................................... 1</p><p>Representar correctamente x + 3y =	15 ..................................................... 1</p><p>Assinalar o polígono ........................................................................................ 3</p><p>Indicar, justificando, os valores de x e de y para os quais é máxima a função objectivo</p><p>(x = 7, y =	2)	 ............................................................................................................... 4</p><p>6. ..................................................................................................................................................... 20 pontos</p><p>Apresenta-se, a seguir, um exemplo de resposta:</p><p>«De acordo com a informação dada pelo laboratório, a percentagem da massa de C14</p><p>encontrado nos manuscritos é de 96% da massa de C14 original, pelo que se tem:</p><p>Concluímos, assim, que a idade do papel é de cerca de 337 anos. Como ele foi analisado</p><p>laboratorialmente em 2008, resulta que terá sido fabricado por volta de 1671, atendendo</p><p>a que 2008 – 337 =1671. Ora, como Leonardo da Vinci morreu em 1519, o manuscrito</p><p>não é da sua autoria.»</p><p>Tal como o exemplo ilustra, a composição deverá contemplar os seguintes tópicos:</p><p>• idade do papel;</p><p>• data em que terá sido fabricado;</p><p>• conclusão pela não autoria de Leonardo da Vinci.</p><p>( ), ,, , , ln ,t tce c e t t− 0 000121 − 0 000121= 0 96 ⇔ = 0 96 ⇔ −0 000121 = 0 96 ⇔ ≈ 337</p><p>Prova 735 • Página C/8/ 9</p><p>Na tabela seguinte, indica-se como deve ser classificado este item, de acordo com os</p><p>níveis de desempenho no domínio da comunicação escrita em língua portuguesa</p><p>descritos nos critérios gerais e os níveis de desempenho no domínio específico da</p><p>disciplina:</p><p>Notas:</p><p>1. Apenas podem ser atribuídas classificações correspondentes a um dos valores constantes do</p><p>quadro. Não há lugar a classificações intermédias. No caso de a resposta não atingir o nível 1</p><p>de desempenho no domínio específico da disciplina, a classificação a atribuir é zero pontos.</p><p>Neste caso, não é classificado o desempenho no domínio da comunicação escrita em língua</p><p>portuguesa.</p><p>2. O examinando pode resolver a equação c–0,000121 t = 0,96 recorrendo à calculadora.</p><p>3. Se o examinando se enganar no cálculo da idade do papel, deve considerar-se que a</p><p>composição não contempla o primeiro tópico. Os tópicos seguintes devem ser analisados de</p><p>acordo com o erro cometido. Em particular, o erro poderá levar o examinando a concluir pela</p><p>possibilidade de a autoria do manuscrito ser de Leonardo da Vinci. No entanto, em caso</p><p>algum pode concluir pela autoria de Leonardo da Vinci. Se o examinando expressar tal</p><p>conclusão, deve considerar-se que a composição não contempla o terceiro tópico.</p><p>Descritores do nível de desempenho no domínio</p><p>da comunicação escrita em língua portuguesa</p><p>Descritores do nível de desempenho</p><p>no domínio específico da disciplina</p><p>Níveis</p><p>1 2 3</p><p>N</p><p>ív</p><p>e</p><p>is</p><p>3 A resposta contempla, correctamente, os três tópicos solicitados. 18 19 20</p><p>2</p><p>A resposta contempla, correctamente, dois dos tópicos solicita-</p><p>dos.</p><p>12 13 14</p><p>1</p><p>A resposta contempla, correctamente, apenas um dos tópicos</p><p>solicitados.</p><p>6 7 8</p><p>Prova 735 • Página C/9/ 9</p><p>APM – Associação de Professores de Matemática</p><p>Sugestão de resolução da prova de exame de Matemática B (código 735)</p><p>2ª fase – 16/07/2008</p><p>1.</p><p>1.1.</p><p>Coordenadas dos pontos:</p><p>A → (0,0)</p>
<p>B → (750, 0)</p><p>C→ (750,450)</p><p>D → (900,450)</p><p>E→ (900,750)</p><p>F→ (500,750)</p><p>1.2.</p><p>Uma equação reduzida da recta é da forma:</p><p>ℜ∈+= bmbmxy ,,</p><p>Como a recta passa na origem do referencial a ordenada na origem é 0, donde b=0.</p><p>Ou seja, a recta pertence à família de rectas da forma ℜ∈= mmxy , .</p><p>Sendo F→ (500,750) um ponto pertencente à recta, temos:</p><p>2</p><p>3</p><p>500750 =⇔×= mm</p><p>A equação da recta é xy</p><p>2</p><p>3=</p><p>2.</p><p>2.1.</p><p>A equação ( ) 1=xP significa que o número de trutas é igual ao número total de peixes</p><p>existentes no tanque, o que é impossível porque existem também 300 robalos.</p><p>2.2.</p><p>Se a proporção de trutas é 25% do total teremos que:</p><p>100</p><p>25</p><p>300100</p><p>25</p><p>)( =</p><p>+</p><p>⇔=</p><p>x</p><p>x</p><p>xP</p><p>Donde resulta: ( ) 10075002510025300100 =⇔=−⇔×+=× xxxxx</p><p>O número total de trutas a introduzir no tanque seria 100.</p><p>3.</p><p>3.1.</p><p>Número total de peixes no tanque: 300+200 = 500</p><p>Número de robalos: 300</p><p>Probabilidade de pescar um robalo =</p><p>500</p><p>300</p><p>ou seja a probabilidade é de</p><p>5</p><p>3</p><p>APM – Associação de Professores de Matemática</p><p>3.2.</p><p>Introduzindo nas listas da calculadora, em L1 (comprimento a) e em L2 (peso p) e, com</p><p>o menu estatístico, pedindo a regressão linear de L1 e L2 obtém-se:</p><p>y=ax+b</p><p>a=1,71373556…</p><p>b=-129,055199…</p><p>com um coeficiente de correlação linear de 0,94270…</p><p>A nuvem de pontos correspondente às duas variáveis, obtida com a calculadora é a</p><p>seguinte:</p><p>Assim, o valor pedido é 0,94, que representa uma correlação positiva muito forte</p><p>significando que a recta de regressão se ajusta bem à nuvem de pontos, a qual traduz o</p><p>facto de o peso e o comprimento aumentarem na mesma proporção.</p><p>4.</p><p>4.1.</p><p>θ 0º 90º 180º 270º 360º</p><p>h 7 12 7 2 7</p><p>( ) ( ) ( )θθθ senPD</p><p>PD</p><p>sen</p><p>OP</p><p>PD</p><p>sen 5</p><p>5</p><p>=⇔=⇔=</p><p>Como º90º0 << θ a distância à base do painel é dada por PDOB + .</p><p>Assim temos que: ( ) ( )θθ senh 57 += .</p><p>4.2.</p><p>Se os raios das circunferências estão em progressão aritmética de razão 0,1 m os</p><p>perímetros das mesmas estão em progressão aritmética de razão ππ 2,021,0 =× .</p><p>Assim, a quantidade de fio para as 10 circunferências é dada pela soma dos 10 primeiros</p><p>termos da sucessão dos perímetros: ( ) ( )12,06 −+= nnP ππ .</p><p>metrosSS</p><p>PP</p><p>S 77,21610</p><p>2</p><p>8,166</p><p>10</p><p>2 1010</p><p>101</p><p>10 ≈⇔×++=⇔×</p><p>+</p><p>= πππ</p><p>O número de metros de fio necessário para executar o trabalho é de 216,77 metros.</p><p>APM – Associação de Professores de Matemática</p><p>5.</p><p>5.1.</p><p>Para fabricar 4 toneladas de FarX e 3 toneladas de FarY são necessários :</p><p>2×4 + 1×3 = 11 quilogramas de vitaminas</p><p>1×4 + 2×3 = 10 quilogramas de sabores</p><p>1×4 + 3×3 = 13 quilogramas de conservantes</p><p>Como temos: 16 quilogramas de vitaminas, 11 quilogramas de sabores e 15 quilogramas</p><p>de conservantes é possível fabricar, num só dia, as 4 toneladas de FarX e as 3 toneladas</p><p>de FarY.</p><p>5.2.</p><p>Sendo x o número de toneladas de FarX e y o número de toneladas de FarY, tendo em</p><p>consideração que as produções devem ser não negativas e as limitações impostas pelos</p><p>componentes, temos as seguintes restrições:</p><p>153</p><p>112</p><p>162</p><p>0</p><p>0</p><p>≤+</p><p>≤+</p><p>≤+</p><p>≥</p><p>≥</p><p>yx</p><p>yx</p><p>yx</p><p>y</p><p>x</p><p>Pretende-se maximizar a produção donde a função objectivo é: F(x,y)=x + y</p><p>Considerando a região admissível,</p><p>representada na figura ao lado, obtida com a</p><p>calculadora gráfica, concluímos que podemos</p><p>produzir diariamente 7 toneladas de FarX e 2</p><p>toneladas de FarY.</p><p>6.</p><p>Como a massa de carbono é 0,96c temos que:</p><p>tecc 000121,096,0 −×= que nos permite calcular a idade do papel.</p><p>( ) anostteecc tt 37,33796,0ln000121,096,096,0 000121,0000121,0 =⇔=−⇔=⇔×= −−</p><p>A idade do papel é de aproximadamente 337 anos.</p><p>Assim, a data em que terá sido fabricado corresponde a 2008−337,37 =1670,63, o que</p><p>significa que foi fabricado no ano de 1670.</p><p>Atendendo a que Leonardo da Vinci viveu entre 1456 e 1519 pode afirmar-se que o</p><p>autor do manuscrito não poderá ter sido ele.</p><p>FIM</p><p>2009</p><p>Prova 735 • Página 1/ 11</p><p>EXAME NACIONAL DO ENSINO SECUNDÁRIO</p><p>Decreto-Lei n.º 74/2004, de 26 de Março</p><p>Prova Escrita de Matemática B</p><p>11.º/12.º Anos de Escolaridade</p><p>Prova 735/1.ª Fase 11 Páginas</p><p>Duração da Prova: 150 minutos. Tolerância: 30 minutos.</p><p>2009</p><p>Utilize apenas caneta ou esferográfica de tinta indelével, azul ou preta, excepto nas respostas</p><p>que impliquem a elaboração de construções, de desenhos ou de outras representações, que</p><p>podem ser, primeiramente, elaborados a lápis, sendo, a seguir, passados a tinta.</p><p>Utilize a régua, o compasso, o esquadro, o transferidor e a calculadora gráfica sempre que for</p><p>necessário.</p><p>Não é permitido o uso de corrector. Em caso de engano, deve riscar, de forma inequívoca,</p><p>aquilo que pretende que não seja classificado.</p><p>Escreva, de forma legível, a numeração dos grupos e dos itens, bem como as respectivas</p><p>respostas. As respostas ilegíveis ou que não possam ser identificadas são classificadas com</p><p>zero pontos.</p><p>Para cada item, apresente apenas uma resposta. Se escrever mais do que uma resposta a um</p><p>mesmo item, apenas é classificada a resposta apresentada em primeiro lugar.</p><p>Em todas as respostas, indique todos os cálculos que tiver de efectuar e todas as justificações necessárias.</p><p>Sempre que, na resolução de um problema, recorrer à calculadora, apresente todos os elementos recolhidos</p><p>na sua utilização. Mais precisamente:</p><p>• sempre que recorrer às capacidades gráficas da calculadora, apresente o(s) gráfico(s) obtido(s), bem</p><p>como as coordenadas de pontos relevantes para a resolução do problema proposto (por exemplo,</p><p>coordenadas de pontos de intersecção de gráficos, máximos, mínimos, etc.);</p><p>• sempre que recorrer a uma tabela obtida na calculadora, apresente todas as linhas da tabela relevantes</p><p>para a resolução do problema proposto;</p><p>• sempre que recorrer a estatísticas obtidas na calculadora (média, desvio padrão, coeficiente de</p><p>correlação, declive e ordenada na origem de uma recta de regressão, etc.), apresente a(s) lista(s) que</p><p>introduziu na calculadora para a(s) obter.</p><p>A prova inclui, na página 11, um Formulário.</p><p>As cotações dos itens encontram-se no final do enunciado da prova.</p><p>Prova 735 • Página 2/ 11</p><p>GRUPO I</p><p>O Stomachion, também conhecido como Caixa de Arquimedes, é um puzzle geométrico cuja invenção é</p><p>atribuída a Arquimedes de Siracusa (287-212 a.C.). É constituído por 14 peças poligonais que formam um</p><p>quadrado como o representado na figura 1.</p><p>Fig. 1</p><p>A figura 2 representa, sobreposto a uma malha</p><p>quadriculada, um Stomachion com 12 unidades de lado.</p><p>Os pontos A, B, C, D, E, F, G e H são vértices da</p><p>malha.</p><p>Fixando um referencial ortogonal e monométrico, de</p><p>origem D, como se sugere na figura 2, o ponto A tem</p><p>coordenadas (0, 6).</p><p>1. Determine as coordenadas do ponto simétrico de C,</p><p>relativamente ao eixo das abcissas.</p><p>2. Uma das propriedades do Stomachion é a seguinte: o</p><p>quociente entre a área de cada peça e a área total do</p><p>Stomachion é sempre um número racional.</p><p>Mostre que essa propriedade se verifica com a peça</p><p>representada, na figura 2, pelo quadrilátero sombreado</p><p>[ABCD ].</p><p>Sugestão: Na sua resposta pode percorrer, sucessivamente, as seguintes etapas:</p><p>• determine a área do quadrado [EFGH ];</p><p>• determine a área da peça sombreada;</p><p>• determine o quociente entre a área da peça sombreada e a área do quadrado;</p><p>• justifique que o quociente obtido é um número racional.</p><p>Prova 735 • Página 3/ 11</p><p>�</p><p>�</p><p>�</p><p>�</p><p>�</p><p>�</p><p>�</p><p>�</p><p>���������</p><p>Fig. 2</p><p>GRUPO II</p><p>Numa determinada região, existe um parque natural no qual vivem diferentes espécies de animais, cada uma</p><p>no seu habitat.</p><p>Uma empresa pretende instalar uma unidade fabril nessa região, a sul do parque natural, e, para tal, aguarda</p><p>decisão das entidades responsáveis. Para apoio dessa decisão, foi elaborado um estudo de impacto</p><p>ambiental.</p><p>1. De acordo com esse estudo, prevê-se que o nível de concentração diário de um poluente, em partes por</p><p>milhão (p.p.m.), originado pelo escoamento de águas residuais, siga uma distribuição normal, N (8, 2),</p><p>de média µ = 8 e desvio padrão σ = 2.</p><p>O estudo refere que o nível de concentração</p>
<p>desse poluente não deverá exceder o equilíbrio ecológico</p><p>aceitável de 10 p.p.m.</p><p>Determine a probabilidade de, num certo dia, o nível de concentração do poluente exceder esse valor.</p><p>Apresente o resultado na forma de percentagem, arredondado às unidades.</p><p>2. O estudo de impacto ambiental inclui dados de uma prospecção geotérmica realizada no parque natural</p><p>por técnicos do Serviço de Geofísica. Os dados mostram que, a maiores profundidades, correspondem</p><p>temperaturas mais elevadas.</p><p>Com base nesses dados, obteve-se a equação y = 0,0290x + 18,36 , que define a recta de regressão</p><p>de y sobre x , com 0 ≤ x ≤ 350, designando x a profundidade, em metros, e y a temperatura,</p><p>em graus Celsius.</p><p>Estime o valor da temperatura a 100 m de profundidade, de acordo com a equação da recta de</p><p>regressão apresentada.</p><p>Apresente o resultado, em graus Celsius, com duas casas decimais.</p><p>Prova 735 • Página 4/ 11</p><p>3. Uma águia, ao efectuar um voo planado à procura de alimento, avistou uma lebre no fundo do vale do</p><p>parque natural. O fundo do vale é uma área plana. De imediato, a águia iniciou um voo picado, a grande</p><p>velocidade, em direcção à presa, capturando-a em poucos segundos. Após a captura, transportou a lebre</p><p>para o cimo de um penhasco, terminando aí o seu voo.</p><p>O momento da captura corresponde ao instante em que a águia atingiu, no seu voo, a distância mínima</p><p>ao fundo do vale.</p><p>Admita que a distância, h , em metros, a que a águia se encontra do fundo do vale, t segundos após o</p><p>início do voo picado, é dada, aproximadamente, por</p><p>3.1. Determine o valor da taxa de variação média de h no intervalo [0; 3]</p><p>Apresente o resultado com aproximação às décimas.</p><p>Em cálculos intermédios, não proceda a arredondamentos.</p><p>3.2. Na figura 3, que não está à escala, apresenta-se um esboço do gráfico de f , função que dá, em</p><p>metros por segundo, a taxa de variação instantânea de h no instante t .</p><p>Admita que a taxa de variação instantânea de h se anula no instante t = 5,4 .</p><p>Descreva o que aconteceu no instante t = 5,4 , no contexto da situação referida, justificando a</p><p>ocorrência através da relação existente entre a monotonia de h e o sinal da respectiva taxa de</p><p>variação instantânea.</p><p>Fig. 3</p><p>�</p><p>�</p><p>������</p><p>�</p><p>h t t t t t t( ) = − + − − + ∈ </p><p></p><p></p><p>0 125 2 5 12 9 11 94 8 0 9 64 3 2, , , , , ; ,com</p><p>Prova 735 • Página 5/ 11</p><p>4. No parque natural, foram plantadas, num certo momento, duas árvores, uma da espécie P e outra da</p><p>espécie C .</p><p>Admita que as alturas, em metros, da árvore da espécie P e da árvore da espécie C , x anos depois de</p><p>terem sido plantadas, são dadas, aproximadamente, por P(x ) e C(x ) :</p><p>Espécie P :</p><p>Espécie C :</p><p>Com base nas funções apresentadas, alguém afirmou que:</p><p>I) quando as árvores foram plantadas, a árvore da espécie P tinha menos 1,1m de altura do que a</p><p>árvore da espécie C ;</p><p>II) foram necessários mais do que oito anos para que a árvore da espécie P ficasse mais alta que a</p><p>árvore da espécie C ;</p><p>III) com o decorrer do tempo, a diferença entre as alturas das duas árvores tenderá a igualar</p><p>os 4 m.</p><p>Elabore uma pequena composição na qual refira se cada uma das afirmações, I), II) e III), está,</p><p>ou não, correcta, explicitando, para cada caso, uma razão que fundamente a sua resposta.</p><p>C x</p><p>e x( ) =</p><p>+ −</p><p>6</p><p>1 2 9 0 12, ,</p><p>P x</p><p>e x( ) =</p><p>+ −</p><p>10</p><p>1 12 5 0 23, ,</p><p>Prova 735 • Página 6/ 11</p><p>GRUPO III</p><p>A BRUGÁS é uma empresa que processa uma variedade de gás usada na confecção de um produto para</p><p>aquecimento. Este produto é classificado em dois tipos: PPremium e PRegular.</p><p>Em cada semana, a BRUGÁS recebe 24m3 de gás e dispõe de 45 horas para os processar.</p><p>Por motivos técnicos, as variedades de gás não podem ser processadas em simultâneo.</p><p>A produção de cada tonelada de PPremium :</p><p>• requer 3 m3 de gás;</p><p>• demora 5 horas;</p><p>• gera um lucro de 1600 euros.</p><p>A produção de cada tonelada de PRegular :</p><p>• requer 2 m3 de gás;</p><p>• demora 5 horas;</p><p>• gera um lucro de 1200 euros.</p><p>Devido a problemas relacionados com o armazenamento, a empresa só pode produzir até 5 toneladas de</p><p>PRegular.</p><p>Represente por x o número de toneladas de PPremium produzidas, semanalmente, pela empresa</p><p>BRUGÁS.</p><p>Represente por y o número de toneladas de PRegular produzidas, semanalmente, pela empresa</p><p>BRUGÁS.</p><p>Quantas toneladas de PPremium e de PRegular devem ser produzidas, semanalmente, pela empresa</p><p>BRUGÁS, para que o lucro semanal seja máximo?</p><p>Na sua resposta, percorra, sucessivamente, as seguintes etapas:</p><p>• indique a função objectivo;</p><p>• indique as restrições do problema;</p><p>• represente, graficamente, a região admissível, referente ao sistema de restrições;</p><p>• calcule os valores das variáveis para os quais é máxima a função objectivo.</p><p>Prova 735 • Página 7/ 11</p><p>GRUPO IV</p><p>Numa feira de agricultura, o Sr. Pedro, negociante de cavalos, pedia por um cavalo puro-sangue a quantia de</p><p>4 000 000 de euros. O Sr. João estava muito interessado em comprar o cavalo, mas considerava o preço</p><p>muito elevado.</p><p>O Sr. Pedro propôs-lhe, então, o seguinte negócio:</p><p>«O cavalo tem 4 ferraduras, e cada uma delas tem 8 cravos. O Sr. João dá-me um cêntimo pelo</p><p>primeiro cravo da ferradura da pata dianteira esquerda; dois cêntimos pelo segundo cravo da</p><p>mesma ferradura, e assim sucessivamente, duplicando sempre, até ao oitavo cravo dessa ferradura,</p><p>pelo qual me dá 1,28 euros.»</p><p>«Repare: pelos oito cravos da ferradura desta pata, o Sr. João paga-me 2,55 euros. Barata a feira!</p><p>Continuemos para os outros cravos. Pelo primeiro cravo da pata dianteira direita, o Sr. João dá-me</p><p>2,56 euros, isto é, o dobro do valor do oitavo cravo da pata dianteira esquerda, e assim</p><p>sucessivamente, duplicando sempre, até se terem esgotado os 32 cravos das ferraduras do cavalo.»</p><p>«O sr. João aceita pagar-me, por este cavalo, a quantia total do valor dos cravos das ferraduras?»</p><p>1. Verifique que o valor total dos cravos da ferradura da pata dianteira esquerda é de 2,55 euros, tal como</p><p>o Sr. Pedro refere.</p><p>2. Mostre que, de acordo com a proposta do Sr. Pedro, o valor a pagar pelo cavalo é superior a</p><p>4000000 de euros.</p><p>Prova 735 • Página 8/ 11</p><p>GRUPO V</p><p>Na figura 4, ilustra-se um método simples para determinar o raio da</p><p>Terra. Este método consiste em medir o ângulo α , ângulo de</p><p>depressão do horizonte, a partir de um ponto de altitude elevada, do</p><p>qual se avista o mar.</p><p>Relativamente a esta figura, que não está à escala, considere que:</p><p>• B representa o ponto de observação;</p><p>• C designa o centro da Terra;</p><p>• α é a amplitude, em graus, do ângulo de depressão do horizonte,</p><p>(0º < α</p><p><</p><p>90º);</p><p>• h é a altitude do lugar, em quilómetros;</p><p>• o triângulo [ABC ] é rectângulo em A ;</p><p>• R é o raio da Terra, em quilómetros;</p><p>• .</p><p>1. Mostre que .</p><p>Sugestão: Comece por determinar cos α no triângulo [ABC ] e, de seguida, resolva a equação obtida em</p><p>ordem a R .</p><p>2. Eratóstenes (276-195 a.C.), por volta do ano 230 a.C.,</p><p>calculou, por um processo diferente e de grande simplicidade,</p><p>o raio da Terra.</p><p>Admita que o valor calculado por Eratóstenes foi de 6316 km.</p><p>O Rodrigo calculou o raio da Terra pelo método acima descrito.</p><p>Utilizando um teodolito, obteve, a partir do cume da ilha do</p><p>Pico, α</p><p>= 1,5564º.</p><p>A altitude do Pico é 2,35 km.</p><p>Determine a diferença entre os valores obtidos pelos dois métodos.</p><p>Apresente o resultado arredondado às unidades.</p><p>Sugestão: Comece por calcular o valor obtido pelo Rodrigo, usando a igualdade .</p><p>FIM</p><p>cos</p><p>cos</p><p>h</p><p>R</p><p>α</p><p>=</p><p>1 − α</p><p>cos</p><p>cos</p><p>h</p><p>R</p><p>α</p><p>=</p><p>1 − α</p><p>BC R h= +</p><p>Prova 735 • Página 9/ 11</p><p>�</p><p>�</p><p>��</p><p>�</p><p>�</p><p>�</p><p>�</p><p>Fig. 4</p><p>Fig. 5</p><p>COTAÇÕES</p><p>GRUPO I ..................................................................................................................................... 30 pontos</p><p>1. ............................................................................................................................. 10 pontos</p><p>2. ............................................................................................................................. 20 pontos</p><p>GRUPO II</p>
<p>.................................................................................................................................... 80 pontos</p><p>1. ............................................................................................................................. 15 pontos</p><p>2. ............................................................................................................................ 15 pontos</p><p>3. ............................................................................................................................ 30 pontos</p><p>3.1. ............................................................................................... 15 pontos</p><p>3.2. ............................................................................................... 15 pontos</p><p>4. ............................................................................................................................ 20 pontos</p><p>GRUPO III ................................................................................................................................... 20 pontos</p><p>GRUPO IV ................................................................................................................................... 30 pontos</p><p>1. ............................................................................................................................. 15 pontos</p><p>2. ............................................................................................................................. 15 pontos</p><p>GRUPO V .................................................................................................................................... 40 pontos</p><p>1. ............................................................................................................................. 20 pontos</p><p>2. ............................................................................................................................. 20 pontos</p><p>_____________</p><p>TOTAL .............................................................. 200 pontos</p><p>Prova 735 • Página 10/ 11</p><p>Prova 735 • Página 11/ 11</p><p>Comprimento de um arco</p><p>de circunferência</p><p>α r (α – amplitude, em radianos,</p><p>do ângulo ao centro; r – raio)</p><p>Áreas de figuras planas</p><p>Losango:</p><p>Trapézio:</p><p>× Altura</p><p>Polígono regular:</p><p>Semiperímetro × Apótema</p><p>Sector circular:</p><p>(α – amplitude, em radianos, do</p><p>ângulo ao centro; r – raio)</p><p>Áreas de superfícies</p><p>Área lateral de um cone:</p><p>π rg (r – raio da base; g – geratriz)</p><p>Área de uma superfície esférica:</p><p>4 π r2 (r – raio)</p><p>Área lateral de um cilindro recto:</p><p>2 π rg (r – raio da base; g – geratriz)</p><p>Volumes</p><p>Pirâmide: × Área da base × Altura</p><p>Cone: × Área da base × Altura</p><p>Esfera: π r3 (r – raio)</p><p>Cilindro: Área da base × Altura</p><p>Progressões</p><p>Soma dos n primeiros termos de uma</p><p>Progressão aritmética: × n</p><p>Progressão geométrica: u1 ×</p><p>Probabilidades e Estatística</p><p>Se X é uma variável aleatória discreta, de</p><p>valores x i com probabilidades pi, então</p><p>• média de X:</p><p>•</p><p>• desvio padrão de X:</p><p>•</p><p>Se X é uma variável aleatória normal, de média µ e</p><p>desvio padrão σ, então:</p><p>•</p><p>( ) ,</p><p>( ) ,</p><p>( ) , 3</p><p>P X</p><p>P X</p><p>P X</p><p>µ σ µ σ</p><p>µ σ µ σ</p><p>µ σ µ σ</p><p>− < < + ≈ 0 6827</p><p>−2 < < + 2 ≈ 0 9545</p><p>−3 < < + 3 ≈ 0 997</p><p>1( ) ... ( )n nx p x pσ µ µ 22</p><p>1= − + + −</p><p>... n nx p x pµ 1 1= + +</p><p>1 – rn</p><p>———–</p><p>1 – r</p><p>u1 + un———–</p><p>2</p><p>4—</p><p>3</p><p>1—</p><p>3</p><p>1—</p><p>3</p><p>α r2</p><p>——</p><p>2</p><p>Base maior + Base menor</p><p>———————————</p><p>2</p><p>Diagonal maior × Diagonal menor</p><p>———————————————</p><p>2</p><p>Formulário</p><p>EXAME NACIONAL DO ENSINO SECUNDÁRIO</p><p>Decreto-Lei n.º 74/2004, de 26 de Março</p><p>Prova Escrita de Matemática B</p><p>11.º/12.º Anos de Escolaridade</p><p>Prova 735/1.ª Fase 11 Páginas</p><p>Duração da Prova: 150 minutos. Tolerância: 30 minutos.</p><p>2009</p><p>COTAÇÕES</p><p>GRUPO I ................................................................................................................... 30 pontos</p><p>1. ....................................................................................................... 10 pontos</p><p>2. ....................................................................................................... 20 pontos</p><p>GRUPO II .................................................................................................................. 80 pontos</p><p>1. ....................................................................................................... 15 pontos</p><p>2. ...................................................................................................... 15 pontos</p><p>3. ...................................................................................................... 30 pontos</p><p>3.1. ......................................................................... 15 pontos</p><p>3.2. ......................................................................... 15 pontos</p><p>4. ...................................................................................................... 20 pontos</p><p>GRUPO III ................................................................................................................. 20 pontos</p><p>GRUPO IV ................................................................................................................. 30 pontos</p><p>1. ....................................................................................................... 15 pontos</p><p>2. ....................................................................................................... 15 pontos</p><p>GRUPO V .................................................................................................................. 40 pontos</p><p>1. ....................................................................................................... 20 pontos</p><p>2. ....................................................................................................... 20 pontos</p><p>______________</p><p>TOTAL ......................................... 200 pontos</p><p>Prova 735 • Página C/1/ 11</p><p>CRITÉRIOS GERAIS DE CLASSIFICAÇÃO</p><p>A classificação a atribuir a cada item é obrigatoriamente:</p><p>– um número inteiro;</p><p>– um dos valores resultantes da aplicação dos critérios gerais e específicos de classificação e previstos na</p><p>respectiva grelha de classificação.</p><p>As respostas que se revelem ilegíveis ou que não possam ser claramente identificadas são classificadas com</p><p>zero pontos. No entanto, em caso de omissão ou de engano na identificação de um item, o mesmo pode ser</p><p>classificado se, pela resposta apresentada, for possível identificá-lo inequivocamente.</p><p>Se o examinando responder a um mesmo item mais do que uma vez, não eliminando inequivocamente a(s)</p><p>resposta(s) que não deseja que seja(m) classificada(s), deve ser apenas considerada a resposta que surgir</p><p>em primeiro lugar.</p><p>Os critérios de classificação destes itens apresentam-se organizados por etapas e/ou por níveis de</p><p>desempenho. A cada nível de desempenho e a cada etapa corresponde uma dada pontuação.</p><p>Nos itens que apresentam critérios específicos de classificação organizados por níveis de desempenho é</p><p>atribuída, a cada um desses níveis, uma única pontuação. No caso de, ponderados todos os dados contidos</p><p>nos descritores, permanecerem dúvidas quanto ao nível a atribuir, deve optar-se pelo nível mais elevado</p><p>de entre os dois tidos em consideração. É classificada com zero pontos qualquer resposta que não atinja o</p><p>nível 1 de desempenho no domínio específico da disciplina.</p><p>Nos itens de resposta aberta com cotação igual ou superior a 20 pontos que impliquem a produção de um texto,</p><p>a classificação a atribuir traduz a avaliação simultânea das competências específicas da disciplina e das</p><p>competências de comunicação em língua portuguesa.</p><p>A avaliação das competências de comunicação escrita em língua portuguesa contribui para valorizar a</p><p>classificação atribuída ao desempenho no domínio das competências específicas da disciplina. Esta</p><p>valorização é cerca de 10% da cotação do item e faz-se de acordo com os níveis de desempenho descritos no</p><p>quadro seguinte.</p><p>Nível Descritor</p><p>3</p><p>Composição bem estruturada, sem erros de sintaxe, de pontuação e/ou de ortografia, ou</p><p>com erros esporádicos, cuja gravidade</p>
<p>não implique perda de inteligibilidade e/ou de</p><p>sentido.</p><p>2</p><p>Composição razoavelmente estruturada, com alguns erros de sintaxe, de pontuação e/ou</p><p>ortografia, cuja gravidade não implique perda de inteligibildade e/ou de sentido.</p><p>1</p><p>Composição sem estruturação aparente, com a presença de erros graves de sintaxe,</p><p>pontuação e/ou de ortografia, cuja gravidade implique perda frequente de inteligibilidade</p><p>e/ou de sentido.</p><p>A classificação da prova deve respeitar integralmente</p><p>os critérios gerais e específicos a seguir apresentados</p><p>Prova 735 • Página C/2/ 11</p><p>No caso de a resposta não atingir o nível 1 de desempenho no domínio específico da disciplina, a</p><p>classificação a atribuir é zero pontos.</p><p>Neste caso, não é classificado o desempenho no domínio da comunicação escrita em língua portuguesa.</p><p>No quadro seguinte, apresentam-se os critérios de classificação a aplicar em situações não descritas</p><p>anteriormente.</p><p>Situação Classificação</p><p>1. Classificação de um item cujo critério se apresenta</p><p>organizado por etapas.</p><p>A cotação indicada para cada etapa é a classificação</p><p>máxima que lhe é atribuível.</p><p>A classificação da resposta resulta da soma das</p><p>classificações das diferentes etapas, à qual,</p><p>eventualmente, se subtraem um ou dois pontos, de acordo</p><p>com o previsto na situação 12.</p><p>2. Classificação de uma etapa dividida em passos. A cotação indicada para cada passo é a classificação</p><p>máxima que lhe é atribuível.</p><p>A classificação da etapa resulta da soma das classificações</p><p>dos diferentes passos.</p><p>3. Classificação de um item ou de uma etapa cujo critério</p><p>se apresenta organizado por níveis de desempenho.</p><p>A resposta é enquadrada numa das descrições apresen-</p><p>tadas.</p><p>À classificação correspondente subtrai-se, eventualmente,</p><p>um ponto, de acordo com o previsto nas situações 7, 8</p><p>e/ou 16.</p><p>4. Utilização de processos de resolução não previstos no</p><p>critério específico de classificação.</p><p>É aceite e classificado qualquer processo de resolução</p><p>cientificamente correcto.</p><p>O critério específico deve ser adaptado ao processo de</p><p>resolução apresentado, mediante distribuição da cotação</p><p>do item pelas etapas* percorridas pelo examinando. Esta</p><p>adaptação do critério deve ser utilizada em todos os</p><p>processos de resolução análogos.</p><p>5. Apresentação apenas do resultado final, embora a</p><p>resolução do item exija cálculos e/ou justificações.</p><p>Deve ser atribuída a classificação de zero pontos.</p><p>6. Ausência de apresentação explícita de uma dada etapa. Se a resolução apresentada permite perceber, inequivoca-</p><p>mente, que a etapa foi percorrida, a mesma é classificada</p><p>com a cotação total para ela prevista.</p><p>7. Transposição incorrecta de dados do enunciado. Se o grau de dificuldade da resolução não diminuir, é</p><p>subtraído um ponto à classificação da etapa.</p><p>Se o grau de dificuldade da resolução da etapa diminuir, a</p><p>classificação máxima a atribuir a essa etapa deve ser a</p><p>parte inteira da metade da cotação prevista.</p><p>8. Ocorrência de um erro ocasional num cálculo. É subtraído um ponto à classificação da etapa em que o</p><p>erro ocorre.</p><p>9. Ocorrência de um erro que revela desconhecimento de</p><p>conceitos, de regras ou de propriedades.</p><p>A classificação máxima a atribuir nessa etapa deve ser a</p><p>parte inteira de metade da cotação prevista.</p><p>Prova 735 • Página C/3/ 11</p><p>* Em situações em que o critério é aplicável tanto a etapas como a passos, utiliza-se apenas o termo «etapas», por razões de</p><p>simplificação da apresentação.</p><p>Situação Classificação</p><p>10. Ocorrência de um erro na resolução de uma etapa. A etapa é classificada de acordo com o erro cometido.</p><p>As etapas subsequentes são classificadas de acordo com os</p><p>efeitos do erro cometido:</p><p>– se o grau de dificuldade das etapas subsequentes não</p><p>diminuir, estas são classificadas de acordo com os</p><p>critérios específicos de classificação;</p><p>– se o grau de dificuldade das etapas subsequentes</p><p>diminuir, a classificação máxima a atribuir a cada uma</p><p>delas deve ser a parte inteira de metade da cotação</p><p>prevista.</p><p>11. Resolução incompleta de uma etapa. Se à resolução da etapa faltar apenas o passo final, é</p><p>subtraído um ponto à classificação da etapa; caso contrário</p><p>a classificação máxima a atribuir deve ser a parte inteira de</p><p>metade da cotação prevista.</p><p>12. Apresentação de cálculos intermédios com um número</p><p>de casas decimais diferente do solicitado e/ou</p><p>apresentação de um arredondamento incorrecto.</p><p>É subtraído um ponto à classificação total da resposta.</p><p>13. A apresentação do resultado final não respeita a forma</p><p>solicitada. [Exemplo: é pedido o resultado em centíme-</p><p>tros, e o examinando apresenta-o em metros.]</p><p>É subtraído um ponto à classificação da etapa correspon-</p><p>dente à apresentação do resultado final.</p><p>14. Omissão da unidade de medida na apresentação do</p><p>resultado final [por exemplo, «15» em vez de «15</p><p>metros».]</p><p>A etapa relativa à apresentação do resultado final é</p><p>classificada com a cotação total para ela prevista.</p><p>15. Apresentação do resultado final com um número de</p><p>casas decimais diferente do solicitado, e/ou</p><p>apresentação do resultado final incorrectamente</p><p>arredondado.</p><p>É subtraído um ponto à classificação da etapa</p><p>correspondente à apresentação do resultado final.</p><p>16. Utilização de simbologias ou de expressões inequivo-</p><p>camente incorrectas do ponto de vista formal.</p><p>É subtraído um ponto à classificação total da resposta,</p><p>excepto:</p><p>– se as incorrecções ocorrerem apenas em etapas já</p><p>classificadas com zero pontos;</p><p>– nos casos de uso do símbolo de igualdade quando, em</p><p>rigor, deveria ter sido usado o símbolo de igualdade</p><p>aproximada.</p><p>Prova 735 • Página C/4/ 11</p><p>CRITÉRIOS ESPECÍFICOS DE CLASSIFICAÇÃO</p><p>GRUPO I</p><p>1. .................................................................................................................................................... 10 pontos</p><p>A resposta a este item deve ser classificada de acordo com os seguintes níveis de</p><p>desempenho.</p><p>Determina as coordenadas do ponto simétrico de C , relativamente ao eixo</p><p>das abcissas, (2, 2) ........................................................................................ 10 pontos</p><p>Determina as coordenadas do ponto simétrico de C , relativamente ao eixo</p><p>das ordenadas, (–2, –2)</p><p>Ou</p><p>Determina as coordenadas do ponto simétrico de C , relativamente à origem do</p><p>referencial, (–2, 2) ......................................................................................... 5 pontos</p><p>Apresenta as coordenadas do ponto C e não determina as coordenadas do</p><p>ponto simétrico de C , ou determina-as incorrectamente, de forma não prevista</p><p>nos descritores anteriores ................................................................................ 2 pontos</p><p>Outras situações .............................................................................................. 0 pontos</p><p>2. .................................................................................................................................................... 20 pontos</p><p>Determina a área de [EFGH ] ... (144) ................................................................. 2 pontos</p><p>Determina a área de [ABCD ] ... (12) .................................................................. 8 pontos</p><p>A área de [ABCD ] pode ser determinada por, pelo menos, dois processos.</p><p>1.º Processo:</p><p>Determinar a área de [ABCD ] como a soma das áreas de [BDC ] e</p><p>[DAB ]:</p><p>Determinar a área de [BDC ] ......................................................... 3 pontos</p><p>Determinar a área de [DAB ] ......................................................... 3 pontos</p><p>Determinar a soma das áreas ........................................................ 2 pontos</p><p>2.º Processo:</p><p>Determinar a área de [ABCD ], inscrevendo-o num polígono de maior área:</p><p>Determinar a área do polígono em que [ABCD ] está inscrito ..... 3 pontos</p><p>Determinar a área não ocupada por [ABCD ] nesse polígono ...... 3 pontos</p><p>Determinar a diferença entre as duas áreas ................................... 2 pontos</p><p>Determina o quociente pedido (ver nota)........................... 8 pontos</p><p>Justifica que o número</p>
<p>obtido é racional .................................................................. 2 pontos</p><p>Nota: Se o examinando responder 12, ou equivalente, esta etapa deve ser classificada com 6 pontos.</p><p> 1    12 </p><p>ou equivalente</p><p>Prova 735 • Página C/5/ 11</p><p>GRUPO II</p><p>1. .................................................................................................................................................... 15 pontos</p><p>A resposta a este item deve ser classificada de acordo com os seguintes</p><p>níveis de desempenho.</p><p>Determina o valor de P(X > 10) ≈ 16% .......................................................... 15 pontos</p><p>Determina o valor de P(X > 12) ........................................................................ 12 pontos</p><p>Determina o valor de P(X < 10) ...................................................................... 12 pontos</p><p>Determina o valor de P(6 < X < 10) ................................................................. 9 pontos</p><p>Determina o valor de P(X < 12) .......................................................................... 7 pontos</p><p>Determina o valor de P(4 < X < 12) ................................................................. 6 pontos</p><p>Determina o valor de P(2 < X < 14) ................................................................. 3 pontos</p><p>Outras situações ...................................................................................................... 0 pontos</p><p>2. .................................................................................................................................................... 15 pontos</p><p>Substitui x por 100 na equação ............................................................................ 5 pontos</p><p>Calcula o valor de y ................................................................................................ 8 pontos</p><p>Apresenta o resultado pedido ... (21, 26) ............................................................... 2 pontos</p><p>3. .................................................................................................................................................... 30 pontos</p><p>3.1. .......................................................................................................................... 15 pontos</p><p>Escreve a expressão (ou equivalente) (ver nota) ... 5 pontos</p><p>Indica h (3) = 32,775 ................................................................. 3 pontos</p><p>Indica h (0) = 94,8 ...................................................................... 3 pontos</p><p>Conclui que tvm h[0; 3] ≈ –20,7 ................................................. 4 pontos</p><p>Nota: Se o examinando se limitar a escrever a expressão (ou equivalente), esta</p><p>etapa deve ser classificada com 2 pontos.</p><p>( ) ( )h b h a</p><p>b a</p><p>−</p><p>−</p><p>( ) ( )h h3 − 0</p><p>3 − 0</p><p>Prova 735 • Página C/6/ 11</p><p>3.2. .......................................................................................................................... 15 pontos</p><p>Descrição correcta, no contexto da situação, do que aconteceu</p><p>no instante t = 5,4 (ver nota) .................................................... 6 pontos</p><p>Nota: Exemplo de resposta: «o instante t = 5,4 é aquele em que a águia captura a presa.»</p><p>Se o examinando se limitar a referir que o instante é aquele em que a águia atinge, no seu</p><p>voo, a distância mínima ao solo, devem ser atribuídos, nesta etapa, quatro pontos.</p><p>Justificação .................................................................................... 9 pontos</p><p>A justificação apresentada deve ser classificada de acordo com os</p><p>seguintes níveis de desempenho (ver nota):</p><p>– refere que a taxa de variação instantânea no instante</p><p>t = 5,4 muda de sinal negativo para sinal positivo, e</p><p>conclui que a função h atinge um mínimo nesse instante</p><p>(ver nota).</p><p>.................................................................................. 9 pontos</p><p>– refere que a taxa de variação instantânea no instante</p><p>t = 5,4 muda de sinal, e conclui que a função h atinge um</p><p>mínimo nesse instante.</p><p>.................................................................................. 6 pontos</p><p>– refere que a taxa de variação instantânea no instante</p><p>t = 5,4 muda de sinal positivo para sinal negativo, e</p><p>conclui que a função h atinge um máximo nesse instante.</p><p>.................................................................................. 4 pontos</p><p>– refere que a taxa de variação instantânea no instante</p><p>t = 5,4 muda o sinal negativo para sinal positivo, e não</p><p>conclui que a função h atinge um mínimo nesse instante.</p><p>.................................................................................. 2 pontos</p><p>– Outras situações ..................................................... 0 pontos</p><p>Nota: A relação entre o sinal da taxa de variação e a monotonia da função pode ser apresentada</p><p>através de um quadro.</p><p>Prova 735 • Página C/7/ 11</p><p>4. .................................................................................................................................................... 20 pontos</p><p>Apresenta-se, a seguir, um exemplo de resposta:</p><p>«A afirmação I) é falsa, porque, quando foram plantadas, as árvores das espécies P e C</p><p>tinham, respectivamente, 0,74m e 1,54m, aproximadamente, pelo que a diferença entre as</p><p>alturas das duas árvores era de 0,8m e não de 1,1m.</p><p>A afirmação II) é falsa, porque a abcissa do ponto de intersecção dos gráficos das duas</p><p>funções tem um valor aproximado de 6,3 anos e é a partir desse instante que a árvore da</p><p>espécie P fica mais alta do que a árvore da espécie C .</p><p>A afirmação III) é verdadeira, pois, atendendo às expressões apresentadas, conclui-se que</p><p>o gráfico da função P tem uma assimptota horizontal de equação y = 10 e que o gráfico da</p><p>função C tem uma assimptota horizontal de equação y = 6. No contexto do problema,</p><p>conclui-se que, com o decorrer do tempo, a altura da árvore da espécie P tenderá para</p><p>10 m, e a da espécie C para 6 m, pelo que a diferença de alturas tenderá para 4 m .»</p><p>Como o exemplo ilustra, a composição deve abordar os seguintes tópicos:</p><p>• uma razão que mostre que a afirmação I) é falsa;</p><p>• uma razão que mostre que a afirmação II) é falsa;</p><p>• uma razão que mostre que a afirmação III) é verdadeira (ver nota).</p><p>Na tabela seguinte, indica-se como deve ser classificado este item, de acordo com os níveis</p><p>de desempenho no domínio da comunicação escrita em língua portuguesa descritos nos</p><p>critérios gerais e com os níveis de desempenho no domínio específico da disciplina.</p><p>** Descritores apresentados nos critérios gerais.</p><p>** Apenas podem ser atribuídas classificações correspondentes a um dos valores constantes do</p><p>quadro. Não há lugar a classificações intermédias.</p><p>No caso de a resposta não atingir o nível 1 de desempenho no domínio específico da disciplina, a</p><p>classificação a atribuir é de zero pontos. Neste caso, não é classificado o desempenho no domínio</p><p>da comunicação escrita em língua portuguesa.</p><p>Nota: Relativamente ao terceiro tópico da composição, a simples utilização de uma tabela não</p><p>prova, por si, que a diferença de alturas entre as duas árvores tenda para 4 m.</p><p>Descritores do nível de desempenho no domínio</p><p>da comunicação escrita em língua portuguesa</p><p>Descritores do nível de desempenho</p><p>no domínio específico da disciplina</p><p>Níveis*</p><p>1 2 3</p><p>N</p><p>ív</p><p>e</p><p>is</p><p>**</p><p>3 A composição aborda, correctamente, os três tópicos. 18 19 20</p><p>2 A composição aborda, correctamente, apenas dois tópicos. 12 13 14</p><p>1 A composição aborda, correctamente, apenas um tópico. 6 7 8</p><p>Prova 735 • Página C/8/ 11</p><p>GRUPO III</p><p>........................................................................................................................................................ 20 pontos</p><p>Indica a função objectivo (L(x, y) = 1600x + 1200y) .................................... 1 pontos</p><p>Indica as restrições .................................................................................................. 8 pontos</p><p>3x + 2y ≤	24 .............................................................................. 2 pontos</p><p>5x</p>
<p>+ 5y ≤	45 .............................................................................. 2 pontos</p><p>y ≤	5 ............................................................................................ 2 pontos</p><p>x ≥	0 ............................................................................................. 1 pontos</p><p>y ≥	0 ............................................................................................ 1 pontos</p><p>Representa graficamente a região admissível ......................................................... 6 pontos</p><p>Representa correctamente 3x + 2y = 24 ................................. 2 pontos</p><p>Representa correctamente 5x + 5y = 45 ................................. 2 pontos</p><p>Representa correctamente y =	5 ................................................ 1 pontos</p><p>Assinala o polígono ....................................................................... 1 pontos</p><p>Calcula o valor da função objectivo em cada um dos vértices da região</p><p>admissível ou implementa o método gráfico ............................................................ 3 pontos</p><p>Conclui que devem ser produzidas 6 toneladas de PPremium e 3 toneladas de</p><p>PRegular ................................................................................................................ 2 pontos</p><p>Prova 735 • Página C/9/ 11</p><p>GRUPO IV</p><p>1. .................................................................................................................................................... 15 pontos</p><p>Regista os valores dos 3 cravos da pata dianteira esquerda (1.º, 2.º e 8.º), referidos</p><p>no enunciado ............................................................................................................. 3 pontos</p><p>Calcula o valor de cada um dos 5 cravos restantes (2 pontos por cada um) ......... 10 pontos</p><p>Calcula a soma ......................................................................................................... 2 pontos</p><p>2. .................................................................................................................................................... 15 pontos</p><p>Este item pode ser resolvido por, pelo menos, dois processos.</p><p>1.º Processo:</p><p>Reconhecer que se trata de uma progressão geométrica de razão igual a 2 e primeiro</p><p>termo igual a 0,01 euros (ou 1 cêntimo) ........................................................................ 7 pontos</p><p>Aplicar correctamente a fórmula da soma dos primeiros n termos de uma</p><p>progressão geométrica ............................................................................................. 5 pontos</p><p>Concluir que é superior a 4000000 de euros ......................................................... 3 pontos</p><p>2.º Processo:</p><p>Construir uma tabela dos termos da sequência das somas ............................................ 12 pontos</p><p>Concluir que é superior a 4000000 de euros ......................................................... 3 pontos</p><p>Nota: Se o examinando construir apenas uma tabela dos termos da progressão geométrica, esta etapa</p><p>deve ser classificada com 5 pontos.</p><p>3.º Processo:</p><p>Calcular o valor do 30.º termo (ou do 31.º, ou do 32.º) ................................................. 12 pontos</p><p>Concluir que é superior a 4000000 de euros ......................................................... 3 pontos</p><p>Prova 735 • Página C/10/ 11</p><p>GRUPO V</p><p>1. .................................................................................................................................................... 20 pontos</p><p>Estabelece ................................................................................... 5 pontos</p><p>Resolve a equação em ordem a R .......................................................................... 13 pontos</p><p>De obtém R cos α	+ h cos α	= R ............... 5 pontos</p><p>De R cos α	+ h cos α	= R obtém R (cos α	–1) = –h cos α .. 5 pontos</p><p>De R (cos α	–1) = –h cos α		obtém ................. 3 pontos</p><p>Conclui que ................................................................................ 2 pontos</p><p>2. .................................................................................................................................................... 20 pontos</p><p>Substitui na expressão os valores pertinentes .............................. 10 pontos</p><p>h = 2,35 km ................................................................................. 5 pontos</p><p>α = 1,5564º .................................................................................. 5 pontos</p><p>Calcula o valor de R ... (6367km) .......................................................................... 5 pontos</p><p>Indica a diferença dos valores obtidos pelos dois métodos</p><p>(6367 – 6316) ou (6316 – 6367) ....................................................................... 3 pontos</p><p>Apresenta o resultado pedido (51 ou –51) ............................................................ 2 pontos</p><p>cos</p><p>cos</p><p>h</p><p>R</p><p>α</p><p>α</p><p>=</p><p>1 −</p><p>cos</p><p>cos</p><p>h</p><p>R</p><p>α</p><p>α</p><p>  =   1 − </p><p>cos</p><p>cos</p><p>h</p><p>R</p><p>α</p><p>α</p><p>−</p><p>=</p><p>−1</p><p>cos</p><p>R</p><p>R h</p><p>α =</p><p>+</p><p>cos</p><p>R</p><p>R h</p><p>α =</p><p>+</p><p>Prova 735 • Página C/11/ 11</p><p>Proposta de Resolução do Exame de Matemática B</p><p>Cod. 735 – 1ª Fase 2009</p><p>GRUPO I</p><p>1. C(2, -2), logo as coordenadas do ponto simétrico de C relativamente a Ox são (2, 2).</p><p>2. A área do quadrilátero [ABCD] é a soma das áreas dos triângulos [ABD] e [BCD].</p><p>A área total do Stomachion é 12</p><p>2</p><p>=144</p><p>Logo, o quociente entre as duas áreas é</p><p>que é um número racional por ser uma razão de números inteiros.</p><p>GRUPO II</p><p>1. Numa distribuição normal, a probabilidade de um valor se situar no intervalo ]µ−σ,µ+σ[ neste caso ]6, 10[, é de</p><p>0,6827. Como a normal é uma distribuição simétrica relativamente à média, tem-se</p><p>A probabilidade pedida é aproximadamente 16%.</p><p>2.</p><p>A temperatura correspondente a uma profundidade de 100 metros é de cerca de 21,26°.</p><p>3.1.</p><p>m/s</p><p>3.2. A taxa de variação instantânea é negativa antes do instante t = 5,4 e positiva depois. Isso significa que h, a</p><p>distância da águia ao fundo do vale, é decrescente antes desse instante e crescente depois. Por isso, t = 5,4</p><p>corresponde ao mínimo da função h, ou seja, ao instante da captura, que se deu a uma distância h ≈ 0,7 m.</p><p>4. Considerem-se os seguintes factos, e os gráficos que se</p><p>podem visualizar na calculadora:</p><p>I. C(0)− P(0) ≈ 1,54 − 0,74 ≈ 0,80</p><p>II. O ponto de intersecção dos dois gráficos tem coordenadas</p><p>aproximadas (6,3; 2,5).</p><p>III. As duas funções dadas são modelos logísticos, logo ambas</p><p>admitem assímptotas horizontais quando x tende para +∞,</p><p>respectivamente y = 10 e y = 6 (parâmetros que figuram nos</p><p>numeradores das fracções).</p><p>Assim, podemos concluir que as afirmações I e II estão incorrectas porque a diferença entre as alturas no</p><p>momento em que as árvores foram plantadas (x = 0) é de cerca de 0,8 metros, ou seja 80 centímetros; ao fim</p><p>de pouco mais de seis anos as árvores atingem a mesma altura de cerca de 2,5 m e ao fim de 7 anos a altura</p><p>das árvores da espécie P é nitidamente maior do que a das árvores da espécie C.</p><p>A afirmação III está correcta, uma vez que com o passar dos anos a altura das árvores da espécie P tende a</p><p>estabilizar em cerca de 10 metros, enquanto a das árvores da espécie C tende a estabilizar em cerca de 6</p><p>metros.</p><p>GRUPO III</p><p>O lucro semanal é dado por L(x, y) = 1600 x + 1200 y (função</p><p>objectivo).</p><p>Restrições do problema</p><p>L(0,5) = 1200×5 = 6000</p><p>L(4,5) = 1600×4 + 1200×5 = 12400</p><p>L(6,3) = 1600×6 + 1200×3 = 13200</p><p>L(8,0) = 1600×8 = 12800</p><p>y=C(x)</p><p>y=P(x)</p><p>0,74</p><p>1,5</p><p>6,297</p><p>2,54</p><p>6</p><p>10</p><p>50</p><p>x</p><p>y</p><p>(9,0) (8,0)</p><p>(6,3)</p><p>(4,5)</p><p>(0,12)</p><p>(0,9)</p><p>(0,5)</p><p>O</p><p>O lucro máximo é de 13200 euros e corresponde a uma produção semanal de 6 toneladas de PPremium e 3</p><p>toneladas de PRegular.</p><p>Grupo IV</p><p>1. Os valores correspondentes aos primeiros oito cravos da ferradura da pata dianteira esquerda estão em</p><p>progressão geométrica em que o primeiro termo é 0,01€ e a razão é 2. Assim usando fórmula da soma dos</p><p>oito primeiros termos da progressão</p>
<p>tem-se que:</p><p>55,2</p><p>21</p><p>21</p><p>01,0 8</p><p>8</p><p>8 =⇔</p><p>−</p><p>−×= SS</p><p>Ou seja, 2,55€.</p><p>2. Considerando o modo como o valor correspondente aos 32 cravos evolui, temos novamente uma progressão</p><p>geométrica em que primeiro termo é 0,01€ e a razão é 2. Assim, o valor é dado por:</p><p>32</p><p>32 32</p><p>1 2</p><p>0,01 42949672</p><p>1 2</p><p>S S</p><p>−= × ⇔ ≈</p><p>−</p><p>Valor que é superior a 4000000€.</p><p>Grupo V</p><p>1.</p><p>( ) .</p><p>cos1</p><p>cos</p><p>cos1cos</p><p>coscoscoscoscoscos</p><p>α</p><p>ααα</p><p>αααααα</p><p>−</p><p>=⇔−=−⇔</p><p>−=−⇔=+⇔</p><p>+</p><p>=⇔=</p><p>h</p><p>RhR</p><p>hRRRhR</p><p>hR</p><p>R</p><p>BC</p><p>AC</p><p>2.</p><p>2,35 cos1º ,5564</p><p>6367</p><p>1 cos1º ,5564</p><p>R R</p><p>×= ⇔ ≈</p><p>−</p><p>A diferença entre o valor obtido pelo Rodrigo e o obtido pelo método de Eratóstenes é: 6367-6316= 51 km.</p><p>Prova 735 • Página 1/ 11</p><p>EXAME NACIONAL DO ENSINO SECUNDÁRIO</p><p>Decreto-Lei n.º 74/2004, de 26 de Março</p><p>Prova Escrita de Matemática B</p><p>11.º/12.º Anos de Escolaridade</p><p>Prova 735/2.ª Fase 11 Páginas</p><p>Duração da Prova: 150 minutos. Tolerância: 30 minutos.</p><p>2009</p><p>Utilize apenas caneta ou esferográfica de tinta indelével, azul ou preta, excepto nas respostas</p><p>que impliquem a elaboração de construções, de desenhos ou de outras representações, que</p><p>podem ser, primeiramente, elaborados a lápis, sendo, a seguir, passados a tinta.</p><p>Utilize a régua, o compasso, o esquadro, o transferidor e a calculadora gráfica sempre que for</p><p>necessário.</p><p>Não é permitido o uso de corrector. Em caso de engano, deve riscar, de forma inequívoca,</p><p>aquilo que pretende que não seja classificado.</p><p>Escreva, de forma legível, a numeração dos grupos e dos itens, bem como as respectivas</p><p>respostas. As respostas ilegíveis ou que não possam ser identificadas são classificadas com</p><p>zero pontos.</p><p>Para cada item, apresente apenas uma resposta. Se escrever mais do que uma resposta a um</p><p>mesmo item, apenas é classificada a resposta apresentada em primeiro lugar.</p><p>Em todas as respostas, indique todos os cálculos que tiver de efectuar e todas as justificações necessárias.</p><p>Sempre que, na resolução de um problema, recorrer à calculadora, apresente todos os elementos recolhidos</p><p>na sua utilização. Mais precisamente:</p><p>• sempre que recorrer às capacidades gráficas da calculadora, apresente o(s) gráfico(s) obtido(s), bem</p><p>como as coordenadas de pontos relevantes para a resolução do problema proposto (por exemplo,</p><p>coordenadas de pontos de intersecção de gráficos, máximos, mínimos, etc.);</p><p>• sempre que recorrer a uma tabela obtida na calculadora, apresente todas as linhas da tabela relevantes</p><p>para a resolução do problema proposto;</p><p>• sempre que recorrer a estatísticas obtidas na calculadora (média, desvio padrão, coeficiente de</p><p>correlação, declive e ordenada na origem de uma recta de regressão, etc.), apresente a(s) lista(s) que</p><p>introduziu na calculadora para a(s) obter.</p><p>A prova inclui, na página 11, um Formulário.</p><p>As cotações dos itens encontram-se no final do enunciado da prova.</p><p>Prova 735 • Página 2/ 11</p><p>GRUPO I</p><p>No Casino ALEA, em LA PLACE, um dos jogos de sorte preferidos é a</p><p>«Roleta das Somas».</p><p>A roleta está dividida em oito sectores iguais, numerados, como</p><p>mostra o esquema da figura 1.</p><p>Cada jogador executa duas jogadas.</p><p>Cada jogada consiste em fazer girar a roleta e, quando esta parar,</p><p>registar o número indicado.</p><p>Admita que, em cada jogada, cada sector tem a mesma probabilidade</p><p>de sair.</p><p>A pontuação que cada jogador obtém é a soma dos números saídos</p><p>nas duas jogadas.</p><p>1. Seja X a variável aleatória «Soma dos números saídos nas duas jogadas».</p><p>Complete a tabela de distribuição de probabilidades de X , apresentando os valores exactos de</p><p>probabilidades, na forma de dízima.</p><p>Para responder, copie a tabela para a sua folha de prova e preencha-a.</p><p>2. Em cada noite de jogo no casino ALEA, a «Roleta das Somas» é usada dezenas de vezes.</p><p>Para efeitos de controlo pelas autoridades competentes, os serviços do casino registam o número total de</p><p>jogadas realizadas em cada noite, especificando quantas vezes sai cada um dos três números diferentes</p><p>registados nos sectores (1, 2 e 3). Este procedimento é utilizado, principalmente, para se verificar que a</p><p>roleta não está viciada.</p><p>Numa certa noite, os serviços do casino registaram 820 jogadas efectuadas com a roleta. Na tabela</p><p>seguinte, apresentam-se as frequências relativas correspondentes ao número de vezes que cada um dos</p><p>três números diferentes saiu nas 820 jogadas.</p><p>Determine a média dos números saídos nas 820 jogadas efectuadas naquela noite.</p><p>Número 1 2 3</p><p>Frequência relativa 55% 20% 25%</p><p>xi 2 3 4 5 6</p><p>P (X = xi)</p><p>Prova 735 • Página 3/ 11</p><p>� �</p><p>�</p><p>��</p><p>�</p><p>��</p><p>Fig. 1</p><p>GRUPO II</p><p>Na figura 2, está representado um quadrado [ABCD ], cujos lados têm</p><p>comprimento �. Em cada um dos lados do quadrado, assinalou-se o respectivo</p><p>ponto médio.</p><p>Unindo os pontos médios, obteve-se o quadrado [PQRS ].</p><p>1. Prove que a área do quadrado [PQRS ] é metade da área do quadrado</p><p>[ABCD ], seja qual for o valor de �.</p><p>Sugestão: Poder-lhe-á ser útil começar por decompor o quadrado</p><p>[ABCD ] em quatro quadrados geometricamente iguais.</p><p>2. Um joalheiro criou uma colecção de peças numeradas (I, II, III, ...), com faces quadradas, de</p><p>4 centímetros de lado.</p><p>Na figura 3, que não está à escala, estão representados apenas os três primeiros exemplares dessa</p><p>colecção.</p><p>Fig. 3</p><p>• cada peça contém uma pedra preciosa, cuja face visível também tem a forma de um quadrado,</p><p>representado pela região sombreada. Tal como a figura 3 sugere, a dimensão da pedra preciosa vai</p><p>diminuindo ao longo da colecção;</p><p>• de cada peça para a seguinte, o joalheiro aplicou ao quadrado central o processo ilustrado na</p><p>figura 2;</p><p>• a face visível da pedra preciosa, de menor dimensão, é um quadrado de 0,25 cm2 de área.</p><p>Considere a sequência das áreas, em cm2, das faces visíveis das pedras preciosas utilizadas nesta</p><p>colecção de peças.</p><p>Determine a soma das áreas das faces visíveis das pedras preciosas de toda a colecção.</p><p>Sugestão: Comece por mostrar que 8 é o primeiro termo da sequência referida.</p><p>� ����� ���</p><p>���� ���� ����</p><p>Prova 735 • Página 4/ 11</p><p>� �</p><p>� �</p><p>��</p><p>�</p><p>�</p><p>�</p><p>Fig. 2</p><p>GRUPO III</p><p>Numa vila, o presidente da Junta de Freguesia vai inaugurar um mural rectangular na praça principal. Nesse</p><p>mural, será exposta uma tapeçaria.</p><p>No projecto, ilustrado na figura 4, o mural está representado pelo rectângulo maior, e a tapeçaria pelo</p><p>rectângulo menor, sombreado; x representa a medida, em metros, de um dos lados do mural.</p><p>Cada um dos lados da tapeçaria ficará paralelo a dois dos lados do mural, com margens de 0,5 m e de</p><p>1 m, como a figura ilustra.</p><p>O mural terá 26 m de perímetro.</p><p>Fig. 4</p><p>1. Mostre que as medidas, em metros, de dois lados não paralelos da tapeçaria, expressas em função</p><p>de x, com x ∈ ]1, 11[ , são dadas por x – 1 e 11 – x .</p><p>2. Mostre que a área da tapeçaria, A, em metros quadrados, em função de x, é dada por</p><p>A(x ) = –x2 + 12x – 11 , x ∈ ]1, 11[ .</p><p>3. Determine o valor de x , com x ∈ ]1, 11[ , para o qual a área da tapeçaria é máxima.</p><p>����</p><p>��</p><p>����</p><p>��</p><p>Prova 735 • Página 5/ 11</p><p>GRUPO IV</p><p>O Tomás gosta muito de aviões e costuma consultar com regularidade o sítio da internet do Instituto Nacional</p><p>de Estatística, onde se encontram publicados os dados referentes ao tráfego aéreo nos diversos aeroportos</p><p>portugueses.</p><p>Ao analisar as tabelas de Dezembro de 2007, referentes ao «número de aeronaves aterradas/descoladas</p><p>nos aeroportos nacionais por localização geográfica, tipo de tráfego e natureza do tráfego», o Tomás reparou</p><p>que o 1 era, com muita frequência, o algarismo inicial de cada número. Por exemplo, o número de aviões</p><p>aterrados num determinado aeroporto, nesse mês, foi de 157, número cujo algarismo inicial é 1.</p><p>Tal facto deixou o Tomás muito intrigado e curioso. Para investigar a situação, resolveu determinar a</p><p>percentagem de vezes em que cada algarismo, de 1 a 9, aparecia como algarismo inicial dos números</p><p>registados. Os resultados, apresentados no gráfico da figura 5, foram surpreendentes.</p><p>Fig. 5</p><p>1. Quando observou o gráfico, o Tomás considerou que uma função logarítmica era um bom modelo para</p><p>esta</p>
<p>distribuição. Depois de introduzir os dados nas listas da sua máquina calculadora, obteve, por</p><p>regressão logarítmica, o modelo</p><p>P = 26,6723 – 10,9399 × ln(S )</p><p>no qual P representa a percentagem de ocorrência do algarismo inicial S , com S∈{1,�2,�3,�4,�5,�6,�7,�8,�9},</p><p>e ln o logaritmo de base e .</p><p>1.1. O gráfico de barras obtido sugere, por exemplo, que o algarismo 8 apresenta uma frequência</p><p>relativa desajustada do modelo logarítmico.</p><p>Determine a diferença entre a percentagem observada no gráfico e a percentagem P , obtida por</p><p>aplicação do modelo, com aproximação às décimas.</p><p>Em cálculos intermédios, conserve, pelo menos, duas casas decimais.</p><p>1.2. Esta distribuição teve por base uma amostra de 216 dados.</p><p>Quantos números se iniciariam com o algarismo 1 , de acordo com o modelo encontrado?</p><p>��������</p><p>�������</p><p>�</p><p>�</p><p>�</p><p>�</p><p>�</p><p>��</p><p>�</p><p>�</p><p>�</p><p>��</p><p>��</p><p>��</p><p>��</p><p>��</p><p>��</p><p>�</p><p>�</p><p>�</p><p>� � � � � � �</p><p>�</p><p>�</p><p>�</p><p>��</p><p>�</p><p>�</p><p>Prova 735 • Página 6/ 11</p><p>2. A distribuição das frequências do algarismo inicial de muitas colecções de números recolhidos da</p><p>realidade, como dados fiscais e índices da Bolsa, é uma distribuição logarítmica.</p><p>Esta distribuição é conhecida por Lei de Benford:</p><p>na qual P(n ) designa a probabilidade de n ser o algarismo inicial de um número com</p><p>n ∈ {1,�2,�3,�4,�5,�6,�7,�8,�9} , e log designa o logaritmo de base 10 .</p><p>Admita que está perante uma colecção de números que segue a Lei de Benford.</p><p>Escolhe-se, ao acaso, um número dessa colecção. A probabilidade de ele começar por um certo</p><p>algarismo é 0,058 (valor arredondado às milésimas).</p><p>De que algarismo se trata?</p><p>( ) logP n</p><p>n</p><p> 1  = 1 +   </p><p>Prova 735 • Página 7/ 11</p><p>GRUPO V</p><p>O Carlos costuma passar as suas férias de Verão no Algarve, na zona da Ria Formosa.</p><p>Num determinado dia de Agosto, o Carlos acompanhou um amigo seu, o António, na apanha do marisco.</p><p>Para isso, foram de manhã cedo para um local adequado da ria, aproveitando a baixa-mar.</p><p>Admita que, nesse dia, o nível das águas do mar, M, em metros, registado pelo marégrafo local, foi dado,</p><p>aproximadamente, por:</p><p>Nesta expressão:</p><p>• a variável t representa o tempo, em horas, contado a partir das zero horas, desse dia;</p><p>• o argumento da função seno é medido em radianos.</p><p>1. O António sabia que, naquele local da ria, era possível efectuar a apanha de marisco enquanto o nível das</p><p>águas não excedesse 1,3m.</p><p>Determine, recorrendo às capacidades da sua calculadora, o período de tempo da manhã em que foi</p><p>possível efectuar a apanha do marisco.</p><p>Apresente os extremos desse período de tempo, em horas e minutos, com os minutos aproximados às</p><p>unidades.</p><p>Apresente o(s) gráfico(s) em que se baseou para dar a resposta.</p><p>Nos cálculos intermédios, utilize, pelo menos, quatro casas decimais.</p><p>M t t t( ) = +( ) + < <1 055 0 507 0 916 1 908 0 2, , , , comsen 44</p><p>Prova 735 • Página 8/ 11</p><p>2. Nesse mesmo dia, um barco ficou encalhado na ria, cerca das 06h 07min, quando o nível das águas do</p><p>mar era de 1,1 m.</p><p>No momento em que o barco foi desencalhado, o nível das águas do mar era de 2,2 m.</p><p>Na figura 6, apresenta-se um esboço do gráfico da função M, anteriormente referida, no qual estão</p><p>assinalados os pontos P (6,11; 1,1) e Q (23,53; 2,2) .</p><p>O gráfico não está à escala.</p><p>Os valores t = 6,11 e t = 23,53 representam, com aproximação às centésimas da hora,</p><p>respectivamente, o instante em que o barco ficou encalhado e o instante em que foi desencalhado.</p><p>Fig. 6</p><p>Dias mais tarde, foi publicada, num jornal local, uma notícia relativa a este incidente. Da notícia publicada,</p><p>apresenta-se o seguinte excerto:</p><p>«Numa zona da ria Formosa, pertencente ao concelho de Olhão, ficou encalhada, desde as</p><p>primeiras horas da manhã, uma pequena embarcação. Era já noite quando, com a ajuda de um</p><p>rebocador, se conseguiu libertar o barco encalhado. Tinham passado, entretanto, cerca de 17 horas e</p><p>42 minutos, desde o momento em que o barco ficou preso.</p><p>No momento em que o barco foi desencalhado, o nível das águas do mar subia a uma taxa</p><p>aproximada de 0,7 metros por hora.</p><p>Desde a última baixa-mar ocorrida horas antes, a maré já tinha subido cerca de 1,5 metros, o que</p><p>facilitou os trabalhos de resgate do barco.»</p><p>Elabore uma pequena composição em que refira a correcção/ incorrecção de cada um dos três valores</p><p>numéricos apresentados na notícia.</p><p>Recorra, para o efeito, à sua calculadora.</p><p>Nos cálculos intermédios, utilize, pelo menos, três casas decimais.</p><p>FIM</p><p>�</p><p>�</p><p>���</p><p>����</p><p>��</p><p>��</p><p>��</p><p>��</p><p>�</p><p>�</p><p>�����</p><p>��</p><p>���</p><p>���</p><p>�</p><p>��</p><p>Prova 735 • Página 9/ 11</p><p>COTAÇÕES</p><p>GRUPO I ..................................................................................................................................... 30 pontos</p><p>1. ............................................................................................................................. 15 pontos</p><p>2. ............................................................................................................................ 15 pontos</p><p>GRUPO II .................................................................................................................................... 35 pontos</p><p>1. ............................................................................................................................. 15 pontos</p><p>2. ............................................................................................................................. 20 pontos</p><p>GRUPO III ................................................................................................................................... 45 pontos</p><p>1. ............................................................................................................................. 15 pontos</p><p>2. ............................................................................................................................. 10 pontos</p><p>3. ............................................................................................................................. 20 pontos</p><p>GRUPO IV ................................................................................................................................... 50 pontos</p><p>1. ............................................................................................................................. 30 pontos</p><p>1.1. ............................................................................................... 15 pontos</p><p>1.2. ............................................................................................... 15 pontos</p><p>2. ............................................................................................................................. 20 pontos</p><p>GRUPO V .................................................................................................................................... 40 pontos</p><p>1. ............................................................................................................................. 20 pontos</p><p>2. ............................................................................................................................. 20 pontos</p><p>______________</p><p>TOTAL .............................................................. 200 pontos</p><p>Prova 735 • Página 10/ 11</p><p>Prova 735 • Página 11/ 11</p><p>Comprimento de um arco</p><p>de circunferência</p><p>α r (α – amplitude, em radianos,</p><p>do ângulo ao centro; r – raio)</p><p>Áreas de figuras planas</p><p>Losango:</p><p>Trapézio:</p><p>× Altura</p><p>Polígono regular:</p><p>Semiperímetro × Apótema</p><p>Sector circular:</p><p>(α – amplitude, em radianos, do</p><p>ângulo ao centro; r – raio)</p><p>Áreas de superfícies</p><p>Área lateral de um cone:</p><p>π rg (r – raio da base; g – geratriz)</p><p>Área de uma superfície esférica:</p><p>4 π r2 (r – raio)</p><p>Área lateral de um cilindro recto:</p><p>2 π rg (r – raio da base; g – geratriz)</p><p>Volumes</p><p>Pirâmide: × Área da base × Altura</p><p>Cone: × Área da base × Altura</p><p>Esfera: π r3 (r – raio)</p><p>Cilindro: Área da base × Altura</p><p>Progressões</p><p>Soma dos n primeiros termos de uma</p><p>Progressão aritmética: × n</p><p>Progressão geométrica: u1 ×</p><p>Probabilidades e Estatística</p><p>Se</p>
<p>35</p><p>5.1. .......................................................................... 10</p><p>5.2. .......................................................................... 10</p><p>5.3. .......................................................................... 15</p><p>6. ............................................................................................. 30</p><p>TOTAL .................................................................................................. 200</p><p>735/11</p><p>Formulário</p><p>Comprimento de um arco de circunferência</p><p>α α< + < ( )amplitude, em radianos, do ângulo ao centro raio;</p><p>Áreas de figuras planas</p><p>Losango: H3+198+67+39<‚H3+198+67/89<</p><p>#</p><p>Trapézio: F+=/7+39</F+=/7/89<</p><p># ‚E6>?<+</p><p>Polígono regular: Semiperímetro Apótema‚</p><p>Sector circular: α <</p><p>#</p><p># (α+ amplitude,</p><p>em radianos, do ângulo ao centro raio; < )</p><p>Áreas de superfícies</p><p>Área lateral de um cone: 1 < 1</p><p>( )< 1 raio da base geratriz;</p><p>Área de uma superfície esférica: % <1 #</p><p>( )< raio</p><p>Volumes</p><p>Pirâmide: "$ ‚ Área da base Altura‚</p><p>Cone: "$ ‚ Área da base Altura‚</p><p>Esfera: %$ $1 ( )< < raio</p><p>Progressões</p><p>Soma dos primeiros termos de uma8</p><p>Prog. Aritmética: ? "?</p><p>#</p><p>" 8 ‚ 8</p><p>Prog. Geométrica: ? ‚"</p><p>") <</p><p>") <</p><p>8</p><p>V.S.F.F.735/C/1</p><p>PROVA 735/C/10 Págs.</p><p>EXAME NACIONAL DO ENSINO SECUNDÁRIO</p><p>11.º Ano de Escolaridade (Decreto-Lei n.º 74/2004, de 26 de Março)</p><p>Curso Científico-Humanístico</p><p>de Artes Visuais</p><p>Duração da prova: 150 minutos 1.ª FASE</p><p>2006</p><p>PROVA ESCRITA DE MATEMÁTICA - B</p><p>COTAÇÕES</p><p>1.</p><p>1.1. ................................................................................................................. 10 pontos</p><p>1.2. ................................................................................................................. 20 pontos</p><p>2.</p><p>2.1. ................................................................................................................. 10 pontos</p><p>2.2. ................................................................................................................. 10 pontos</p><p>2.3. ................................................................................................................. 10 pontos</p><p>3.</p><p>3.1. ................................................................................................................ 15 pontos</p><p>3.2. ................................................................................................................. 15 pontos</p><p>4.</p><p>4.1.</p><p>4.1.1. ..................................................................................................... 15 pontos</p><p>4.1.2. ..................................................................................................... 15 pontos</p><p>4.2. ................................................................................................................. 15 pontos</p><p>5.</p><p>5.1. ................................................................................................................. 10 pontos</p><p>5.2. ................................................................................................................. 10 pontos</p><p>5.3. ................................................................................................................. 15 pontos</p><p>6. ........................................................................................................................... 3 0 pontos</p><p>TOTAL 200 pontos ..........................................................</p><p>735/C/2</p><p>CRITÉRIOS DE CLASSIFICAÇÃO</p><p>Critérios gerais</p><p>1. Se o examinando se enganar na identificação do item a que está a responder, ou se a omitir,</p><p>mas, pela resolução apresentada, for possível identificá-lo inequivocamente, a resposta</p><p>deve ser vista e classificada.</p><p>2. Se o examinando apresentar mais do que uma resposta a um item, e não indicar, de forma</p><p>inequívoca, a que pretende que seja classificada, deve ser vista e classificada apenas a que</p><p>se encontra em primeiro lugar, na folha de resposta.</p><p>3. A cotação a atribuir a cada item deverá ser sempre um número inteiro, não negativo, de</p><p>pontos.</p><p>4. Num item em que a respectiva resolução exija cálculos e/ou justificações, a cotação a</p><p>atribuir deve estar de acordo com o seguinte critério:</p><p>• Se o examinando se limitar a apresentar o resultado final, a cotação deve ser de 0 (zero)</p><p>pontos.</p><p>• Se o examinando não se limitar a apresentar o resultado final, a cotação deve ser a soma</p><p>algébrica das cotações atribuídas a cada etapa, de acordo com o disposto nos pontos 6,</p><p>7, 8, 9 e 10 destes critérios gerais, e das penalizações previstas nos pontos 11 e 12</p><p>destes critérios gerais. Se a soma for negativa, a cotação a atribuir é de 0 (zero) pontos.</p><p>5. Alguns itens da prova podem ser correctamente resolvidos por mais do que um processo.</p><p>Sempre que o examinando utilizar um processo de resolução não contemplado nos critérios</p><p>específicos, caberá ao professor classificador adoptar um critério de distribuição da cotação</p><p>que julgue adequado e utilizá-lo em situações idênticas. Salienta-se que deve ser aceite</p><p>qualquer processo cientificamente correcto, mesmo que envolva conhecimentos não</p><p>contemplados no programa da disciplina.</p><p>6. A cotação de cada item está subdividida pelas etapas que o examinando deve percorrer</p><p>para o resolver.</p><p>6.1. Em cada etapa, a cotação indicada é a máxima a atribuir.</p><p>6.2. O classificador não pode subdividir, em cotações parcelares, a cotação atribuída a</p><p>cada etapa.</p><p>Caso uma etapa envolva um único passo, testando apenas o conhecimento de um só</p><p>conceito ou propriedade, e a sua resolução não esteja completamente correcta, deve</p><p>ser atribuída a cotação de 0 (zero) pontos.</p><p>Caso uma etapa envolva mais do que um passo (por exemplo, a resolução de uma</p><p>equação, a obtenção de uma expressão em função de uma variável, etc.) e a sua</p><p>resolução esteja incompleta, ou contenha incorrecções, a cotação a atribuir deve estar</p><p>de acordo com o grau de incompletude e/ou a gravidade dos erros cometidos. Por</p><p>exemplo:</p><p>• erros de contas ocasionais devem ser penalizados em um ponto;</p><p>• erros que revelem desconhecimento de conceitos, regras ou propriedades devem</p><p>ser penalizados em, pelo menos, metade da cotação da etapa;</p><p>• transposições erradas de dados do enunciado devem ser penalizadas em um ponto,</p><p>desde que o grau de dificuldade da etapa não diminua;</p><p>• transposições erradas de dados do enunciado devem ser penalizadas em, pelo</p><p>menos, metade da cotação da etapa, caso o grau de dificuldade da etapa diminua.</p><p>V.S.F.F.735/C/3</p><p>6.3. Nas etapas cuja cotação se encontra discriminada por níveis de desempenho, o</p><p>classificador deve enquadrar a resposta do examinando numa das descrições</p><p>apresentadas. O classificador não pode atribuir uma cotação diferente das indicadas.</p><p>6.4. No caso de o examinando cometer um erro numa das etapas, as etapas subsequentes</p><p>devem merecer a respectiva cotação, desde que o grau de dificuldade não tenha</p><p>diminuído, e o examinando as execute correctamente, de acordo com o erro que</p><p>cometeu.</p><p>6.5. Caso o examinando cometa, numa etapa, um erro que diminua o grau de dificuldade</p><p>das etapas subsequentes, cabe ao classificador decidir a cotação máxima a atribuir a</p><p>cada uma destas etapas. Em particular, se, devido a um erro cometido pelo</p><p>examinando, o grau de dificuldade das etapas seguintes diminuir significativamente, a</p><p>cotação máxima a atribuir a cada uma delas não deverá exceder metade da cotação</p><p>indicada.</p><p>6.6. Pode acontecer que o examinando, ao resolver um item, não percorra explicitamente</p><p>todas as etapas previstas nos critérios específicos. Todas as etapas não percorridas</p><p>explicitamente pelo examinando, mas cuja utilização e/ou conhecimento estejam</p><p>inequivocamente implícitos na resolução do item, devem receber a cotação indicada.</p><p>7. Existem, por vezes, etapas em que está previsto o recurso à calculadora. Nessas etapas, os</p><p>critérios específicos subdividem-se em: «Explicação do método utilizado» e «Apresentação</p><p>do(s) valor(es)».</p><p>7.1. Explicação do método utilizado:</p><p>De acordo com as instruções gerais para a realização da prova, o examinando deve</p><p>apresentar todos os elementos recolhidos</p>
<p>X é uma variável aleatória discreta, de</p><p>valores x i com probabilidades pi, então</p><p>• média de X:</p><p>•</p><p>• desvio padrão de X:</p><p>•</p><p>Se X é uma variável aleatória normal, de média µ e</p><p>desvio padrão σ, então:</p><p>•</p><p>( ) ,</p><p>( ) ,</p><p>( ) , 3</p><p>P X</p><p>P X</p><p>P X</p><p>µ σ µ σ</p><p>µ σ µ σ</p><p>µ σ µ σ</p><p>− < < + ≈ 0 6827</p><p>−2 < < + 2 ≈ 0 9545</p><p>−3 < < + 3 ≈ 0 997</p><p>1( ) ... ( )n nx p x pσ µ µ 22</p><p>1= − + + −</p><p>... n nx p x pµ 1 1= + +</p><p>1 – rn</p><p>———–</p><p>1 – r</p><p>u1 + un———–</p><p>2</p><p>4—</p><p>3</p><p>1—</p><p>3</p><p>1—</p><p>3</p><p>α r2</p><p>——</p><p>2</p><p>Base maior + Base menor</p><p>———————————</p><p>2</p><p>Diagonal maior × Diagonal menor</p><p>———————————————</p><p>2</p><p>Formulário</p><p>EXAME NACIONAL DO ENSINO SECUNDÁRIO</p><p>Decreto-Lei n.º 74/2004, de 26 de Março</p><p>Prova Escrita de Matemática B</p><p>11.º/12.º Anos de Escolaridade</p><p>Prova 735/2.ª Fase 12 Páginas</p><p>Duração da Prova: 150 minutos. Tolerância: 30 minutos.</p><p>2009</p><p>COTAÇÕES</p><p>GRUPO I ................................................................................................................... 30 pontos</p><p>1. ....................................................................................................... 15 pontos</p><p>2. ....................................................................................................... 15 pontos</p><p>GRUPO II .................................................................................................................. 35 pontos</p><p>1. ....................................................................................................... 15 pontos</p><p>2. ...................................................................................................... 20 pontos</p><p>GRUPO III ................................................................................................................. 45 pontos</p><p>1. ....................................................................................................... 15 pontos</p><p>2. ....................................................................................................... 10 pontos</p><p>3. ....................................................................................................... 20 pontos</p><p>GRUPO IV ................................................................................................................. 50 pontos</p><p>1. ....................................................................................................... 30 pontos</p><p>1.1. ......................................................................... 15 pontos</p><p>1.2. ......................................................................... 15 pontos</p><p>2. ....................................................................................................... 20 pontos</p><p>GRUPO V .................................................................................................................. 40 pontos</p><p>1. ....................................................................................................... 20 pontos</p><p>2. ....................................................................................................... 20 pontos</p><p>_____________</p><p>TOTAL ......................................... 200 pontos</p><p>Prova 735 • Página C/1/ 12</p><p>CRITÉRIOS GERAIS DE CLASSIFICAÇÃO</p><p>A classificação a atribuir a cada item é obrigatoriamente:</p><p>– um número inteiro;</p><p>– um dos valores resultantes da aplicação dos critérios gerais e específicos de classificação e previstos na</p><p>respectiva grelha de classificação.</p><p>As respostas que se revelem ilegíveis ou que não possam ser claramente identificadas são classificadas com</p><p>zero pontos. No entanto, em caso de omissão ou de engano na identificação de um item, o mesmo pode ser</p><p>classificado se, pela resposta apresentada, for possível identificá-lo inequivocamente.</p><p>Se o examinando responder a um mesmo item mais do que uma vez, não eliminando inequivocamente a(s)</p><p>resposta(s) que não deseja que seja(m) classificada(s), deve ser apenas considerada a resposta que surgir</p><p>em primeiro lugar.</p><p>Os critérios de classificação destes itens apresentam-se organizados por etapas e/ou por níveis de</p><p>desempenho. A cada nível de desempenho e a cada etapa corresponde uma dada pontuação.</p><p>Nos itens que apresentam critérios específicos de classificação organizados por níveis de desempenho é</p><p>atribuída, a cada um desses níveis, uma única pontuação. No caso de, ponderados todos os dados contidos</p><p>nos descritores, permanecerem dúvidas quanto ao nível a atribuir, deve optar-se pelo nível mais elevado</p><p>de entre os dois tidos em consideração. É classificada com zero pontos qualquer resposta que não atinja o</p><p>nível 1 de desempenho no domínio específico da disciplina.</p><p>Nos itens de resposta aberta com cotação igual ou superior a 20 pontos que impliquem a produção de um texto,</p><p>a classificação a atribuir traduz a avaliação simultânea das competências específicas da disciplina e das</p><p>competências de comunicação em língua portuguesa.</p><p>A avaliação das competências de comunicação escrita em língua portuguesa contribui para valorizar a</p><p>classificação atribuída ao desempenho no domínio das competências específicas da disciplina. Esta</p><p>valorização é cerca de 10% da cotação do item e faz-se de acordo com os níveis de desempenho descritos no</p><p>quadro seguinte.</p><p>Nível Descritor</p><p>3</p><p>Composição bem estruturada, sem erros de sintaxe, de pontuação e/ou de ortografia, ou</p><p>com erros esporádicos, cuja gravidade não implique perda de inteligibilidade e/ou de</p><p>sentido.</p><p>2</p><p>Composição razoavelmente estruturada, com alguns erros de sintaxe, de pontuação e/ou</p><p>ortografia, cuja gravidade não implique perda de inteligibildade e/ou de sentido.</p><p>1</p><p>Composição sem estruturação aparente, com a presença de erros graves de sintaxe,</p><p>pontuação e/ou de ortografia, cuja gravidade implique perda frequente de inteligibilidade</p><p>e/ou de sentido.</p><p>A classificação da prova deve respeitar integralmente</p><p>os critérios gerais e específicos a seguir apresentados</p><p>Prova 735 • Página C/2/ 12</p><p>No caso de a resposta não atingir o nível 1 de desempenho no domínio específico da disciplina, a</p><p>classificação a atribuir é zero pontos.</p><p>Neste caso, não é classificado o desempenho no domínio da comunicação escrita em língua portuguesa.</p><p>No quadro seguinte, apresentam-se os critérios de classificação a aplicar em situações não descritas</p><p>anteriormente.</p><p>Situação Classificação</p><p>1. Classificação de um item cujo critério se apresenta</p><p>organizado por etapas.</p><p>A cotação indicada para cada etapa é a classificação</p><p>máxima que lhe é atribuível.</p><p>A classificação da resposta resulta da soma das</p><p>classificações das diferentes etapas, à qual,</p><p>eventualmente, se subtraem um ou dois pontos, de acordo</p><p>com o previsto nas situações 12 e/ou 16.</p><p>2. Classificação de uma etapa dividida em passos. A cotação indicada para cada passo é a classificação</p><p>máxima que lhe é atribuível.</p><p>A classificação da etapa resulta da soma das classificações</p><p>dos diferentes passos.</p><p>3. Classificação de um item ou de uma etapa cujo critério</p><p>se apresenta organizado por níveis de desempenho.</p><p>A resposta é enquadrada numa das descrições apresen-</p><p>tadas.</p><p>À classificação correspondente subtrai-se, eventualmente,</p><p>um ponto, de acordo com o previsto nas situações 7, 8</p><p>e/ou 16.</p><p>4. Utilização de processos de resolução não previstos no</p><p>critério específico de classificação.</p><p>É aceite e classificado qualquer processo de resolução</p><p>cientificamente correcto.</p><p>O critério específico deve ser adaptado ao processo de</p><p>resolução apresentado, mediante distribuição da cotação</p><p>do item pelas etapas* percorridas pelo examinando. Esta</p><p>adaptação do critério deve ser utilizada em todos os</p><p>processos de resolução análogos.</p><p>5. Apresentação apenas do resultado final, embora a</p><p>resolução do item exija cálculos e/ou justificações.</p><p>Deve ser atribuída a classificação de zero pontos.</p><p>6. Ausência de apresentação explícita de uma dada etapa. Se a resolução apresentada permite perceber, inequivoca-</p><p>mente, que a etapa foi percorrida, a mesma é classificada</p><p>com a cotação total para ela prevista.</p><p>7. Transposição incorrecta de dados do enunciado. Se o grau de dificuldade da resolução não diminuir, é</p><p>subtraído um ponto à classificação</p>
<p>da etapa.</p><p>Se o grau de dificuldade da resolução da etapa diminuir, a</p><p>classificação máxima a atribuir a essa etapa deve ser a</p><p>parte inteira da metade da cotação prevista.</p><p>8. Ocorrência de um erro ocasional num cálculo. É subtraído um ponto à classificação da etapa em que o</p><p>erro ocorre.</p><p>9. Ocorrência de um erro que revela desconhecimento de</p><p>conceitos, de regras ou de propriedades.</p><p>A classificação máxima a atribuir nessa etapa deve ser a</p><p>parte inteira de metade da cotação prevista.</p><p>Prova 735 • Página C/3/ 12</p><p>* Em situações em que o critério é aplicável tanto a etapas como a passos, utiliza-se apenas o termo «etapas», por razões de</p><p>simplificação da apresentação.</p><p>Situação Classificação</p><p>10. Ocorrência de um erro na resolução de uma etapa. A etapa é classificada de acordo com o erro cometido.</p><p>As etapas subsequentes são classificadas de acordo com os</p><p>efeitos do erro cometido:</p><p>– se o grau de dificuldade das etapas subsequentes não</p><p>diminuir, estas são classificadas de acordo com os</p><p>critérios específicos de classificação;</p><p>– se o grau de dificuldade das etapas subsequentes</p><p>diminuir, a classificação máxima a atribuir a cada uma</p><p>delas deve ser a parte inteira de metade da cotação</p><p>prevista.</p><p>11. Resolução incompleta de uma etapa. Se à resolução da etapa faltar apenas o passo final, é</p><p>subtraído um ponto à classificação da etapa; caso contrário</p><p>a classificação máxima a atribuir deve ser a parte inteira de</p><p>metade da cotação prevista.</p><p>12. Apresentação de cálculos intermédios com um número</p><p>de casas decimais diferente do solicitado e/ou</p><p>apresentação de um arredondamento incorrecto.</p><p>É subtraído um ponto à classificação total da resposta.</p><p>13. A apresentação do resultado final não respeita a forma</p><p>solicitada. [Exemplo: é pedido o resultado em centíme-</p><p>tros, e o examinando apresenta-o em metros.]</p><p>É subtraído um ponto à classificação da etapa correspon-</p><p>dente à apresentação do resultado final.</p><p>14. Omissão da unidade de medida na apresentação do</p><p>resultado final [por exemplo, «15» em vez de «15</p><p>metros».]</p><p>A etapa relativa à apresentação do resultado final é</p><p>classificada com a cotação total para ela prevista.</p><p>15. Apresentação do resultado final com um número de</p><p>casas decimais diferente do solicitado, e/ou</p><p>apresentação do resultado final incorrectamente</p><p>arredondado.</p><p>É subtraído um ponto à classificação da etapa</p><p>correspondente à apresentação do resultado final.</p><p>16. Utilização de simbologias ou de expressões inequivo-</p><p>camente incorrectas do ponto de vista formal.</p><p>É subtraído um ponto à classificação total da resposta,</p><p>excepto:</p><p>– se as incorrecções ocorrerem apenas em etapas já</p><p>classificadas com zero pontos;</p><p>– nos casos de uso do símbolo de igualdade quando, em</p><p>rigor, deveria ter sido usado o símbolo de igualdade</p><p>aproximada.</p><p>Prova 735 • Página C/4/ 12</p><p>CRITÉRIOS ESPECÍFICOS DE CLASSIFICAÇÃO</p><p>GRUPO I</p><p>1. .................................................................................................................................................... 15 pontos</p><p>Indica os casos possíveis (ver nota 1) .................................................................. 5 pontos</p><p>Preenche a tabela ... 10 (2 + 2 + 2 + 2 + 2) (ver nota 2) ............................ 10 pontos</p><p>Notas:</p><p>1. O examinando pode limitar-se a apresentar, por exemplo, uma tabela de dupla entrada ou um</p><p>diagrama de árvore, para indicar os casos possíveis.</p><p>2. Se o examinando apresentar alguma das probabilidades pedidas na forma de fracção, a</p><p>classificação nesta etapa deve ser desvalorizada em um ponto.</p><p>2. .................................................................................................................................................... 15 pontos</p><p>Escreve a expressão 1 × 0,55 + 2 × 0,20 + 3 × 0,25 (ou equivalente) (ver nota) .. 10 pontos</p><p>Determina a média (1,7) ......................................................................................... 5 pontos</p><p>Nota: Se o examinando se limitar a escrever a expressão 1 × 55 + 2 × 20 + 3 × 25 (ou equivalente),</p><p>esta etapa deve ser classificada em 5 pontos.</p><p>xi 2 3 4 5 6</p><p>P (X = xi) 0,25 0,25 0,3125 0,125 0,0625</p><p>Prova 735 • Página C/5/ 12</p><p>GRUPO II</p><p>1. .................................................................................................................................................... 15 pontos</p><p>Este item pode ser resolvido por, pelo menos, três processos.</p><p>1.º Processo (de acordo com a sugestão):</p><p>Subdividir o quadrado [ABCD ] em quatro quadrados [APOS ], [BQOP ],</p><p>[QCRO ], [SORD ], sendo O o centro do quadrado [ABCD ] ........................... 5 pontos</p><p>Referir que cada um destes quadrados está dividido em dois triângulos congruentes,</p><p>sendo um deles sombreado e o outro não sombreado................................................ 8 pontos</p><p>Concluir que ................................................................ 2 pontos</p><p>2.º Processo (de acordo com a sugestão):</p><p>Subdividir o quadrado [ABCD ] em dois rectângulos congruentes (por exemplo,</p><p>[APRD ] e [PBCR ] ............................................................................................... 5 pontos</p><p>Referir que em cada um dos rectângulos, a área do triângulo sombreado inscrito é</p><p>metade da área do rectângulo ................................................................................... 8 pontos</p><p>Concluir que ................................................................ 2 pontos</p><p>3.º Processo (ver nota):</p><p>Calcular a área do quadrado [ABCD ] ... (� 2) ...................................................... 3 pontos</p><p>Calcular a área do triângulo [PBQ ] ou de outro triângulo congruente a [PBQ ] ...... 7 pontos</p><p>Considerar .......................................................... 2 pontos</p><p>Escrever (ou equivalente) ................................. 2 pontos</p><p>Concluir que ............................................................. 3 pontos</p><p>Determinar a expressão da área da região não sombreada .............. 3 pontos</p><p>Concluir que ........................................................................... 2 pontos</p><p>Nota: No caso de o examinando particularizar, atribuindo um valor numérico a � (número real</p><p>positivo), e na resolução seguir o processo indicado, então as cotações máximas das</p><p>sucessivas etapas passam de (3 + 7 (2 + 2 + 3) + 3 + 2) para (1 + 3 (1 + 1 + 1) + 1 + 2), num</p><p>total de 7 pontos.</p><p>[ ]</p><p>[ ]</p><p>ABCD</p><p>PQRS</p><p>A</p><p>A =</p><p>2</p><p>ou</p><p>2 2 4    8 2  </p><p>� �</p><p>[ ]PBQA</p><p>2</p><p>=</p><p>8</p><p>�</p><p>[ ]PBQA</p><p>×</p><p>2 2=</p><p>2</p><p>� �</p><p>PB BQ= =</p><p>2</p><p>�</p><p>[ ]</p><p>[ ]</p><p>ABCD</p><p>PQRS</p><p>A</p><p>A =</p><p>2</p><p>[ ]</p><p>[ ]</p><p>ABCD</p><p>PQRS</p><p>A</p><p>A =</p><p>2</p><p>Prova 735 • Página C/6/ 12</p><p>4.º Processo (ver nota):</p><p>Calcular a área do quadrado [ABCD ] ... (�2) ...................................................... 3 pontos</p><p>Calcular a área do quadrado [PQRS ], aplicando o Teorema de Pitágoras ........... 10 pontos</p><p>Escrever (ou equivalente) ............................ 4 pontos</p><p>Escrever (ou equivalente) ................................ 3 pontos</p><p>Escrever (ou equivalente) .......................................... 3 pontos</p><p>Concluir que ................................................................. 2 pontos</p><p>Nota: No caso de o examinando particularizar, atribuindo um valor numérico a � (número real</p><p>positivo) e na resolução seguir o processo indicado, então as cotações máximas das sucessivas</p><p>etapas passam de (3 + 10 (4 + 3 + 3) + 2) para (1 + 4 (2 + 1 + 1) + 2), num total de 7 pontos.</p><p>2. .................................................................................................................................................... 20 pontos</p><p>Mostra que 8 é o valor do primeiro termo da sequência........................................... 4 pontos</p><p>Mostra que são seis os termos da sequência (ver nota) ......................................... 6 pontos</p><p>Nota: Se o examinando considerar que são três os termos da sequência, esta etapa deve ser</p><p>classificada com zero pontos. Relativamente à classificação da última etapa, deve ser aplicado o</p><p>critério 10 dos critérios gerais de classificação.</p><p>Esta etapa pode ser resolvida</p>
<p>por, pelo menos, dois processos.</p><p>1.º Processo:</p><p>Escrever: ................................................... 6 pontos</p><p>2.º Processo:</p><p>Escrever a condição (ou equivalente) ............. 3 pontos</p><p>Resolver a condição (n = 6) ............................................................ 3 pontos</p><p>Somar os seis termos ............................................................................................ 10 pontos</p><p>Esta etapa pode ser resolvida por, pelo menos, dois processos.</p><p>1.º Processo:</p><p>Escrever ......................................... 6 pontos</p><p>Calcular a soma (15,75) .................................................................. 4 pontos</p><p>, ,8 + 4 + 2 + 1 + 0 5 + 0 25</p><p>,</p><p>n−1 1 8× = 0 25  2 </p><p>; ; ; ; , ; ,8 4 2 1 0 5 0 25</p><p>[ ]</p><p>[ ]</p><p>ABCD</p><p>PQRS</p><p>A</p><p>A =</p><p>2</p><p>PQ</p><p>2</p><p>2</p><p>=</p><p>2</p><p>�</p><p>PQ</p><p>2 2</p><p>2</p><p>= +</p><p>4 4</p><p>� �</p><p>PQ</p><p>2 2</p><p>2       = +     2 2   </p><p>� �</p><p>Prova 735 • Página C/7/ 12</p><p>2.º Processo:</p><p>Escrever .......................................................... 4 pontos</p><p>Obter (ou equivalente)......................................... 4 pontos</p><p>Concluir que ................................................. 2 pontos´</p><p>GRUPO III</p><p>1. .................................................................................................................................................... 15 pontos</p><p>Esta etapa pode ser resolvida por, pelo menos, três processos.</p><p>1.º Processo:</p><p>Determina a medida de um dos lados da tapeçaria (x – 1) ................................... 5 pontos</p><p>Determina a medida de outro lado não paralelo (11 – x ) ................................... 10 pontos</p><p>Escreve (ou equivalente) ....................................... 7 pontos</p><p>Simplifica a expressão escrita (11 – x ) ......................................... 3 pontos</p><p>2.º Processo:</p><p>Mostra que (x – 1) + (x – 1) + (11 – x) + (11 – x) = 20 (ou equivalente) .. 8 pontos</p><p>Explica a diferença 26 – 20, pelas dimensões das margens ................................. 7 pontos</p><p>3.º Processo:</p><p>Escreve (x – 1) + 1 (ou equivalente), para a medida de um lado do mural ............ 4 pontos</p><p>Escreve (11 – x) + 2 (ou equivalente), para a medida do outro lado do mural ...... 4 pontos</p><p>Mostra que 2 [(x – 1) + 1] + 2 [(11 – x ) + 2] = 26 ......................................... 7 pontos</p><p>2. .................................................................................................................................................... 10 pontos</p><p>Determina a área da tapeçaria (–x 2 + 12x – 11) ............................................... 10 pontos</p><p>Escreve (x – 1)(11 – x ) .............................................................. 4 pontos</p><p>Desembaraça de parêntesis .......................................................... 3 pontos</p><p>Simplifica a expressão .................................................................... 3 pontos</p><p>x26 − 2</p><p>− 2</p><p>2</p><p>ou, S6</p><p> 63 = 15 75   4</p><p>S6</p><p>1</p><p>1 −</p><p>64= 8×</p><p>1</p><p>2</p><p>S</p><p>6</p><p>6</p><p> 1 1 −   2 </p><p>= 8×</p><p>1</p><p>1 −</p><p>2</p><p>Prova 735 • Página C/8/ 12</p><p>3. .................................................................................................................................................... 20 pontos</p><p>Este item pode ser resolvido por, pelo menos, dois processos.</p><p>1.º Processo:</p><p>Apresentar o gráfico da função A ............................................................................ 15 pontos</p><p>Respeitar o domínio de A (ver nota 1) ......................................... 5 pontos</p><p>Assinalar a ordenada do vértice da parábola ................................. 2 pontos</p><p>Assinalar a abcissa do vértice da parábola .................................... 4 pontos</p><p>Respeitar a forma parabólica .......................................................... 4 pontos</p><p>Indicar o maximizante da função (ver nota 2) (x = 6).......................................... 5 pontos</p><p>Notas:</p><p>1. No caso de algum dos pontos de coordenadas (1, 0) e (11, 0) não ser assinalado com bola</p><p>aberta, a classificação desta etapa deve ser desvalorizada em um ponto.</p><p>2. Não deve ser desvalorizada uma resposta que difira de 6, no máximo, numa décima. Qualquer</p><p>outra resposta deve ser classificada com zero pontos.</p><p>2.º Processo:</p><p>Referir que o gráfico da função A é uma parábola com a concavidade voltada para</p><p>baixo .......................................................................................................................... 5 pontos</p><p>Concluir que o vértice da parábola tem abcissa 6 ................................................... 10 pontos</p><p>Concluir que 6 é o maximizante da função .............................................................. 5 pontos</p><p>Prova 735 • Página C/9/ 12</p><p>GRUPO IV</p><p>1. .................................................................................................................................................... 30 pontos</p><p>1.1. ........................................................................................................................ 15 pontos</p><p>Indica a percentagem observada no gráfico (8) ............................ 3 pontos</p><p>Calcula P(8) (3,92) ...................................................................... 7 pontos</p><p>Calcula a diferença entre o valor observado e P(8) (4,1)</p><p>(ver nota)......................................................................................... 5 pontos</p><p>Nota: Se o examinando responder (–4,1), esta etapa deve ser classificada com 3 pontos.</p><p>1.2. ........................................................................................................................ 15 pontos</p><p>Determina P(1) = 26,6723 (ver nota 1) ..................................... 6 pontos</p><p>Determina 26,6723% de 216 ....................................................... 6 pontos</p><p>Conclui (57 ou 58) ou conclui (58) (ver nota 2)........................... 3 pontos</p><p>Notas:</p><p>1. Se o examinando considerar P(1) = 29, utilizando o gráfico em vez do modelo, esta etapa</p><p>deve ser classificada com 2 pontos.</p><p>2. Se o examinando concluir «57», esta etapa deve ser classificada com 2 pontos.</p><p>2. .................................................................................................................................................... 20 pontos</p><p>Este item pode ser resolvido por, pelo menos, três processos.</p><p>1.º Processo (ver nota):</p><p>Por substituição, verificar que P(7) ≈ 0,058 ...................................................... 15 pontos</p><p>Concluir a unicidade ................................................................................................. 5 pontos</p><p>Nota: Este processo pode ser executado recorrendo à tabela referente à função ,</p><p>obtida na calculadora gráfica.</p><p>2.º Processo:</p><p>Escrever a condição .......................................................... 5 pontos</p><p>Apresentar o gráfico da função definida por , num</p><p>subconjunto de ]–∞,		–1[ ∪	 ]0 , +∞[ que inclua o conjunto {1, 2, 3, ... 9} 5 pontos</p><p>Esboçar a recta de equação y = 0,058 ................................................................ 3 pontos</p><p>Assinalar a abcissa do ponto de intersecção ........................................................... 2 pontos</p><p>Concluir que se trata do algarismo 7 ....................................................................... 5 pontos</p><p>( ) logP x</p><p>x</p><p> 1 = 1 +   </p><p>log ,</p><p>n</p><p> 1  1 + = 0 058   </p><p>( ) logP x</p><p>x</p><p>1</p><p>= 1 +</p><p>     </p><p>Prova 735 • Página C/10/ 12</p><p>3.º Processo:</p><p>Escrever a condição ........................................................... 5 pontos</p><p>Resolver analiticamente a condição anterior ........................................................... 10 pontos</p><p>Concluir que se trata do algarismo 7 ....................................................................... 5 pontos</p><p>GRUPO V</p><p>1. .................................................................................................................................................... 20 pontos</p><p>Escreve a inequação M(t ) ≤ 1,3 (ou equivalente) (ver nota 1) .......................... 2 pontos</p><p>Apresenta correctamente o(s) gráfico(s) que permite(m)</p>
<p>resolver o problema ........ 8 pontos</p><p>Esboça o gráfico da função M , no domínio [0, 24] (ver notas 2 e 3) .. 5 pontos</p><p>Representa graficamente a recta horizontal, de equação y = 1,3 .... 1 pontos</p><p>Apresenta as abcissas dos 2 pontos de intersecção da recta com o</p><p>gráfico da função M , relevantes para a resposta ao problema</p><p>(ver nota 4) ........................................................................................... 2 pontos</p><p>Indica, em horas e minutos, os valores dos extremos do intervalo de tempo</p><p>pedido (ver notas 5 e 6) .......................................................................................... 10 pontos</p><p>5h 36min (ver notas 7 e 8) .......................................................... 5 pontos</p><p>9h 22min (ver notas 7 e 8) .......................................................... 5 pontos</p><p>Notas:</p><p>1. Se o examinando apresentar a equação M(t ) = 1,3 em vez da inequação e, na sua resposta,</p><p>resolver de forma correcta o problema, então a classificação desta etapa deve ser atribuída na</p><p>totalidade.</p><p>2. Se o gráfico não apresentar uma forma sinusoidal, não evidenciando a periodicidade da função seno,</p><p>esta etapa é desvalorizada em, pelo menos, 3 pontos.</p><p>3. Se o examinando não respeitar o domínio da função, a classificação desta etapa deve ser desvalorizada</p><p>em 2 pontos.</p><p>4. Se o examinando não apresentar as abcissas dos dois pontos de intersecção do gráfico de M com a</p><p>recta de equação y = 1,3 , que são relevantes para a resposta pedida, mas apenas assinalar um</p><p>desses valores, a classificação desta etapa é desvalorizada em metade da cotação.</p><p>5. Se o examinando apresentar como resposta, para além do período da manhã, também o período</p><p>tarde/noite, compreendido entre as 18h 00min e as 21h 46min, esta etapa deve ser classificada,</p><p>no máximo, com 6 pontos.</p><p>6. Nesta etapa só devem ser classificados os extremos do período de tempo requerido. O examinando</p><p>não tem que apresentar a resposta na forma de dupla desigualdade, nem na forma de intervalo. Porém,</p><p>se o examinando apresentar como resposta final algum dos seguintes intervalos: [5,6012; 9,3747];</p><p>[5,6012; 9,3747[; ]5,6012; 9,3747]; ]5,6012; 9,3747[, esta etapa deve ser classificada com</p><p>4 pontos.</p><p>7. Se o examinando se limitar a apresentar na forma decimal o valor pedido, não o convertendo em horas</p><p>e minutos, como requerido, é atribuída a classificação máxima de 2 pontos neste passo.</p><p>8. Se ao apresentar o resultado em horas e minutos o examinando cometer um erro de aproximação,</p><p>relativamente ao valor correcto, que não apresente um desvio superior a 1 minuto, a classificação do</p><p>passo correspondente é desvalorizado em 1 ponto.</p><p>log ,</p><p>n</p><p> 1  1 + = 0 058   </p><p>Prova 735 • Página C/11/ 12</p><p>2. .................................................................................................................................................... 20 pontos</p><p>Apresenta-se, a seguir, um exemplo de resposta:</p><p>«O barco esteve encalhado cerca de 17h 25 min e não 17 h 42 min , como é referido na</p><p>notícia (23,53 – 6,11 = 17,42 h e 0,42 × 60 ≈ 25min). No momento em que o barco</p><p>é desencalhado (t = 23,53h), o valor da taxa de variação é de, aproximadamente,</p><p>0,5 m/h e não 0,7 m/h , como é dito erradamente na notícia, o que significa, no contexto</p><p>da situação, que, nesse instante, o nível das águas do mar está a subir à taxa aproximada de</p><p>meio metro por hora. O nível das águas registado, no momento em que o barco foi</p><p>desencalhado, excedia em cerca de 1,3 metros o nível registado na baixa-mar</p><p>imediatamente anterior (2,199 – 0,853 = 1,346 ≈ 1,3m), pelo que o valor apresentado</p><p>de 1,5 metros é incorrecto.»</p><p>Como o exemplo ilustra, a composição deve abordar os seguintes tópicos, referindo que os</p><p>respectivos valores numéricos apresentados na notícia são incorrectos:</p><p>• o tempo, em horas e minutos, durante o qual o barco esteve encalhado;</p><p>• o valor da taxa de variação no instante em que o barco foi desencalhado;</p><p>• a diferença, em metros, entre os níveis das águas do mar registados, respectivamente, no</p><p>momento em que o barco foi desencalhado e no momento da baixa-mar imediatamente</p><p>anterior.</p><p>Na tabela seguinte, indica-se como deve ser classificado este item, de acordo com os níveis</p><p>de desempenho no domínio da comunicação escrita em língua portuguesa descritos nos</p><p>critérios gerais e com os níveis de desempenho no domínio específico da disciplina.</p><p>** Descritores apresentados nos critérios gerais.</p><p>** Apenas podem ser atribuídas classificações correspondentes a um dos valores constantes do</p><p>quadro. Não há lugar a classificações intermédias.</p><p>No caso de a resposta não atingir o nível 1 de desempenho no domínio específico da disciplina, a</p><p>classificação a atribuir é de zero pontos. Neste caso, não é classificado o desempenho no domínio</p><p>da comunicação escrita em língua portuguesa.</p><p>Nota: Em cada tópico considerado, não basta que o examinando indique na sua resposta que o valor</p><p>referido na notícia é incorrecto. Deverá justificar a sua resposta, apresentando o valor correcto</p><p>correspondente.</p><p>Descritores do nível de desempenho no domínio</p><p>da comunicação escrita em língua portuguesa</p><p>Descritores do nível de desempenho</p><p>no domínio específico da disciplina</p><p>Níveis*</p><p>1 2 3</p><p>N</p><p>ív</p><p>e</p><p>is</p><p>**</p><p>3 A composição aborda, correctamente, os três tópicos. 18 19 20</p><p>2 A composição aborda, correctamente, apenas dois tópicos. 12 13 14</p><p>1 A composição aborda, correctamente, apenas um tópico. 6 7 8</p><p>Prova 735 • Página C/12/ 12</p><p>Página | 1 de 5</p><p>Proposta de Resolução do Exame de Matemática B</p><p>Cod. 735 – 2ª Fase 2009</p><p>Grupo I</p><p>1. Sejam X1 e X2 as variáveis “Número saído na 1ª jogada” e “Número saído na 2ª jogada”, respectiva-</p><p>mente.</p><p>Tem-se P(X1=1) = P(X2=1) =</p><p>4</p><p>8 =</p><p>1</p><p>2</p><p>P(X1=2) = P(X2=2) =</p><p>2</p><p>8 =</p><p>1</p><p>4</p><p>P(X1=3) = P(X2=3) =</p><p>2</p><p>8 =</p><p>1</p><p>4</p><p>A tabela seguinte dá-nos os valores de X, para os valores de X1 e X2</p><p>P(X=2) = P(X1=1) × P(X2=1) =</p><p>1</p><p>2 ×</p><p>1</p><p>2 =</p><p>1</p><p>4</p><p>P(X=3) = P(X1=1) × P(X2=2) + P(X1=2) × P(X2=1)</p><p>=</p><p>1</p><p>2 ×</p><p>1</p><p>4 +</p><p>1</p><p>4 ×</p><p>1</p><p>2 =</p><p>1</p><p>4</p><p>P(X=4) = P(X1=1) × P(X2=3) + P(X1=2) × P(X2=2) + P(X1=3) × P(X2=1)</p><p>=</p><p>1</p><p>2 ×</p><p>1</p><p>4 +</p><p>1</p><p>4 ×</p><p>1</p><p>4 +</p><p>1</p><p>4 ×</p><p>1</p><p>2 =</p><p>5</p><p>16</p><p>P(X=5) = P(X1=2) × P(X2=3) + P(X1=3) × P(X2=2) =</p><p>1</p><p>4 ×</p><p>1</p><p>4 +</p><p>1</p><p>4 ×</p><p>1</p><p>4 =</p><p>1</p><p>8</p><p>P(X=6) = P(X1=3) × P(X2=3) =</p><p>1</p><p>4 ×</p><p>1</p><p>4 =</p><p>1</p><p>16</p><p>Então a tabela de distribuição de probabilidades de X é</p><p>X2</p><p>1 2 3</p><p>X1</p><p>1 2 3 4</p><p>2 3 4 5</p><p>3 4 5 6</p><p>Página | 2 de 5</p><p>xi 2 3 4 5 6</p><p>P(X=xi)</p><p>1</p><p>4 = 0,25</p><p>1</p><p>4 = 0,25</p><p>5</p><p>16 = 0,3125</p><p>1</p><p>8 = 0,125</p><p>1</p><p>16 = 0,0625</p><p>2. Média dos números saídos nas 820 jogadas é 1 × 0,55 + 2 × 0,20 + 3 × 0,25 = 1,7.</p><p>Grupo II</p><p>1. Dividindo o quadrado [ABCD] pelas suas medianas, obtemos 8 triângulos rec-</p><p>tângulos geometricamente iguais entre si. Seja A a área de cada um.</p><p>Então</p><p>Área do quadrado [ABCD] = 8 A</p><p>Área do quadrado [PQRS] = 4 A</p><p>Logo, Área do quadrado [PQRS] =</p><p>1</p><p>2 Área do quadrado [ABCD].</p><p>2.</p><p>Peça 1 2 3 4 5 6</p><p>Área da pedra</p><p>preciosa (cm2)</p><p>8 4 2 1 0,5 0,25</p><p>Como a área do exterior é 4 × 4 = 16 cm2 e no ponto 1 já foi verificado que o quadrado inscrito com</p><p>vértices nos pontos médios do quadrado exterior tem metade da área do quadrado exterior, então a</p><p>área da maior pedra preciosa é 8 cm2.</p><p>Soma das áreas das faces visíveis das pedras = 8 + 4 + 2 + 1 + 0,5 + 0,25 = 15,75</p><p>Somas das áreas 15,75 cm2.</p><p>Grupo III</p><p>1. Se o perímetro do mural é 26 m, a soma de dois lados consecutivos é 13 e portanto, as suas medi-</p><p>das, em metros, são</p><p>x e 13 − x</p><p>Para obter as medidas da tapeçaria basta subtrair os valores correspondentes às margens respecti-</p><p>vas:</p><p>x − 2×0,5 = x − 1 13 − x − 2×1 = 11 − x</p><p>A B</p><p>C D</p><p>P</p><p>Q</p><p>R</p><p>S</p><p>Página | 3 de 5</p><p>Como as medidas dos lados têm de ser números positivos, tem-se</p><p>x − 1 > 0 ∧ 11 − x > 0 ⇔ x > 1 ∧ x < 11 ⇔ x∈]1, 11[</p><p>2. Área da tapeçaria = base × altura</p><p>A (x) = (x − 1)(11 − x) = 11x − x2− 11 +</p>
<p>x = − x2</p><p>+ 12x − 11 x∈]1, 11[</p><p>3. A função A (x) é uma quadrática em que a<0, logo o gráfico é parte de uma</p><p>parábola com a concavidade voltada para baixo. O valor máximo é a ordena-</p><p>da do vértice.</p><p>Zeros: x = 1 ∨ x = 11</p><p>Abcissa do vértice: x =</p><p>1 + 11</p><p>2 = 6</p><p>Ordenada do vértice: A(6) = 25</p><p>A tapeçaria tem uma área máxima de 25 m2, no caso em que x = 6 metros.</p><p>Grupo IV</p><p>1.</p><p>1.1. O algarismo 8 tem 8% de ocorrências</p><p>8 − P (8) ≈ 4, 0765</p><p>Com a aproximação pedida, 8 − P (8) ≈ 4, 1</p><p>1.2. De acordo com o modelo encontrado, a percentagem de ocorrência do algarismo 1 é</p><p>�(1) = 26,6723 − 10,9399 ��(1) = 26,6723.</p><p>Se a distribuição tem por base uma amostra de 216 dados,</p><p>0,266723 � 216 ≈ 57,6 ou seja 58 números começam com o algarismo 1.</p><p>2.</p><p>Introduzindo a função na calculadora e pedindo uma tabela com valores arredondados às milé-</p><p>simas, obtém-se</p><p>25</p><p>6 x 1 11</p><p>A</p><p>Página | 4 de 5</p><p>n 1 2 3 4 5 6 7</p><p>P(n) 0,301 0,176 0,125 0,097 0,079 0,067 0,058</p><p>Trata-se do algarismo 7.</p><p>Grupo V</p><p>1. Representando a função M(t) e a recta y = 1,3 e determinando a intersecção dos dois gráficos,</p><p>considerando só o período até às 13 horas visto que se quer o período da manhã, tem-se:</p><p>Um dos pontos de intersecção corresponde t ≈ 5,6012, ou seja</p><p>5 horas e 36 minutos.</p><p>O outro ponto de intersecção corresponde t ≈ 9,3747, ou seja às 9 horas e 22 minutos.</p><p>Logo será entre cerca das 5h e 36 minutos e as 9 horas e 22 minutos que é possível apanhar o</p><p>marisco.</p><p>2.</p><p>O barco esteve encalhado desde as 6,11 horas, até às 23,53 horas, totalizando 23,53 −</p><p>6,11 = 17,42.</p><p>17,42 horas correspondem a 17 horas e 25,2 minutos, ou seja, passaram cerca de 17 horas e 25</p><p>minutos desde que o barco ficou encalhado até ser libertado, o que contraria a informação dada</p><p>pela comunicação do jornal local.</p><p>Página | 5 de 5</p><p>O barco foi desencalhado no instante � = 23,53, altura em que o</p><p>nível das águas do mar subia a uma taxa aproximada de 0,51</p><p>metros por hora, como se pode identificar na calculadora. Na</p><p>notícia, dava-se a informação errada de que a taxa seria de 0,7</p><p>metros por hora.</p><p>Na última baixa-mar, antes da libertação da embarcação, o nível</p><p>das águas do mar era de 0,853 metros.</p><p>Às 23,53 horas o nível era de 2,199 metros, o que nos permite</p><p>concluir que a maré já tinha subido 1,346 metros (2,199 –</p><p>1,346),</p><p>um valor inferior ao comunicado pelo jornal local.</p><p>2010</p><p>Prova 735 • Página 1/ 11</p><p>EXAME NACIONAL DO ENSINO SECUNDÁRIO</p><p>Decreto-Lei n.º 74/2004, de 26 de Março</p><p>Prova Escrita de Matemática B</p><p>10.º e 11.º Anos de Escolaridade</p><p>Prova 735/1.ª Fase 11 Páginas</p><p>Duração da Prova: 150 minutos. Tolerância: 30 minutos.</p><p>2010</p><p>Utilize apenas caneta ou esferográfica de tinta indelével, azul ou preta, excepto nas respostas</p><p>que impliquem a elaboração de construções, de desenhos ou de outras representações, que</p><p>podem ser, primeiramente, elaborados a lápis, sendo, a seguir, passados a tinta.</p><p>Utilize a régua, o compasso, o esquadro, o transferidor e a calculadora gráfica sempre que for</p><p>necessário.</p><p>Não é permitido o uso de corrector. Em caso de engano, deve riscar, de forma inequívoca,</p><p>aquilo que pretende que não seja classificado.</p><p>Escreva, de forma legível, a numeração dos grupos e dos itens, bem como as respectivas</p><p>respostas. As respostas ilegíveis ou que não possam ser identificadas são classificadas com</p><p>zero pontos.</p><p>Para cada item, apresente apenas uma resposta. Se escrever mais do que uma resposta a um</p><p>mesmo item, apenas é classificada a resposta apresentada em primeiro lugar.</p><p>Em todas as respostas, indique todos os cálculos que tiver de efectuar e todas as justificações necessárias.</p><p>Sempre que, na resolução de um problema, recorrer à calculadora, apresente todos os elementos recolhidos</p><p>na sua utilização. Mais precisamente:</p><p>• sempre que recorrer às capacidades gráficas da calculadora, apresente o(s) gráfico(s) obtido(s), bem</p><p>como as coordenadas de pontos relevantes para a resolução do problema proposto (por exemplo,</p><p>coordenadas de pontos de intersecção de gráficos, máximos, mínimos, etc.);</p><p>• sempre que recorrer a uma tabela obtida na calculadora, apresente todas as linhas da tabela relevantes</p><p>para a resolução do problema proposto;</p><p>• sempre que recorrer a estatísticas obtidas na calculadora (média, desvio padrão, coeficiente de</p><p>correlação, declive e ordenada na origem de uma recta de regressão, etc.), apresente a(s) lista(s) que</p><p>introduziu na calculadora para as obter.</p><p>A prova inclui, na página 11, um Formulário.</p><p>As cotações dos itens encontram-se no final do enunciado da prova.</p><p>Prova 735 • Página 2/ 11</p><p>Prova 735 • Página 3/ 11</p><p>GRUPO I</p><p>A gerência de um hotel de uma zona turística encomendou a um artista plástico um painel decorativo.</p><p>O painel será composto por uma sequência de dez telas quadradas, espaçadas entre si, todas com</p><p>12 decímetros de lado e com diferentes pinturas.</p><p>A Figura 1 representa as três primeiras telas dessa sequência, ordenadas da esquerda para a direita.</p><p>Figura 1</p><p>O artista pintou as telas de acordo com o seguinte processo:</p><p>• na primeira tela, pintou o círculo inscrito;</p><p>• dividiu a segunda tela em quatro quadrados geometricamente iguais, nos quais pintou os quatro</p><p>círculos inscritos, tal como se vê na figura;</p><p>• dividiu a terceira tela em nove quadrados geometricamente iguais, nos quais pintou os nove círculos</p><p>inscritos, tal como se vê na figura;</p><p>• e assim sucessivamente, até à décima tela.</p><p>1. Mostre que a área do círculo pintado na primeira tela é igual à soma das áreas dos círculos pintados na</p><p>segunda tela.</p><p>2. Determine o número de círculos pintados na décima tela do painel.</p><p>3. O artista vai decorar, com fio dourado, as dez telas que compõem o painel.</p><p>Em cada uma das dez telas, o artista aplicará o fio, percorrendo todos os lados dos quadrados nos quais</p><p>pintou os círculos inscritos. A aplicação será feita sem qualquer sobreposição do fio.</p><p>Quantos metros de fio dourado terá de aplicar o artista?</p><p>GRUPO II</p><p>Numa unidade de turismo de habitação, existem três reservatórios de água: um em forma de cilindro e</p><p>dois em forma de cone, todos com a mesma altura e com bases iguais.</p><p>Houve necessidade de despejar o reservatório cilíndrico, para se proceder a uma reparação.</p><p>A Figura 2 representa o reservatório cilíndrico, cheio de água.</p><p>Figura 2</p><p>Parte dessa água foi despejada nos dois reservatórios em forma de cone, que ficaram cheios.</p><p>Na Figura 3, estão representados esses reservatórios, cheios de água, e o reservatório cilíndrico, com a</p><p>água restante.</p><p>Figura 3</p><p>Considere que:</p><p>• a é a altura, em metros, de cada um dos reservatórios;</p><p>• r é o raio, em metros, da(s) base(s) de cada um dos reservatórios;</p><p>• x é a altura, em metros, da água que ficou no reservatório cilíndrico;</p><p>• a espessura do material de que são feitos os reservatórios é desprezável.</p><p>1. Mostre que a relação entre a altura da água que ficou no reservatório cilíndrico (x ) e a altura de cada</p><p>um dos reservatórios (a ) é dada por</p><p>a</p><p>x =</p><p>3</p><p>�</p><p>�</p><p>�</p><p>�</p><p>�</p><p>�</p><p>�</p><p>Prova 735 • Página 4/ 11</p><p>2. Posteriormente, a água que restava no reservatório cilíndrico, depois de se terem enchido os dois</p><p>reservatórios cónicos, foi sendo retirada, até aquele ficar vazio.</p><p>Admita que a altura h , em metros, da água que restava no reservatório cilíndrico, t horas após ter sido</p><p>começada a retirar até o reservatório ficar completamente vazio, é dada por:</p><p>2.1. Calcule a altura, em metros, do reservatório cilíndrico.</p><p>Na sua resolução, percorra, sucessivamente, as seguintes etapas:</p><p>• calcular h (0)</p><p>• relacionar h (0) com x</p><p>• utilizar a igualdade</p><p>• calcular a altura pedida.</p><p>2.2. Quanto tempo demorou o reservatório cilíndrico a esvaziar, a partir do momento em que se começou</p><p>a retirar a água que ficara nesse reservatório, depois de se terem enchido os dois reservatórios</p><p>cónicos?</p><p>Apresente o resultado em horas e minutos.</p>
<p>2.3. Relativamente à situação descrita, fez-se a seguinte afirmação:</p><p>«Durante o período de esvaziamento do reservatório cilíndrico, existe um certo intervalo de tempo,</p><p>no qual a taxa de variação média da função h tem um valor positivo.»</p><p>Esta afirmação é verdadeira? Justifique a sua resposta.</p><p>a</p><p>x =</p><p>3</p><p>–</p><p>( )</p><p>–</p><p>t</p><p>h t</p><p>t</p><p>5 16</p><p>=</p><p>10</p><p>Prova 735 • Página 5/ 11</p><p>GRUPO III</p><p>O Diogo é estudante de um curso de Gestão Hoteleira. Para uma das disciplinas do curso, realizou dois</p><p>trabalhos. O primeiro trabalho consistiu na elaboração de um programa turístico. O segundo trabalho incluiu</p><p>a análise de vários indicadores sociológicos relativos a alguns países da União Europeia.</p><p>1. O programa turístico elaborado pelo Diogo incluía uma visita a Lisboa e um passeio de barco no rio Tejo.</p><p>A fim de aproveitar o tempo disponível para as diversas actividades, o Diogo teve em conta que a duração</p><p>da exposição solar varia ao longo do ano.</p><p>Com base em dados do Observatório Astronómico de Lisboa, obteve os modelos que dão,</p><p>aproximadamente, a hora a que o Sol nasceu, N, e a hora a que o Sol se pôs, P, em Lisboa, em cada</p><p>dia do ano de 2009:</p><p>Considere que:</p><p>• x representa a ordem do dia do ano, sendo</p><p>• o argumento da função seno está em radianos</p><p>• a duração da exposição solar é dada por P(x ) – N(x )</p><p>1.1. Determine a duração da exposição solar no último dia do ano de 2009, em Lisboa, de acordo com os</p><p>modelos apresentados.</p><p>Apresente o resultado em horas e minutos, com os minutos arredondados às unidades.</p><p>Em cálculos intermédios, se proceder a arredondamentos, mantenha, pelo menos, três casas</p><p>decimais.</p><p>1.2. Determine o número de dias do ano de 2009 nos quais, de acordo com os modelos apresentados, a</p><p>duração da exposição solar, em Lisboa, foi superior a 10 horas.</p><p>Em cálculos intermédios, se proceder a arredondamentos, mantenha, pelo menos, uma casa</p><p>decimal.</p><p>1.3. Qual foi, em 2009, a ordem do dia do ano com a maior duração de exposição solar, em Lisboa?</p><p>Resolva o problema, recorrendo às capacidades gráficas da sua calculadora.</p><p>Em cálculos intermédios, se proceder a arredondamentos, mantenha, pelo menos, quatro casas</p><p>decimais.</p><p>x ∈ { , , ..., }1 2 365</p><p>N x x</p><p>P x</p><p>( ) , , ( , , )</p><p>( ) , ,</p><p>= + +</p><p>= +</p><p>6 5987 1 3424 0 0161 1 8287</p><p>18 6745 1 3875</p><p>sen</p><p>senn( , , )0 0164 11955x −</p><p>Prova 735 • Página 6/ 11</p><p>2. Para a elaboração do segundo trabalho, o Diogo recolheu diversos dados estatísticos e procedeu à</p><p>realização de um inquérito a turistas que visitaram Portugal.</p><p>2.1. O Diogo consultou os registos referentes à esperança média de vida à nascença, para homens e</p><p>mulheres de países da União Europeia.</p><p>Organizou esses registos numa tabela, que se apresenta a seguir, na qual x designa o número</p><p>médio de anos de vida esperados à nascença para as mulheres e y designa o número médio de</p><p>anos de vida esperados à nascença para os homens.</p><p>Esperança média de vida à nascença para homens e mulheres</p><p>Fontes: INE e Eurostat</p><p>O Diogo não registou na tabela os valores referentes a alguns países da União Europeia, como, por</p><p>exemplo, os referentes à Áustria.</p><p>Admita que os valores da esperança média de vida à nascença para homens e mulheres referentes</p><p>à Áustria seguem o modelo de regressão linear obtido a partir dos dados da tabela.</p><p>Estime o valor da esperança média de vida à nascença de um homem austríaco, sabendo que a</p><p>esperança média de vida à nascença de uma mulher austríaca é 83,0 anos.</p><p>Apresente os valores dos parâmetros da equação da recta de regressão linear de y sobre x com,</p><p>pelo menos, seis casas decimais.</p><p>Apresente o resultado final arredondado às décimas.</p><p>PAÍSES</p><p>MULHERES</p><p>(x )</p><p>HOMENS</p><p>(y )</p><p>Portugal 81,7 75,5</p><p>Espanha 85,0 78,9</p><p>França 84,3 77,5</p><p>Irlanda 81,6 76,8</p><p>Reino Unido 81,7 77,6</p><p>Bélgica 83,5 77,5</p><p>Holanda 82,3 78,3</p><p>Alemanha 82,4 77,2</p><p>Itália 84,1 78,8</p><p>Grécia 82,5 77,5</p><p>Prova 735 • Página 7/ 11</p><p>2.2. O Diogo recolheu, através do inquérito que realizou, informação sobre alguns indicadores</p><p>socioeconómicos de turistas que visitaram Portugal.</p><p>A partir da informação obtida, concluiu que, no grupo de turistas que responderam ao inquérito, o</p><p>valor do vencimento mensal individual auferido, em euros, seguia, aproximadamente, a distribuição</p><p>normal, N(2400, 300), de média µ = 2400 e desvio padrão σ = 300</p><p>Admita que se selecciona, ao acaso, um elemento do referido grupo de turistas.</p><p>Será mais provável que o valor do seu vencimento mensal individual seja superior a 2900 euros ou</p><p>que seja inferior a 2000 euros ? Justifique.</p><p>Se recorrer à sua calculadora, apresente cada valor obtido arredondado às centésimas.</p><p>Prova 735 • Página 8/ 11</p><p>GRUPO IV</p><p>Uma equipa de biólogos estuda, desde o início do ano de 2000, a evolução do número de elementos das</p><p>populações de milhafres e de esquilos existentes numa Reserva Natural, onde se desenvolvem actividades</p><p>de ecoturismo.</p><p>De acordo com os estudos efectuados, sabe-se que:</p><p>• o número de milhafres tem vindo a aumentar ao longo do tempo, embora lentamente, registando-se, na</p><p>actualidade, um número ligeiramente inferior às cinco centenas de efectivos;</p><p>• o número de esquilos tem vindo a diminuir significativamente ao longo do tempo, registando-se,</p><p>actualmente, um número de elementos inferior a meia centena de efectivos, o que coloca em risco a</p><p>sobrevivência desta espécie na Reserva.</p><p>Sejam M e E as funções que dão, respectivamente, o número aproximado, em centenas, de milhafres</p><p>e de esquilos, x meses após o início do ano de 2000, sendo x ≥�0</p><p>Em cada uma das quatro opções seguintes, apresenta-se uma expressão para definir a função M e</p><p>outra para definir a função E , sendo x ≥�0</p><p>Apenas a opção D) está correcta, de acordo com os estudos efectuados.</p><p>Numa pequena composição, apresente, para cada uma das outras três opções, uma razão que justifique</p><p>a sua rejeição, com base em propriedades das funções definidas pelas expressões apresentadas em cada</p><p>uma dessas opções.</p><p>Nota – Poderá ser-lhe útil recorrer às capacidades gráficas da sua calculadora. Se o fizer, deve reproduzir, na sua folha</p><p>de prova, o(s) gráfico(s) obtido(s).</p><p>FIM</p><p>0,05</p><p>0,05</p><p>ln( 2)</p><p>( ) 0,8 7 1,03 ; ( )</p><p>ln(2,5)</p><p>5</p><p>( ) ; ( ) 0,8 7 1,03</p><p>1 6</p><p>ln( 2)</p><p>( ) ; ( ) 9 1,03</p><p>ln(2,5)</p><p>5</p><p>( ) ; ( ) 9 1,03</p><p>1 6</p><p>x</p><p>x</p><p>x</p><p>x</p><p>x</p><p>x</p><p>x</p><p>M x E x</p><p>M x E x</p><p>e</p><p>x</p><p>M x E x</p><p>M x E x</p><p>e</p><p>+</p><p>= + × =</p><p>= = + ×</p><p>+ ×</p><p>+</p><p>= = ×</p><p>= = ×</p><p>+ ×</p><p>A)</p><p>B)</p><p>C)</p><p>D)</p><p>−</p><p>−</p><p>−</p><p>−</p><p>−</p><p>−</p><p>Prova 735 • Página 9/ 11</p><p>COTAÇÕES</p><p>GRUPO I</p><p>1. ........................................................................................................................... 15 pontos</p><p>2. ........................................................................................................................... 10 pontos</p><p>3. .......................................................................................................................... 15 pontos</p><p>40 pontos</p><p>GRUPO II</p><p>1. .............................................................................................................................. 15 pontos</p><p>2.</p><p>2.1. ..................................................................................................................... 15 pontos</p><p>2.2. ...................................................................................................................... 15 pontos</p><p>2.3. ..................................................................................................................... 10 pontos</p><p>55 pontos</p><p>GRUPO III</p><p>1.</p><p>1.1. ...................................................................................................................... 15 pontos</p><p>1.2. ...................................................................................................................... 20 pontos</p><p>1.3. ...................................................................................................................... 20 pontos</p><p>2.</p><p>2.1. ...................................................................................................................... 15 pontos</p><p>2.2. ......................................................................................................................</p>
<p>15 pontos</p><p>85 pontos</p><p>GRUPO IV</p><p>.................................................................................................................................. 20 pontos</p><p>20 pontos</p><p>_______________</p><p>TOTAL .............................................................. 200 pontos</p><p>Prova 735 • Página 10/ 11</p><p>Prova 735 • Página 11/ 11</p><p>Comprimento de um arco</p><p>de circunferência</p><p>α r (α – amplitude, em radianos,</p><p>do ângulo ao centro; r – raio)</p><p>Áreas de figuras planas</p><p>Losango:</p><p>Trapézio:</p><p>× Altura</p><p>Polígono regular:</p><p>Semiperímetro × Apótema</p><p>Sector circular:</p><p>(α – amplitude, em radianos, do</p><p>ângulo ao centro; r – raio)</p><p>Áreas de superfícies</p><p>Área lateral de um cone:</p><p>π rg (r – raio da base; g – geratriz)</p><p>Área de uma superfície esférica:</p><p>4 π r2 (r – raio)</p><p>Área lateral de um cilindro recto:</p><p>2 π rg (r – raio da base; g – geratriz)</p><p>Volumes</p><p>Pirâmide: × Área da base × Altura</p><p>Cone: × Área da base × Altura</p><p>Esfera: π r3 (r – raio)</p><p>Cilindro: Área da base × Altura</p><p>Progressões</p><p>Soma dos n primeiros termos de uma</p><p>Progressão aritmética: × n</p><p>Progressão geométrica: u1 ×</p><p>Probabilidades e Estatística</p><p>Se X é uma variável aleatória discreta, de</p><p>valores x i com probabilidades pi, então:</p><p>• média de X:</p><p>•</p><p>• desvio padrão de X:</p><p>•</p><p>Se X é uma variável aleatória normal, de média µ e</p><p>desvio padrão σ, então:</p><p>•</p><p>( ) ,</p><p>( ) ,</p><p>( ) , 3</p><p>P X</p><p>P X</p><p>P X</p><p>µ σ µ σ</p><p>µ σ µ σ</p><p>µ σ µ σ</p><p>− < < + ≈ 0 6827</p><p>−2 < < + 2 ≈ 0 9545</p><p>−3 < < + 3 ≈ 0 997</p><p>1( ) ... ( )n np x p xσ µ µ2 2</p><p>1= − + + −</p><p>... n np x p xµ 1 1= + +</p><p>1 – rn</p><p>———–</p><p>1 – r</p><p>u1 + un———–</p><p>2</p><p>4—</p><p>3</p><p>1—</p><p>3</p><p>1—</p><p>3</p><p>α r2</p><p>——</p><p>2</p><p>Base maior + Base menor</p><p>———————————</p><p>2</p><p>Diagonal maior × Diagonal menor</p><p>———————————————</p><p>2</p><p>Formulário</p><p>EXAME NACIONAL DO ENSINO SECUNDÁRIO</p><p>Decreto-Lei n.º 74/2004, de 26 de Março</p><p>Prova Escrita de Matemática B</p><p>10.º e 11.º Anos de Escolaridade</p><p>Prova 735/1.ª Fase 15 Páginas</p><p>Duração da Prova: 150 minutos. Tolerância: 30 minutos.</p><p>2010</p><p>COTAÇÕES</p><p>GRUPO I</p><p>1. ........................................................................................................... 15 pontos</p><p>2. ........................................................................................................... 10 pontos</p><p>3. ........................................................................................................... 15 pontos</p><p>40 pontos</p><p>GRUPO II</p><p>1. ............................................................................................................ 15 pontos</p><p>2.</p><p>2.1. ................................................................................................... 15 pontos</p><p>2.2. ................................................................................................... 15 pontos</p><p>2.3. ................................................................................................... 10 pontos</p><p>55 pontos</p><p>GRUPO III</p><p>1.</p><p>1.1. ................................................................................................... 15 pontos</p><p>1.2. ................................................................................................... 20 pontos</p><p>1.3. ................................................................................................... 20 pontos</p><p>2.</p><p>2.1. ................................................................................................... 15 pontos</p><p>2.2. ................................................................................................... 15 pontos</p><p>85 pontos</p><p>GRUPO IV</p><p>............................................................................................................... 20 pontos</p><p>20 pontos</p><p>___________</p><p>TOTAL ..................................... 200 pontos</p><p>Prova 735 • Página C/1/ 15</p><p>CRITÉRIOS GERAIS DE CLASSIFICAÇÃO</p><p>A classificação a atribuir a cada resposta resulta da aplicação dos critérios gerais e dos critérios específicos</p><p>de classificação apresentados para cada item e é expressa por um número inteiro, previsto na grelha de</p><p>classificação.</p><p>As respostas ilegíveis ou que não possam ser claramente identificadas são classificadas com zero pontos.</p><p>No entanto, em caso de omissão ou de engano na identificação de uma resposta, esta pode ser classificada</p><p>se for possível identificar, inequivocamente, o item a que diz respeito.</p><p>Se o examinando responder a um mesmo item mais do que uma vez, não eliminando, inequivocamente, a(s)</p><p>resposta(s) que não deseja que seja(m) classificada(s), deve ser considerada apenas a resposta que surgir</p><p>em primeiro lugar.</p><p>Os itens apresentam critérios específicos de classificação organizados por etapas e/ou por níveis de</p><p>desempenho. A cada etapa e a cada nível de desempenho corresponde uma dada pontuação. Nos itens que</p><p>apresentam critérios específicos de classificação organizados por níveis de desempenho, no caso de,</p><p>ponderados todos os dados contidos nos descritores, permanecerem dúvidas quanto ao nível a atribuir, deve</p><p>optar-se pelo nível mais elevado de entre os dois tidos em consideração.</p><p>As respostas que apresentem pontos de vista diferentes dos mencionados nos critérios específicos de</p><p>classificação, devem ser classificadas se o seu conteúdo for considerado cientificamente válido e estiver</p><p>adequado ao solicitado. Nestes casos, os elementos cientificamente válidos devem ser classificados,</p><p>seguindo procedimentos análogos aos previstos nas etapas e/ou nos descritores apresentados.</p><p>Nos itens de resposta aberta com cotação igual ou superior a 20 pontos que impliquem a produção de um texto,</p><p>a classificação a atribuir traduz a avaliação simultânea das competências específicas da disciplina e das</p><p>competências de comunicação em língua portuguesa.</p><p>A avaliação das competências de comunicação escrita em língua portuguesa contribui para valorizar a</p><p>classificação atribuída ao desempenho no domínio das competências específicas da disciplina. Esta</p><p>valorização é cerca de 10% da cotação do item e faz-se de acordo com os níveis de desempenho descritos no</p><p>quadro seguinte.</p><p>No caso de a resposta não atingir o nível 1 de desempenho no domínio específico da disciplina, a</p><p>classificação a atribuir é zero pontos.</p><p>Nível Descritor</p><p>3</p><p>Composição bem estruturada, sem erros de sintaxe, de pontuação e/ou de ortografia, ou</p><p>com erros esporádicos, cuja gravidade não implique perda de inteligibilidade e/ou de</p><p>sentido.</p><p>2</p><p>Composição razoavelmente estruturada, com alguns erros de sintaxe, de pontuação e/ou</p><p>ortografia, cuja gravidade não implique perda de inteligibilidade e/ou de sentido.</p><p>1</p><p>Composição sem estruturação aparente, com erros graves de sintaxe, pontuação e/ou de</p><p>ortografia, cuja gravidade implique perda frequente de inteligibilidade e/ou de sentido.</p><p>A classificação da prova deve respeitar integralmente</p><p>os critérios gerais e os critérios específicos a seguir apresentados.</p><p>Prova 735 • Página C/2/ 15</p><p>Neste caso, não é classificado o desempenho no domínio da comunicação escrita em língua portuguesa.</p><p>Havendo escolas em que os alunos já contactam com as novas regras ortográficas, uma vez que o Acordo</p><p>Ortográfico de 1990 já foi ratificado e dado que qualquer cidadão, nesta fase de transição, pode optar pela</p><p>ortografia prevista quer no Acordo de 1945, quer no de 1990, são consideradas correctas, na classificação</p><p>das provas de exame nacional, as grafias que seguirem o que se encontra previsto em qualquer um destes</p><p>normativos.</p><p>No quadro seguinte, apresentam-se os critérios de classificação a aplicar em situações não descritas</p><p>anteriormente.</p><p>Situação Classificação</p><p>1. Classificação de um item cujo critério se apresenta</p><p>organizado por etapas.</p><p>A cotação indicada para cada etapa é a classificação</p><p>máxima que lhe é atribuível.</p><p>A classificação da resposta resulta da soma das</p><p>pontuações das diferentes etapas, à qual, eventualmente,</p><p>se subtraem um ou dois pontos, de acordo com o previsto</p><p>nas situações 12 e/ou 16.</p><p>2. Pontuação de uma etapa dividida em passos. A cotação indicada para cada passo é a classificação</p><p>máxima que lhe é atribuível.</p><p>A pontuação da etapa resulta da soma das classificações</p><p>dos diferentes passos.</p><p>3. Classificação</p>
<p>de um item ou pontuação de uma etapa</p><p>cujo critério se apresenta organizado por níveis de</p><p>desempenho.</p><p>A resposta é enquadrada numa das descrições apresen-</p><p>tadas.</p><p>À classificação/pontuação correspondente subtrai-se,</p><p>eventualmente, um ponto, de acordo com o previsto nas</p><p>situações 7, 8 e/ou 16.</p><p>4. Utilização de processos de resolução que não estão</p><p>previstos no critério específico de classificação.</p><p>É aceite e classificado qualquer processo de resolução</p><p>cientificamente correcto.</p><p>O critério específico deve ser adaptado ao processo de</p><p>resolução apresentado, mediante distribuição da cotação</p><p>do item pelas etapas* percorridas pelo examinando. Esta</p><p>adaptação do critério deve ser utilizada em todos os</p><p>processos de resolução análogos.</p><p>5. Apresentação apenas do resultado final, embora a</p><p>resolução do item exija cálculos e/ou justificações.</p><p>Deve ser atribuída a classificação de zero pontos.</p><p>6. Ausência de apresentação explícita de uma dada etapa. Se a resolução apresentada permite perceber, inequivoca-</p><p>mente, que a etapa foi percorrida, a mesma é pontuada</p><p>com a cotação total para ela prevista.</p><p>7. Transposição incorrecta de dados do enunciado. Se o grau de dificuldade da resolução não diminuir, é</p><p>subtraído um ponto à pontuação da etapa.</p><p>Se o grau de dificuldade da resolução da etapa diminuir, a</p><p>pontuação máxima a atribuir a essa etapa deve ser a parte</p><p>inteira da metade da cotação prevista.</p><p>8. Ocorrência de um erro ocasional num cálculo. É subtraído um ponto à pontuação da etapa em que o erro</p><p>ocorre.</p><p>9. Ocorrência de um erro que revela desconhecimento de</p><p>conceitos, de regras ou de propriedades.</p><p>A pontuação máxima a atribuir nessa etapa deve ser a</p><p>parte inteira de metade da cotação prevista.</p><p>Prova 735 • Página C/3/ 15</p><p>* Em situações em que o critério é aplicável tanto a etapas como a passos, utiliza-se apenas o termo «etapas», por razões de</p><p>simplificação da apresentação.</p><p>Situação Classificação</p><p>10. Ocorrência de um erro na resolução de uma etapa. A etapa é pontuada de acordo com o erro cometido.</p><p>As etapas subsequentes são pontuadas de acordo com os</p><p>efeitos do erro cometido:</p><p>– se o grau de dificuldade das etapas subsequentes não</p><p>diminuir, estas são pontuadas de acordo com os critérios</p><p>específicos de classificação;</p><p>– se o grau de dificuldade das etapas subsequentes</p><p>diminuir, a pontuação máxima a atribuir a cada uma delas</p><p>deve ser a parte inteira de metade da cotação prevista.</p><p>11. Resolução incompleta de uma etapa. Se à resolução da etapa faltar apenas o passo final, é</p><p>subtraído um ponto à pontuação da etapa; caso contrário a</p><p>pontuação máxima a atribuir deve ser a parte inteira de</p><p>metade da cotação prevista.</p><p>12. Apresentação de cálculos intermédios com um número</p><p>de casas decimais diferente do solicitado e/ou</p><p>apresentação de um arredondamento incorrecto.</p><p>É subtraído um ponto à classificação total da resposta.</p><p>13. A apresentação do resultado final não respeita a forma</p><p>solicitada. [Exemplo: é pedido o resultado em centíme-</p><p>tros, e o examinando apresenta-o em metros.]</p><p>É subtraído um ponto à pontuação da etapa correspondente</p><p>à apresentação do resultado final.</p><p>14. Omissão da unidade de medida na apresentação do</p><p>resultado final [por exemplo, «15» em vez de «15</p><p>metros»].</p><p>A etapa relativa à apresentação do resultado final é</p><p>pontuada com a cotação total para ela prevista.</p><p>15. Apresentação do resultado final com um número de</p><p>casas decimais diferente do solicitado, e/ou</p><p>apresentação do resultado final incorrectamente</p><p>arredondado.</p><p>É subtraído um ponto à pontuação da etapa correspondente</p><p>à apresentação do resultado final.</p><p>16. Utilização de simbologias ou de expressões inequivo-</p><p>camente incorrectas do ponto de vista formal.</p><p>É subtraído um ponto à classificação total da resposta,</p><p>excepto:</p><p>– se as incorrecções ocorrerem apenas em etapas já</p><p>pontuadas com zero pontos;</p><p>– nos casos de uso do símbolo de igualdade quando, em</p><p>rigor, deveria ter sido usado o símbolo de igualdade</p><p>aproximada.</p><p>Prova 735 • Página C/4/ 15</p><p>CRITÉRIOS ESPECÍFICOS DE CLASSIFICAÇÃO</p><p>GRUPO I</p><p>1. .................................................................................................................................................... 15 pontos</p><p>Este item pode ser resolvido por, pelo menos, dois processos.</p><p>1.º Processo:</p><p>Determinar a área do círculo pintado na primeira tela (ver nota).................................... 6 pontos</p><p>Indicar o raio do círculo (6 dm) ..................................................... 3 pontos</p><p>Calcular a área do círculo (36π dm2) ........................................ 3 pontos</p><p>Determinar a soma das áreas dos círculos pintados na segunda tela (ver nota) ........... 8 pontos</p><p>Indicar o raio de um dos círculos (3 dm) ...................................... 3 pontos</p><p>Calcular a área de um dos círculos (9π dm2) ............................ 3 pontos</p><p>Calcular a soma das áreas dos quatro círculos (36π dm2) ........ 2 pontos</p><p>Concluir que a soma das áreas dos círculos da segunda tela é igual à área do</p><p>círculo da primeira tela ............................................................................................. 1 pontos</p><p>Nota – O examinando pode apresentar um resultado aproximado das áreas. Se os valores não forem</p><p>apresentados com a mesma aproximação, a classificação total da resposta deverá ser</p><p>desvalorizada em 2 pontos.</p><p>2.º Processo:</p><p>Referir que cada círculo da segunda tela é semelhante ao círculo da primeira</p><p>tela, sendo a razão de semelhança .......................................................................... 5 pontos</p><p>Referir que a razão entre a área de cada círculo da segunda tela e a área</p><p>do círculo da primeira tela é ..................................................................................... 5 pontos</p><p>Concluir que a soma das áreas dos círculos da segunda tela é igual à área do</p><p>círculo da primeira tela ............................................................................................. 5 pontos</p><p>2. .................................................................................................................................................... 10 pontos</p><p>Referir que os lados da décima tela estão divididos em dez partes iguais .............. 5 pontos</p><p>Calcular o número de círculos pintados (100) ........................................................ 5 pontos</p><p>1</p><p>4</p><p>1</p><p>2</p><p>Prova 735 • Página C/5/ 15</p><p>3. .................................................................................................................................................... 15 pontos</p><p>Este item pode ser resolvido por, pelo menos, quatro processos.</p><p>1.º Processo:</p><p>Determinar o 1.º termo da progressão aritmética (an), em que an é o comprimento</p><p>do fio, em dm, aplicado na n-ésima tela, para n ≤ 10 (48) .............................. 1 pontos</p><p>Indicar a razão da progressão aritmética (24) ....................................................... 4 pontos</p><p>Determinar o 10.º termo da progressão aritmética (264) ....................................... 5 pontos</p><p>Determinar a soma dos dez primeiros termos da progressão aritmética (1560) ... 4 pontos</p><p>Apresentar o resultado (156 m) ............................................................................ 1 pontos</p><p>2.º Processo:</p><p>Determinar os dez primeiros termos da progressão aritmética (an), em que an é o</p><p>comprimento do fio, em dm, aplicado na n-ésima tela, para n ≤ 10 (ver nota) .. 10 pontos</p><p>Calcular a soma desses termos (1560) ................................................................. 4 pontos</p><p>Apresentar o resultado (156 m) ............................................................................ 1 pontos</p><p>3.º Processo:</p><p>Determinar o primeiro termo da progressão aritmética (bn), em que bn é o</p><p>número de segmentos, de comprimento igual ao lado da tela, a serem decorados</p><p>com fio dourado, na n-ésima tela, para n ≤ 10 (4)............................................. 1 pontos</p><p>Indicar a razão da progressão aritmética (2) .........................................................</p>
<p>3 pontos</p><p>Assinalar os pontos do gráfico de abcissa 1 e de</p><p>abcissa 365 ........................................................... 1 ponto</p><p>Escrever, no eixo das abcissas, os valores 1 e 365 .. 1 ponto</p><p>Apresentar o gráfico apenas em [1; 365].................. 1 ponto</p><p>P N−� �</p><p>�P x x( ) , , ( , – , )= +18 6745 1 3875 0 0164 11955sen</p><p>�N x x( ) , , ( , , )= + +6 5987 1 3424 0 0161 1 8287sen</p><p>Prova 735 • Página C/9/ 15</p><p>Obter f (365) (9,365) ........................................................................................ 4 pontos</p><p>Apresentar o valor pedido (9 horas e 22 minutos) ......................................... 3 pontos</p><p>1.2. ................................................................................................................................................. 20 pontos</p><p>Este item pode ser resolvido por, pelo menos, três processos.</p><p>1.º Processo:</p><p>Usar a função f = (ver nota prévia) ..................................................... 2 pontos</p><p>Representar o gráfico da função f ........................................................................ 4 pontos</p><p>Respeitar a forma sinusoidal ....................................................... 1 pontos</p><p>Respeitar o domínio .................................................................... 3 pontos</p><p>Assinalar os pontos do gráfico de abcissa 1 e de</p><p>abcissa 365 ........................................................... 1 ponto</p><p>Escrever, no eixo das abcissas, os valores 1 e 365 .. 1 ponto</p><p>Apresentar o gráfico apenas em [1; 365].................. 1 ponto</p><p>Representar graficamente a recta de equação y = 10 ...................................... 3 pontos</p><p>Determinar as abcissas dos pontos de intersecção dos gráficos (ver nota 1) ..... 4 pontos</p><p>Interpretar os valores das abcissas dos pontos de intersecção dos gráficos, de</p><p>acordo com a situação (24 e 323) (ver notas 2 e 3) ............................................ 4 pontos</p><p>Apresentar a resposta (300 dias) (ver nota 4) ................................................... 3 pontos</p><p>2.º Processo:</p><p>Usar a função g = + 10 (ver nota prévia) ............................................... 2 pontos</p><p>Representar, no mesmo referencial, os gráficos das funções e g (ver nota</p><p>prévia) .................................................................................................................. 7 pontos</p><p>Respeitar a forma sinusoidal dos gráficos de e de g ..(1+1).. 2 pontos</p><p>Respeitar o domínio de e de g ............................................. 5 pontos</p><p>Assinalar os pontos dos gráficos de e de g</p><p>de abcissa 1 e de abcissa 365 ......... (1 + 1)........ 2 pontos</p><p>Escrever, no eixo das abcissas, os valores 1 e 365 .. 1 pontos</p><p>Apresentar os gráficos de e de g apenas em</p><p>[1; 365] ............................(1 + 1) ........................... 2 pontos</p><p>Determinar as abcissas dos pontos de intersecção dos gráficos (ver nota 1) ..... 4 pontos</p><p>Interpretar os valores das abcissas dos pontos de intersecção dos gráficos, de</p><p>acordo com a situação (24 e 323) (ver notas 2 e 3) ............................................ 4 pontos</p><p>Apresentar a resposta (300 dias) (ver nota 4) .................................................. 3 pontos</p><p>P�</p><p>P�</p><p>P�</p><p>P�</p><p>P�</p><p>N�</p><p>P N−� �</p><p>Prova 735 • Página C/10/ 15</p><p>3.º Processo:</p><p>Usar a função h = (ver nota prévia) ................................................. 2 pontos</p><p>Representar, no mesmo referencial, os gráficos das funções Ñ e h (ver nota</p><p>prévia) .................................................................................................................. 7 pontos</p><p>Respeitar a forma sinusoidal dos gráficos de Ñ e de h ..(1+1).. 2 pontos</p><p>Respeitar o domínio de Ñ e de h ............................................... 5 pontos</p><p>Assinalar os pontos dos gráficos de Ñ e de h</p><p>de abcissa 1 e de abcissa 365 .........(1 + 1)......... 2 pontos</p><p>Escrever, no eixo das abcissas, os valores 1 e 365 .. 1 pontos</p><p>Apresentar os gráficos de Ñ e de h apenas em</p><p>[1; 365] ........................... (1 + 1) ........................... 2 pontos</p><p>Determinar as abcissas dos pontos de intersecção dos gráficos (ver nota 1) ..... 4 pontos</p><p>Interpretar os valores das abcissas dos pontos de intersecção dos gráficos, de</p><p>acordo com a situação (24 e 323) (ver notas 2 e 3) ............................................ 4 pontos</p><p>Apresentar a resposta (300 dias) (ver nota 4) .................................................. 3 pontos</p><p>Notas:</p><p>1. Se o examinando assinalar apenas os pontos de intersecção dos gráficos, a pontuação a atribuir</p><p>nesta etapa deverá ser 2 pontos.</p><p>2. Se o examinando apresentar o valor 23 dias, em vez de 24, a pontuação a atribuir nesta etapa</p><p>deverá ser desvalorizada em 1 ponto. Se o examinando apresentar o valor 324 dias, em vez de</p><p>323, a pontuação a atribuir nesta etapa deverá ser desvalorizada em 1 ponto.</p><p>3. Se o examinando não interpretar os valores das abcissas dos pontos de intersecção dos gráficos</p><p>mas efectuar correctamente a diferença entre os valores obtidos, a pontuação a atribuir nesta</p><p>etapa não deverá ser desvalorizada.</p><p>4. Se o examinando apresentar como resposta a diferença entre os valores determinados na etapa</p><p>anterior, não lhe adicionando uma unidade, a pontuação a atribuir nesta etapa deverá ser</p><p>2 pontos.</p><p>P −10�</p><p>Prova 735 • Página C/11/ 15</p><p>1.3. ................................................................................................................................................. 20 pontos</p><p>Usar a função f = (ver nota prévia) .................................................... 2 pontos</p><p>Representar o gráfico da função f ...................................................................... 4 pontos</p><p>Respeitar a forma sinusoidal ....................................................... 1 pontos</p><p>Respeitar o domínio .................................................................... 3 pontos</p><p>Assinalar os pontos do gráfico de abcissa 1 e de</p><p>abcissa 365 ........................................................... 1 ponto</p><p>Escrever, no eixo das abcissas, os valores 1 e 365 .. 1 ponto</p><p>Apresentar o gráfico apenas em [1; 365].................. 1 ponto</p><p>Assinalar o ponto do gráfico correspondente ao máximo de f ................................ 3 pontos</p><p>Determinar a abcissa desse ponto ........................................................................ 4 pontos</p><p>Mostrar que f (174) > f (173) ........................................................................... 4 pontos</p><p>Apresentar a resposta (174) ............................................................................... 3 pontos</p><p>2.1. ................................................................................................................................................. 15 pontos</p><p>Indicar o valor do declive (0,544378) ................................................................. 4 pontos</p><p>Indicar o valor da ordenada na origem (32,425635) ......................................... 4 pontos</p><p>Obter o valor pedido ............................................................................................. 7 pontos</p><p>O valor pedido pode ser obtido por, pelo menos, dois processos.</p><p>1.º Processo:</p><p>Escrever 0,544378 × 83,0 + 32,425635 ............................. 4 pontos</p><p>Apresentar a resposta (77,6) ..................................................... 3 pontos</p><p>2.º Processo:</p><p>Representar graficamente a recta de regressão linear ............... 2 pontos</p><p>Assinalar o ponto da recta de abcissa 83,0 ............................... 2 pontos</p><p>Apresentar a resposta (77,6) .................................................... 3 pontos</p><p>2.2. ................................................................................................................................................. 15 pontos</p><p>Este item pode ser resolvido por, pelo menos, quatro processos.</p><p>Nos processos apresentados, é utilizada a variável aleatória X para designar o «valor, em</p><p>euros, do vencimento mensal individual auferido pelo turista</p>
<p>na utilização da calculadora. Esta</p><p>apresentação deve ser cotada de acordo com o critério que se segue, no qual, para</p><p>cada nível de desempenho, é indicada uma percentagem. Esta percentagem deve ser</p><p>aplicada sobre a cotação prevista para a explicação do método utilizado, e o valor</p><p>obtido deve ser arredondado às unidades (por excesso, se a mantissa do número a</p><p>arredondar for 0,5 ou superior).</p><p>Apresentação correcta e completa de todos os elementos relevantes....................100%</p><p>Apresentação correcta, mas com ausência de alguns elementos</p><p>relevantes</p><p>ou</p><p>Apresentação completa, mas com algumas incorrecções (por exemplo,</p><p>não respeitar o domínio de uma função) .................................................................. 60%</p><p>Apresentação incompleta e com algumas incorrecções .......................................... 20%</p><p>Ausência de explicação ou simples referências do tipo «Vi na</p><p>calculadora»................................................................................................................. 0 %</p><p>7.2. Apresentação do(s) valor(es):</p><p>Para cada valor que o examinando deve apresentar, os critérios específicos podem</p><p>indicar um intervalo admissível. O valor apresentado pelo examinando pode pertencer,</p><p>ou não, a esse intervalo.</p><p>• Se o valor pertencer ao intervalo, deve ser atribuída a cotação máxima prevista</p><p>para essa apresentação, a menos de qualquer penalização prevista nos critérios</p><p>específicos, por desrespeito relativo ao número de casas decimais com que o</p><p>resultado deve ser apresentado.</p><p>• Se o valor não pertencer ao intervalo, deve ser atribuída a cotação de 0 (zero)</p><p>pontos.</p><p>735/C/4</p><p>8. Quando, num item, é pedida uma forma específica de apresentação do resultado final (por</p><p>exemplo, «em minutos», «em percentagem», etc.), este deve ser apresentado na forma</p><p>pedida. Se o resultado final apresentado pelo examinando não respeitar a forma pedida no</p><p>enunciado (por exemplo, se o enunciado pedir o resultado em minutos, e o examinando o</p><p>apresentar em horas), devem ser atribuídos 0 (zero) pontos à etapa correspondente ao</p><p>resultado final. No entanto, o examinando não deve ser penalizado se não indicar a unidade</p><p>em que é pedido o resultado (por exemplo, se o resultado final for 12 minutos, ou 12 metros,</p><p>e o examinando escrever simplesmente 12, não deve ser penalizado).</p><p>9. O examinando deve respeitar sempre a instrução relativa à apresentação de todos os</p><p>cálculos e de todas as justificações. Se, numa etapa, o examinando não respeitar esta</p><p>instrução, apresentando algo (valor, quadro, tabela, gráfico, etc.) que não resulte de trabalho</p><p>anterior, deve ser atribuída a cotação de 0 (zero) pontos a essa etapa. Todas as etapas</p><p>subsequentes que dela dependam devem ser igualmente cotadas com 0 (zero) pontos.</p><p>10. O examinando deve respeitar sempre qualquer instrução relativa ao método a utilizar na</p><p>resolução de um item (por exemplo, «equacione o problema», «resolva graficamente», etc.).</p><p>Na resolução apresentada pelo examinando, deve ser inequívoco, pela apresentação de</p><p>todos os cálculos e de todas as justificações, o cumprimento da instrução. Se tal não</p><p>acontecer, considera-se que o examinando não respeitou a instrução. A etapa em que se dá</p><p>o desrespeito e todas as subsequentes que dela dependam devem ser cotadas com 0 (zero)</p><p>pontos.</p><p>11. Se, na resolução de um item, o examinando utilizar simbologia, ou escrever uma expressão,</p><p>inequivocamente incorrecta do ponto de vista formal (por exemplo, se escrever o símbolo de</p><p>igualdade onde deveria estar o símbolo de equivalência), deve ser penalizado em um ponto,</p><p>na cotação total a atribuir a esse item. Esta penalização não se aplica no caso em que tais</p><p>incorrecções ocorram apenas em etapas cotadas com 0 (zero) pontos, nem a eventuais</p><p>utilizações do símbolo de igualdade, onde, em rigor, deveria estar o símbolo de igualdade</p><p>aproximada.</p><p>12. Existem itens em cujo enunciado é dada uma instrução relativa ao número mínimo de casas</p><p>decimais que o examinando deve conservar, sempre que, em cálculos intermédios, proceder</p><p>a arredondamentos. Indicam-se, a seguir, as penalizações a aplicar, na cotação total a</p><p>atribuir ao item, em caso de desrespeito dessa instrução e/ou de arredondamentos mal</p><p>efectuados.</p><p>Todos os valores intermédios estão de acordo com a instrução, mas existe,</p><p>pelo menos, um valor intermédio mal arredondado...................................................... -1 ponto</p><p>Todos os valores intermédios estão bem arredondados, mas existe, pelo</p><p>menos, um que não está de acordo com a instrução................................................... -1 ponto</p><p>Existe, pelo menos, um valor intermédio mal arredondado e existe, pelo</p><p>menos, um que não está de acordo com a instrução ................................................ -2 pontos</p><p>V.S.F.F.735/C/5</p><p>Critérios específicos</p><p>1.1. .................................................................................................................. 10</p><p>Averiguar da viabilidade de execução da proposta da Isabel ...............................5</p><p>Número de margaridas necessárias ......... 1 !( ‚ "' $ ( ‚ ) œ "')</p><p>Número de rosas necessárias .........................1 !( ‚ % $ ( ‚ ) œ )%</p><p>Número de violetas necessárias ................... 1 !( ‚ ) $ ( ‚ ) œ ""#</p><p>Concluir que a proposta é viável .............................................................. 2</p><p>Averiguar da viabilidade de execução da proposta do Dinis ................................ 5</p><p>Número de margaridas necessárias !"! ‚ "' $ & ‚ ) œ #!!</p><p>ouNúmero de violetas necessárias ................. 3 !"! ‚ ) $ & ‚ ) œ "#!</p><p>Concluir que a proposta não é viável .......................................................2</p><p>1.2. .................................................................................................................. 20</p><p>Indicar a função objectivo, , onde designa oP œ $B $ # C B</p><p>número de arranjos do tipo A e designa o número deC</p><p>arranjos do tipo B ................................................................................................. 2</p><p>Indicar as restrições .............................................................................................. 8</p><p>B � ! .....................................................................................................1</p><p>C � ! ..................................................................................................... 1</p><p>"' B $ ) C Ÿ "*# ...............................................................................2</p><p>% B $ ) C Ÿ )) ................................................................................... 2</p><p>) B $ ) C Ÿ ""# ................................................................................. 2</p><p>Apresentar o gráfico da região admissível ............................................................ 5</p><p>Indicar os valores de e para os quais é máxima a funçãoB C</p><p>objectivo e ............................................................................. 5 ÐB œ "! C œ %Ñ</p><p>2.1. .................................................................................................................. 10</p><p>Equacionar o problema ...........................................7 Œ #"! " &#</p><p># ‚ 8 œ %'&</p><p>Resolver a equação ou verificar que é solução ............................................3 "&</p><p>735/C/6</p><p>2.2. .................................................................................................................. 10</p><p>Equacionar o problema ........................................................ 7 !&# œ "! $ "% 5</p><p>Resolver a equação, concluindo que .......................................................3 5 œ $</p><p>2.3. .................................................................................................................. 10</p><p>Concluir que há lugares com má visibilidade ........................................ 2 # ‚ "&</p><p>Determinar a probabilidade pedida ...................................................................... 8</p><p>Concluir que o número de casos</p><p>favoráveis é ......................................................</p>
<p>seleccionado, ao acaso, de entre o</p><p>grupo de turistas inquiridos».</p><p>1.º Processo:</p><p>Representar graficamente o problema .................................................................. 10 pontos</p><p>Respeitar a forma da curva de Gauss ............................................ 2 pontos</p><p>Assinalar, no gráfico, o valor médio da distribuição ........................ 2 pontos</p><p>Assinalar, no gráfico, a região correspondente a P(X > 2900) ... 3 pontos</p><p>Assinalar, no gráfico, a região correspondente a P(X < 2000) ... 3 pontos</p><p>Concluir que é mais provável que o valor do vencimento mensal individual seja</p><p>inferior a 2000 euros .......................................................................................... 5 pontos</p><p>P N−� �</p><p>Prova 735 • Página C/12/ 15</p><p>2.º Processo:</p><p>Determinar, com o auxílio da calculadora, o valor aproximado</p><p>de P(X > 2900) (0,05) (ver nota 1) ............................................................... 5 pontos</p><p>Determinar, com o auxílio da calculadora, o valor aproximado</p><p>de P(X < 2000) (0,09) (ver nota 2) ................................................................. 5 pontos</p><p>Concluir que é mais provável que o valor do vencimento mensal individual seja</p><p>inferior a 2000 euros ............................................................................................ 5 pontos</p><p>3.º Processo:</p><p>Escrever (ou equivalente) ............................... 4 pontos</p><p>Escrever</p><p>(ou equivalente) ..................................................................................................... 5 pontos</p><p>Referir que (ou equivalente) ................................. 1 pontos</p><p>Concluir que é mais provável que o valor do vencimento mensal individual seja</p><p>inferior a 2000 euros .......................................................................................... 5 pontos</p><p>4.º Processo:</p><p>Escrever (ou equivalente) .............................. 4 pontos</p><p>Escrever</p><p>(ou equivalente) .................................................................................................... 5 pontos</p><p>Referir que (ou equivalente) .......................................... 1 pontos</p><p>Concluir que é mais provável que o valor do vencimento mensal individual seja</p><p>inferior a 2000 euros .......................................................................................... 5 pontos</p><p>Notas:</p><p>1. Se o examinando apresentar o valor 0,04 ou um valor correctamente aproximado de</p><p>P(X > 2900), com mais do que duas casas decimais, a pontuação a atribuir nesta etapa deverá</p><p>ser desvalorizada em 1 ponto.</p><p>2. Se o examinando apresentar o valor 0,08 ou um valor correctamente aproximado de</p><p>P(X < 2000), com mais do que duas casas decimais, a pontuação a atribuir nesta etapa deverá</p><p>ser desvalorizada em 1 ponto.</p><p>( )P X1900≤ < 2000</p><p>( ) ( ) ( )P X P X P X< 2000 = < 1900 + 1900 ≤ < 2000</p><p>( ) ( )P X P X> 2900 = < 1900</p><p>( )P X2800 < ≤ 2900 > 0</p><p>( ) ( ) ( )P X P X P X> 2800 = 2800 < ≤ 2900 + > 2900</p><p>( ) ( )P X P X< 2000 = > 2800</p><p>Prova 735 • Página C/13/ 15</p><p>GRUPO IV</p><p>......................................................................................................................................................... 20 pontos</p><p>Apresenta-se, a seguir, um exemplo de resposta:</p><p>«A opção A) não está correcta, pois a expressão apresentada para M define uma função</p><p>decrescente, o que significaria que o número de milhafres estava a diminuir em vez de</p><p>aumentar. [ver nota 1]</p><p>A opção B) não está correcta, devido ao facto de a expressão apresentada para E definir</p><p>uma função que nunca toma valores inferiores a 0,8 , porque 7 × 1,03 −x é maior que</p><p>zero para qualquer valor de x . Esta propriedade implicaria que o número de esquilos nunca</p><p>fosse inferior a oitenta efectivos, contrariando a informação de que o número de esquilos da</p><p>Reserva é, actualmente, inferior a cinquenta. [ver nota 2]</p><p>A opção C) também não está correcta, devido à expressão apresentada para M, pois,</p><p>embora a expressão corresponda a uma função crescente, verificar-se-ia, por exemplo, que</p><p>M(100) ≈ 5,05. Na situação descrita, isto significaria que, passados 100 meses desde o</p><p>início do ano de 2000, ou seja, no início de Maio de 2008, o número de milhafres já seria</p><p>cerca de 505 e, portanto, mais do que 500, o que contrariaria a informação dada.</p><p>[ver nota 3]»</p><p>Notas:</p><p>1. O examinando também pode justificar a incorrecção desta opção pelo facto de a expressão</p><p>apresentada para E definir uma função crescente, o que significaria que a população de esquilos</p><p>estaria a aumentar, e não a diminuir, conforme é referido no enunciado.</p><p>2. O examinando também pode justificar a incorrecção desta opção referindo-se à assímptota</p><p>horizontal de equação y = 0,8 , relativa ao gráfico da função definida pela expressão apresentada</p><p>para E.</p><p>3. O examinando também pode justificar a incorrecção desta opção sem apresentar um exemplo,</p><p>desde que, na sua resposta, esteja explícito que a expressão apresentada para M levaria à</p><p>conclusão de que, actualmente, o número de milhafres já teria excedido as cinco centenas.</p><p>No caso de o examinando apresentar um exemplo, tal como o referido, M (100) ≈ 5,05 , deve</p><p>interpretar os valores, considerando a situação descrita.</p><p>Tal como o exemplo de resposta ilustra, a composição deve abordar os seguintes tópicos:</p><p>• apresentação de uma razão cientificamente válida que justifique, inequivocamente, que a</p><p>opção A) está incorrecta;</p><p>• apresentação de uma razão cientificamente válida que justifique, inequivocamente, que a</p><p>opção B) está incorrecta;</p><p>• apresentação de uma razão cientificamente válida que justifique, inequivocamente, que a</p><p>opção C) está incorrecta.</p><p>Prova 735 • Página C/14/ 15</p><p>Na tabela seguinte, indica-se como deve ser classificado este item, de acordo com os níveis</p><p>de desempenho no domínio da comunicação escrita em língua portuguesa, descritos nos</p><p>critérios gerais, e com os níveis de desempenho no domínio específico da disciplina.</p><p>** Descritores apresentados nos critérios gerais.</p><p>** Apenas podem ser atribuídas classificações correspondentes a um dos valores constantes do</p><p>quadro. Não há lugar a classificações intermédias.</p><p>No caso de a resposta não atingir o nível 1 de desempenho no domínio específico da disciplina, a</p><p>classificação a atribuir é zero pontos. Neste caso, não é classificado o desempenho no domínio da</p><p>comunicação escrita em língua portuguesa.</p><p>Descritores do nível de desempenho no domínio</p><p>da comunicação escrita em língua portuguesa</p><p>Descritores do nível de desempenho</p><p>no domínio específico da disciplina</p><p>Níveis*</p><p>1 2 3</p><p>N</p><p>ív</p><p>e</p><p>is</p><p>**</p><p>3 A composição contempla correctamente os três tópicos. 18 19 20</p><p>2 A composição contempla correctamente apenas dois tópicos. 12 13 14</p><p>1 A composição contempla correctamente apenas um tópico. 6 7 8</p><p>Prova 735 • Página C/15/ 15</p><p>Proposta de Resolução</p><p>1. Lado do quadrado: 12 dm</p><p>Figura 1: raio do círculo = 6 dm</p><p>Área do círculo =</p><p>Figura 2: raio de cada círculo = 3 dm</p><p>Soma das áreas dos círculos =</p><p>2. O número de círculos é sempre o quadrado do n.º de ordem da figura. O número de círculos do</p><p>10º painel é, portanto, 10</p><p>3. 1ª tela: 4 � 12 dm = 48</p><p>2ª tela: 6 � 12 dm = 72</p><p>3ª tela: 8 � 12 dm = 96</p><p>....</p><p>A quantidade de fio em cada tela, em dm, é uma progressão aritmética de primeiro termo 48 e</p><p>razão 2 � 12 � 24.</p><p>Então, a 10ª tela precisa de</p><p>No total, as 10 telas precisam de</p><p>O artista terá de aplicar 156</p><p>1. Volume do cilindro = Área da base</p><p>Volume de cada cone = Volume do cilindro</p><p>Como foram enchidos dois cones, resta um terço do volume de água</p><p>Volume de água restante no cilindro =</p><p>Mas o volume de água restante no cilindro é um cilindro com a mesma base e altura</p><p>Então</p><p>roposta de Resolução da Prova de Matemática B</p><p>(21 de Junho de 2010)</p><p>GRUPO I</p><p>m</p><p>raio do círculo = 6 dm</p><p>Área do círculo = � � 6	 = 36 �</p><p>raio de cada círculo = 3 dm</p><p>Soma das áreas dos círculos = 4 � � � 3	 = 36 �</p><p>O número de círculos é sempre o quadrado do n.º de ordem da figura. O número de círculos do</p><p>10º painel é, portanto, 102</p>
<p>5 %'& 6 # ‚ "&</p><p>Concluir que a probabilidade pedida é</p><p>#*</p><p>$" ( ) .......................................................................3 ver nota</p><p>ou</p><p>Concluir que a probabilidade de lhe</p><p>calhar um lugar com má visibilidade é</p><p>$!</p><p>%'& ......................................................................................... 3</p><p>Concluir que a probabilidade pedida é</p><p>dada por .............................................................. 3 " 6 $!</p><p>%'&</p><p>Concluir que a probabilidade pedida é</p><p>#*</p><p>$" ( ) .......................................................................2 ver nota</p><p>Nota:Se o examinando não apresentar o resultado na forma de fracção irredutível, a</p><p>cotação a atribuir à sua resposta deve ser desvalorizada em 1 ponto.</p><p>V.S.F.F.735/C/7</p><p>3.1. .................................................................................................................. 15</p><p>Área da mancha ao fim de cinco segundos .............................................3 !"'1</p><p>Equacionar o problema .................................................. 4 Œ #"!!</p><p>""% /&5 œ "'1</p><p>Resolver a equação ( )............................................................................ 8 ver nota</p><p>Nota:O examinando pode resolver a equação analiticamente ou graficamente.</p><p>Se o examinando resolver a equação graficamente, com recurso à</p><p>calculadora, a cotação desta etapa deve ser repartida da seguinte forma:</p><p>Explicação do método utilizado (ver critério geral 7.1.) ..................6</p><p>5 ¸ 6 ! #), .................................................................................. 2</p><p>O intervalo admissível para o valor de é 5 Ò 6 ! #* 6 ! #(Ó, ; ,</p><p>(ver critério geral 7.2.)</p><p>Se o examinando resolver a equação analiticamente, a cotação desta etapa</p><p>deve ser repartida da seguinte forma:</p><p>"!!</p><p>""% /&5 œ "' Í / œ 6 "1 & 5 " "!!</p><p>% "'ˆ ‰</p><p>1</p><p>..................3</p><p>5 ¸ 6 ! #), .................................................................................. 5</p><p>3.2. .................................................................................................................. 15</p><p>Referir que a taxa de variação média da função noE</p><p>intervalo é dada por .................................................... 5 Ò! %Ó, E % +E !</p><p>%+!</p><p>� � � �</p><p>E % +E !</p><p>%+!</p><p>� � � � ¸ & ......................................................................4 (ver nota)</p><p>Interpretar o valor obtido ......................................................................................6</p><p>(«Nos primeiros quatro segundos, a área de tecido</p><p>ocupada pela mancha aumenta, em média, por& -7 #</p><p>segundo. Nos» ou, de uma forma mais informal, «</p><p>primeiros quatro segundos, a mancha aumenta, em</p><p>média, por segundo.& -7 # »)</p><p>Nota:Se o examinando não apresentar o resultado arredondado às unidades, a</p><p>cotação a atribuir à sua resposta deve ser desvalorizada em 2 pontos.</p><p>735/C/8</p><p>4.1.1. ............................................................................................................... 15</p><p>6 " Ÿ -9= -B Ÿ "� � ........................................................................................ 5</p><p>6 , Ÿ , -9= -B Ÿ ,� � ......................................................................................5</p><p>+ 6 , Ÿ + $ , -9= -B Ÿ + $ ,� � ...................................................................5</p><p>4.1.2. ............................................................................................................... 15</p><p>Estabelecer a equivalência: é período da função #</p><p>-</p><p>1 Í</p><p>Í +$ , -9= - B $ œ + $ , -9= Ð-BÑ ” •Œ ##</p><p>-</p><p>1 .........................................5</p><p>+ $ , -9= - B $ œ + $ , -9= - B $ #” •Œ # !#</p><p>-</p><p>1</p><p>1 .......................................4</p><p>+ $ , -9= - B $ # œ + $ , -9= - B ! !1 .........................................................6</p><p>4.2. .................................................................................................................. 15</p><p>Determinar e .................................................................................................. 8 + ,</p><p>+ 6 , œ 6 ! (" + $ , œ ! )(, , e ......................................... 4</p><p>+ œ ! !), .......................................................................................2</p><p>, œ ! (*, .......................................................................................2</p><p>Determinar .......................................................................................................... 7 -</p><p>#</p><p>-</p><p>1 œ ! !!#, ................................................................................4</p><p>- ¸ $"%# ......................................................................................3</p><p>735/C/9</p><p>5.1. .................................................................................................................. 10</p><p>Concluir que o número de casos favoráveis é .......................................4 "$ &!!</p><p>Concluir que o número de casos possíveis é ........................................ 4 #) &!!</p><p>Determinar a probabilidade pedida ( )........................................2 Œ #*</p><p>"* ver nota</p><p>Nota: Se o examinando não apresentar o resultado na forma de fracção</p><p>irredutível, a cotação a atribuir à sua resposta deve ser desvalorizada em 1</p><p>ponto.</p><p>5.2. .................................................................................................................. 10</p><p>Indicar o coeficiente de correlação ....................................................2 � �6 ! *(,</p><p>Reproduzir as listas introduzidas na calculadora ..................................... (2+2) 4</p><p>Interpretar o valor obtido ...................................................................................... 4</p><p>Interpretação relativa ao sinal .................................................... 2</p><p>Interpretação relativa ao valor absoluto ..................................... 2</p><p>5.3. .................................................................................................................. 15</p><p>; œ "!!!8 : ( ) ..................................................................................... 7 ver nota</p><p>Substituir por .......................................................................4 8 6 ! !$ : $ "!,</p><p>Apresentar a expressão na forma de um polinómio reduzido ............................. 4</p><p>Nota:</p><p>Caso o examinando considere , a cotação a atribuir a esta etapa; œ 8:</p><p>deverá ser de 4 pontos.</p><p>6. ..................................................................................................................... 30</p><p>Determinar a amplitude, em radianos, do ângulo ........................ 5 α ,Ð" !*%*Ñ</p><p>Determinar o perímetro da base do cone ,Ð$* %"'%Ñ ......................................7</p><p>Determinar o raio da base do cone ,Ð' #($$Ñ ................................................. 5</p><p>Determinar a altura do cone ,Ð"' )("%Ñ ...........................................................6</p><p>Determinar o volume do cone ,Ð'*& $Ñ .............................................................5</p><p>Responder à questão colocada ............................................................................ 2</p><p>EXAMES NACIONAIS DO ENSINO SECUNDÁRIO, 2006 – 1.ª FASE GRELHA DE CLASSIFICAÇÃO DE MATEMÁTICA – B (Cód. 735)</p><p>1.1. 1.2. 2.1. 2.2. 2.3. 3.1. 3.2. 4.1.1. 4.1.2. 4.2. 5.1. 5.2. 5.3. 6. Código</p><p>Confidencial</p><p>da Escola</p><p>Número</p><p>Convencional</p><p>da Prova (10) (20) (10) (10) (10) (15) (15) (15) (15) (15) (10) (10) (15) (30)</p><p>TOTAL</p><p>DA</p><p>PROVA</p><p>(200)</p>
<p>Data ____/____/____ O Professor Classificador _________________________________________</p><p>1</p><p>Resolução da Prova 735 (Matemática B)</p><p>1.</p><p>1.1</p><p>Proposta da Isabel:</p><p>margaridas rosas violetas</p><p>7 arranjos tipo A 112 28 56</p><p>7 arranjos tipo B 56 56 56</p><p>Total de flores necessárias 168 84 112</p><p>Proposta do Dinis:</p><p>margaridas rosas violetas</p><p>10 arranjos tipo A 160 40 80</p><p>5 arranjos tipo B 40 40 40</p><p>Total de flores necessárias 200 80 120</p><p>A proposta da Isabel é viável e a proposta do Dinis não é viável, uma vez que não</p><p>existem margaridas (nem violetas) em número suficiente.</p><p>1.2. Sejam x = n.º arranjos do tipo A e</p><p>y = n.º arranjos do tipo B.</p><p>Pretendemos maximizar a função yxL 23 += (função objectivo).</p><p>De acordo com o problema podemos organizar os dados do seguinte modo:</p><p>n.º margaridas n.º rosas n. violetas</p><p>x arranjos do tipo A 16x 4x 8x</p><p>y arranjos do tipo B 8y 8y 8y</p><p>n.º total de flores 16x+8y 4x+8y 8x+8y</p><p>constrangimentos 16x+8y≤ 192 4x+8y≤ 88 8x+8y≤ 112</p><p>As restrições para as variáveis são, então,</p><p>⎪</p><p>⎪</p><p>⎩</p><p>⎪</p><p>⎪</p><p>⎨</p><p>⎧</p><p>∈</p><p>≤+</p><p>≤+</p><p>≤+</p><p>0,</p><p>11288</p><p>8884</p><p>192816</p><p>lNyx</p><p>yx</p><p>yx</p><p>yx</p><p>⇔</p><p>⎪</p><p>⎩</p><p>⎪</p><p>⎨</p><p>⎧</p><p>+−≤</p><p>+−≤</p><p>+−≤</p><p>14</p><p>115,0</p><p>242</p><p>xy</p><p>xy</p><p>xy</p><p>2</p><p>Geometricamente, tem-se:</p><p>1.º processo:</p><p>2</p><p>5,123 LxyyxL +−=⇔+= .</p><p>Esta expressão define a família de rectas com declive 5,1− . A recta da família</p><p>que nos dá a informação sobre o maior lucro é, por observação geométrica, a que</p><p>contém o ponto de coordenadas (10, 4).</p><p>Logo, devem produzir-se 10 arranjos do tipo A e 4 do tipo B (o lucro será de 38</p><p>euros ( )42103 ×+× ).</p><p>2.º processo:</p><p>A solução óptima é, habitualmente, um dos vértices do polígono de</p><p>constrangimentos. Assim, basta testar cada uma das soluções.</p><p>Verifica-se que o lucro máximo é no ponto (10, 4).</p><p>x y L =3x+2y</p><p>0 11 22</p><p>6 8 34</p><p>10 4 38</p><p>12 0 36</p><p>3</p><p>2.</p><p>2.1. O n.º de cadeiras de cada uma das n filas da plateia são termos consecutivos</p><p>de uma progressão aritmética. Sabemos que a soma destes n termos é igual a 465.</p><p>Assim,</p><p>1546531465</p><p>2</p><p>5210</p><p>=⇔=⇔=×</p><p>+ nnn</p><p>Logo, confirma-se que a plateia tem 15 filas.</p><p>2.2.</p><p>1.ª fila 2.ª fila 3.ª fila … 15.ª fila</p><p>10 10 + k 10 + 2k … 10 + 14k</p><p>Tem-se, então,</p><p>3141052 =⇔+= kk</p><p>Assim, o valor de k é igual a 3.</p><p>2.3. Das 15 filas da plateia existem 30 lugares com má visibilidade, 2 em cada</p><p>uma das filas. Assim, a Nazaré verá satisfeita a sua pretensão se lhe for atribuído um</p><p>dos 435 bilhetes correspondentes aos restantes lugares.</p><p>Tem-se:</p><p>31</p><p>29</p><p>465</p><p>435</p><p>==p ou</p><p>31</p><p>29</p><p>465</p><p>301 =−=p</p><p>Logo, a probabilidade pedida é igual a</p><p>31</p><p>29</p><p>3.</p><p>3.1. ππ 1644 2 =×=⇒= Ar (cm2) → área da mancha circular de raio 4</p><p>Para determinar o valor pedido tem de resolver-se a condição π16)5( =A ,</p><p>equivalente a</p><p>π16</p><p>41</p><p>100</p><p>5 =</p><p>+ ke</p><p>.</p><p>Considerando as funções xe</p><p>y 51 41</p><p>100</p><p>+</p><p>= e π162 =y , vamos calcular o ponto de</p><p>intersecção dos seus gráficos.</p><p>No editor de funções da calculadora obtém-se:</p><p>O valor de k é aproximadamente igual a -0,28.</p><p>y1 = 16π</p><p>y2</p><p>4</p><p>3.2.</p><p>[ ]</p><p>( ) ( ) 5</p><p>4</p><p>20461,40</p><p>4</p><p>04...</p><p>4;0</p><p>≈</p><p>−</p><p>=</p><p>−</p><p>=</p><p>AAvmt (cm2/s)</p><p>Durante os quatro primeiros segundos, a área da mancha</p><p>aumentou, em média, 5 2cm por segundo.</p><p>4.</p><p>4.1.1. Sabe-se que</p><p>1)(cos1 ≤≤− cx , para todo o valor de x . Como 0>b , virá</p><p>bcxbb ≤≤− )(cos e</p><p>bacxbaba +≤+≤− )(cos , ou seja,</p><p>bayba +≤≤−</p><p>Assim, como queríamos mostrar, o contradomínio da função é o intervalo</p><p>[ ]baba +− , .</p><p>4.1.2.</p><p>c</p><p>π2 é período da função se e só se</p><p>( )xy</p><p>c</p><p>xy =⎟</p><p>⎠</p><p>⎞</p><p>⎜</p><p>⎝</p><p>⎛ +</p><p>π2 , para todo o valor de x do domínio da função.</p><p>Tem-se</p><p>Confirma-se, assim, o pretendido.</p><p>4.2. Por observação das figuras 1 e 2 o contradomínio da função é</p><p>[ ]87,0;71,0− , intervalo de amplitude 1,58.</p><p>Amplitude do intervalo [ ] bbaba 2, =+− .</p><p>Logo, 79,058,12 =⇔= bb .</p><p>Como 87,0=+ ba , temos 08,079,087,0 =−=a</p><p>Finalmente, dois maximizantes consecutivos são 0,002 e 0,004. O período</p><p>positivo mínimo da função é 0,002. Assim,</p><p>3142002,02</p><p>≈⇔= c</p><p>c</p><p>π</p><p>Logo, 79,0,08,0 == ba e 3142≈c .5.</p><p>π2 é período da função co-seno</p><p>( ) ( ) ( )xyxcbaxcba</p><p>c</p><p>xcba</p><p>c</p><p>xy =+=++=⎥</p><p>⎦</p><p>⎤</p><p>⎢</p><p>⎣</p><p>⎡</p><p>⎟</p><p>⎠</p><p>⎞</p><p>⎜</p><p>⎝</p><p>⎛ ++=⎟</p><p>⎠</p><p>⎞</p><p>⎜</p><p>⎝</p><p>⎛ + cos2cos2cos2 πππ</p><p>5</p><p>5.1. O número total de clientes é igual ao número total de telemóveis vendidos.</p><p>Assim, a empresa vendeu 28,5 milhares de telemóveis (7 + 6,5 + 5 + 4,5 + 3 + 2,5).</p><p>Destes, 13,5 milhares foram vendidos a um preço inferior a 180 euros.</p><p>A probabilidade pedida é, assim,</p><p>19</p><p>9</p><p>5,28</p><p>5,13</p><p>==p .</p><p>5.2. Introduzindo em L1 o preço, em euros, de cada telemóvel e em L2 o número</p><p>de unidades vendidas, em milhares, obtém-se</p><p>Para este conjunto de dados, o coeficiente de</p><p>correlação linear é aproximadamente igual a -0,97 (ver</p><p>figura ao lado).</p><p>Este valor indica-nos que existe uma correlação</p><p>negativa muito forte entre as variáveis n e p . As variáveis</p><p>variam inversamente, isto é, à medida que o preço do</p><p>telemóvel aumenta, o número de unidades vendidas</p><p>diminui e vice-versa.</p><p>5.3. A quantia, em euros, que a empresa prevê vir a receber pela venda dos</p><p>telemóveis do novo modelo é dada por</p><p>pnq ×= 1000</p><p>Dado que 1003,0 +−= pn , vem</p><p>( ) pppppnq 10000301003,010001000 2 +−=+−=×=</p><p>6.</p><p>• 0949,1</p><p>18</p><p>16</p><p>≈⇒= ααsen (radianos)</p><p>• Sabe-se que</p><p>απ 22</p><p>18. conedobaseraiodecircunf PP</p><p>=</p><p>Assim,</p><p>0949,122</p><p>182</p><p>×</p><p>=</p><p>× conedobaseP</p><p>π</p><p>π</p><p>⇔ 4164,39=conedobaseP cm</p><p>6</p><p>• Cálculo do raio da base do cone:</p><p>2733,6</p><p>2</p><p>4164,394164,3922 ≈=⇔=⇔=</p><p>π</p><p>ππ rrrPbase cm</p><p>• Cálculo da altura do cone:</p><p>cmh</p><p>h</p><p>8714,16</p><p>2733,618 22</p><p>≈</p><p>⇔−=</p><p>• Cálculo do volume do cone:</p><p>3</p><p>2</p><p>3,695</p><p>8714,162733,6</p><p>3</p><p>1</p><p>cmV</p><p>V</p><p>≈</p><p>⇔×××= π</p><p>Ora, 33 5003,695 cmcm > , pelo que o filtro construído tem capacidade</p><p>superior a meio litro.</p><p>Fim</p><p>Esta proposta de resolução também pode ser consultada em http://www.apm.pt</p><p>V.S.F.F.</p><p>735/1</p><p>PROVA 735/11 Págs.</p><p>EXAME NACIONAL DO ENSINO SECUNDÁRIO</p><p>11.º Ano de Escolaridade (Decreto-Lei n.º 74/2004, de 26 de Março)</p><p>Curso Científico-Humanístico</p><p>de Artes Visuais</p><p>Duração da prova: 150 minutos 2.ª FASE</p><p>2006</p><p>PROVA ESCRITA DE MATEMÁTICA B</p><p>735/2</p><p>Identifique claramente os grupos e os itens a que responde.</p><p>Utilize apenas caneta ou esferográfica de tinta azul ou preta</p><p>(excepto nas respostas que impliquem a elaboração de</p><p>construções, desenhos ou outras representações).</p><p>É interdito o uso de «esferográfica-lápis» e de corrector.</p><p>As cotações da prova encontram-se na página 10.</p><p>A prova inclui um formulário (pág. 11).</p><p>V.S.F.F.735/3</p><p>Em todas as questões da prova, apresente o seu raciocínio de forma clara,</p><p>indicando todos os cálculos que tiver de efectuar e todas as justificações</p><p>necessárias.</p><p>Apresente uma única resposta a cada item. Se escrever mais do que uma</p><p>resposta, deve indicar de forma inequívoca a que pretende que seja classificada</p><p>(riscando todas as que pretende anular).</p><p>Sempre que, na resolução de um problema, recorrer à sua calculadora, apresente</p><p>todos os elementos recolhidos na sua utilização. Mais precisamente:</p><p>• sempre que recorrer às capacidades gráficas da sua calculadora, apresente o</p><p>gráfico, ou gráficos, obtido(s), bem como coordenadas de pontos relevantes para</p><p>a resolução do problema proposto (por exemplo, coordenadas de pontos de</p><p>intersecção de gráficos, máximos, mínimos, etc.);</p><p>• sempre que recorrer a uma tabela obtida na sua calculadora, asapresente todas</p><p>linhas da tabela relevantes para a resolução do problema proposto;</p><p>• sempre que recorrer a estatísticas obtidas na sua calculadora (média, desvio</p><p>padrão, coeficiente de correlação, declive e ordenada na origem de uma recta de</p><p>regressão, etc.), as listas que introduziu na calculadora para as obter.apresente</p><p>735/4</p><p>V.S.F.F.735/5</p><p>1. Num certo concelho do nosso país, uma empresa de informática vai facultar um estágio,</p><p>durante as férias do Verão, aos alunos do 11.º</p>
<p>ano, das escolas desse concelho, que</p><p>tenham obtido classificação final superior a 15 valores, quer a Matemática, quer a</p><p>Informática.</p><p>As classificações finais nas disciplinas de Matemática e de Informática obtidas pelos 50</p><p>alunos desse concelho que satisfaziam as condições requeridas foram tratadas</p><p>estatisticamente.</p><p>Desse tratamento resultaram os gráficos apresentados a seguir.</p><p>Matemática Informática</p><p>1.1. Depois de ter calculado, para cada uma das disciplinas, a média e o desvio padrão</p><p>das classificações, a Ângela comentou: «As médias das classificações a Matemática</p><p>e a Informática são iguais, mas o mesmo não se passa com os desvios padrão».</p><p>1.1.1. Conclua que a Ângela tem razão na sua afirmação, calculando, para cada</p><p>uma das disciplinas, a média e o desvio padrão das classificações.</p><p>1.1.2. O Pedro, que estava a tratar os dados em conjunto com a Ângela,</p><p>comentou: «Quando me disseste que as médias eram iguais, eu,</p><p>observando os gráficos, concluí logo que os desvios padrão eram</p><p>diferentes».</p><p>Tendo em conta que o desvio padrão mede a variabilidade dos dados</p><p>relativamente à média, explique como poderá o Pedro ter chegado àquela</p><p>conclusão.</p><p>1.2. Sabe-se que, dos alunos que obtiveram 20 a Informática, metade obteve também 20</p><p>a Matemática.</p><p>A empresa vai sortear um prémio entre os alunos que obtiveram classificação igual</p><p>ou superior a 19, na disciplina de Matemática.</p><p>Qual é a probabilidade de o prémio sair a um aluno que obteve 20 nas duas</p><p>disciplinas? Apresente o resultado na forma de fracção irredutível.</p><p>735/6</p><p>2. A Ana e a Fátima têm de ler, para a disciplina de Português, um livro com 255 páginas</p><p>numeradas, da página 1 (primeira página do livro) à página 255 (última página do livro).</p><p>2.1. As duas raparigas começam a ler o livro no mesmo dia, na página 1.</p><p>A Ana lê uma página no primeiro dia e, em cada um dos dias seguintes, lê o dobro</p><p>do número de páginas do dia anterior.</p><p>A Fátima lê três páginas no primeiro dia e, em cada um dos dias seguintes, lê mais</p><p>duas páginas do que no dia anterior.</p><p>2.1.1. Verifique que, ao fim de dias, a Ana já leu páginas e a Fátima8 # " "8</p><p>já leu páginas.8 $ #8#</p><p>2.1.2. Admita que a Ana acaba de ler o livro no dia 18 de Abril. Em que dia acaba</p><p>a Fátima de ler o livro? Justifique a sua resposta.</p><p>2.2. Escolhida, ao acaso, uma das 255 páginas numeradas do mesmo livro, qual é a</p><p>probabilidade de o número dessa página ter, pelo menos, dois algarismos e</p><p>começar por 2? Apresente o resultado na forma de percentagem, arredondado às</p><p>unidades.</p><p>3. Admita que, em condições ambientais normais, o número aproximado de aves de uma</p><p>certa população, anos após um determinado instante inicial, é dado por></p><p>RÐ>Ñ œ > � ! E"#&E</p><p>E$Ð"#&&EÑ / ! # >,</p><p>!e constante positiva</p><p>3.1. Verifique que é o número de aves existentes no instante inicial.E</p><p>3.2. Ao longo dos cinco anos que se seguiram ao instante inicial, a população cresceu</p><p>em condições ambientais normais. Nasceram 80 aves e morreram 57, não tendo</p><p>entrado nem saído mais aves da população.</p><p>Estime o número de aves que havia nessa população, no instante inicial, sabendo</p><p>que esse número era inferior a 25.</p><p>V.S.F.F.735/7</p><p>4. Na figura, está representado um projecto de uma escultura em cimento para o jardim de</p><p>uma escola, constituída por uma esfera colocada sobre um cubo.</p><p>Pretende-se que a escultura tenha uma altura total de 2 metros.</p><p>Apresentam-se, a seguir, as vistas de frente de três possíveis concretizações desse</p><p>projecto.</p><p>4.1. Designemos por o raio da esfera (em metros).B</p><p>4.1.1. Indique, na forma de intervalo de números reais, o conjunto dos valores</p><p>que a variável pode assumir.B</p><p>4.1.2. Mostre que o volume total, , em metros cúbicos, da escultura é dado, emZ</p><p>função de , porB</p><p>Z ÐBÑ œ B $ #%B " #%B $ )% &#%</p><p>$</p><p>1 $ #</p><p>4.1.3. e a aresta do cuboDetermine o raio da esfera de modo que o volume total</p><p>da escultura seja mínimo. Apresente os resultados em metros,</p><p>arredondados às centésimas.</p><p>4.2. Admita agora que o raio da esfera é metade da aresta do cubo.</p><p>Pretende-se pintar toda a superfície da escultura, excepto, naturalmente, a face do</p><p>cubo que está assente no chão.</p><p>Cada litro da tinta que vai ser utilizada permite pintar uma superfície de .# &7, #</p><p>Admitindo que esta tinta só é vendida em latas de 1 litro, quantas latas será</p><p>necessário comprar?</p><p>735/8</p><p>5. Como sabe, a Terra descreve</p><p>uma órbita elíptica em torno do</p><p>Sol.</p><p>Na figura está representado um</p><p>esquema dessa órbita. Está</p><p>assinalado o , o pontoperiélio</p><p>da órbita da Terra mais próximo</p><p>do Sol.</p><p>Na figura está assinalado um ângulo de amplitude radianos .B B − Ò!ß # Ò ˆ ‰1</p><p>Este ângulo tem o seu vértice no Sol, o seu lado origem passa no e o seu ladoperiélio</p><p>extremidade passa na Terra.</p><p>A distância , em milhões de quilómetros, da Terra ao Sol, é (aproximadamente) dada,.</p><p>em função de porBß , , . œ "%* ' Ð" " ! !"'( BÑcos</p><p>5.1. Determine a distância máxima e a distância mínima da Terra ao Sol.</p><p>Apresente os valores pedidos em milhões de quilómetros, arredondados às</p><p>décimas.</p><p>5.2. Sabe-se que verifica a relação em queB œ B " ! !"'( B# ></p><p>X</p><p>1 , , sen</p><p>• é o tempo, em dias, que decorre desde a passagem da Terra pelo > periélio até</p><p>ao instante em que atinge a posição correspondente ao ângulo ;B</p><p>• é o tempo que a Terra demora a descrever uma órbita completa ( dias).X $'& #%,</p><p>5.2.1. Mostre que, para , se tem .B œ > œ1</p><p>X</p><p># Interprete este resultado no contexto da situação descrita.</p><p>5.2.2. Sabe-se que a última passagem da Terra pelo ocorreu a uma certaperiélio</p><p>hora do dia 4 de Janeiro. Determine a distância a que a Terra se</p><p>encontrava do Sol, à mesma hora do dia 14 de Fevereiro. Apresente o</p><p>resultado em milhões de quilómetros, arredondado às décimas. Nos</p><p>valores intermédios, utilize, no mínimo, quatro casas decimais.</p><p>: a resolução desta questão envolve uma equação que deve serNota</p><p>resolvida graficamente, com recurso à calculadora.</p><p>V.S.F.F.735/9</p><p>6. Para estudar a Lei do Arrefecimento de um Corpo, a Joana aqueceu uma pequena</p><p>quantidade de água. Em seguida, deixou-a a arrefecer, medindo a temperatura em vários</p><p>instantes, a partir de um certo instante inicial.</p><p>De acordo com a referida lei, em cada instante, a taxa de variação da temperatura é</p><p>directamente proporcional à diferença entre a temperatura da água, nesse instante, e a</p><p>temperatura ambiente, que se considera constante.</p><p>Tem-se, portanto, que</p><p>X Ð>Ñ œ 5 X Ð>Ñ " Ew c d</p><p>em que:</p><p>• X Ð>Ñ >designa a temperatura da água, no instante ;</p><p>• X Ð>Ñw designa a taxa de variação da temperatura, nesse mesmo instante;</p><p>• E designa a temperatura ambiente;</p><p>• 5 é a constante de proporcionalidade.</p><p>Admita que, durante a experiência, o tempo foi medido em minutos e a temperatura em</p><p>graus Celsius.</p><p>Na tabela seguinte, estão valores da temperatura da água, registados de em ! & ! &, ,</p><p>minutos, com início no instante .> œ #</p><p>> # # & $ $ &</p><p>XÐ>Ñ )& ! )$ ) )# ' )" &</p><p>, ,</p><p>, , , ,</p><p>Tendo em conta os dados desta tabela e sabendo que a temperatura ambiente, no local</p><p>da experiência, era de 25 graus Celsius, estime o valor de .5</p><p>Apresente o resultado arredondado às centésimas.</p><p>Percorra sucessivamente as seguintes etapas:</p><p>• Determine a taxa de variação média da temperatura da água, nos intervalos</p><p>c d c d c d# à $ & # à $ # à # &, , e , .</p><p>• Tendo em conta os valores obtidos, estime a taxa de variação instantânea da</p><p>temperatura da água, no instante .> œ #</p><p>• Tendo em conta a fórmula dada acima, estime o valor de .5</p><p>FIM</p><p>735/10</p><p>COTAÇÕES</p><p>1. ............................................................................................. 25</p><p>1.1. .......................................................................... 15</p><p>1.1.1. .................................................... 8</p><p>1.1.2. .................................................... 7</p><p>1.2. .......................................................................... 10</p><p>2. .............................................................................................</p>
<p>32</p><p>2.1. .......................................................................... 22</p><p>2.1.1. .................................................. 12</p><p>2.1.2. .................................................. 10</p><p>2.2. .......................................................................... 10</p><p>3. ............................................................................................. 30</p><p>3.1. ......................................................................... 15</p><p>3.2. .......................................................................... 15</p><p>4. ............................................................................................. 43</p><p>4.1. ......................................................................... 30</p><p>4.1.1. .................................................... 5</p><p>4.1.2. .................................................. 15</p><p>4.1.3. .................................................. 10</p><p>4.2. .......................................................................... 13</p><p>5. ............................................................................................. 45</p><p>5.1. .......................................................................... 15</p><p>5.2. .......................................................................... 30</p><p>5.2.1. .................................................. 15</p><p>5.2.2. .................................................. 15</p><p>6. ............................................................................................. 25</p><p>TOTAL .................................................................................................. 200</p><p>735/11</p><p>Formulário</p><p>Comprimento de um arco de circunferência</p><p>α α< " < ( )amplitude, em radianos, do ângulo ao centro raio; "</p><p>Áreas de figuras planas</p><p>Losango: H3+198+67+39<‚H3+198+67/89<</p><p>#</p><p>Trapézio: F+=/7+39<0F+=/7/89<</p><p># ‚ E6>?<+</p><p>Polígono regular: Semiperímetro Apótema‚</p><p>Sector circular: α <#</p><p># (α" amplitude,</p><p>em radianos, do ângulo ao centro raio; < " )</p><p>Áreas de superfícies</p><p>Área lateral de um cone: 1 < 1</p><p>( )< 1" "raio da base geratriz;</p><p>Área de uma superfície esférica: % <1 #</p><p>( )< " raio</p><p>Volumes</p><p>Pirâmide: "$ ‚ Área da base Altura‚</p><p>Cone: "$ ‚ Área da base Altura‚</p><p>Esfera: %$ $1 ( )< < " raio</p><p>Progressões</p><p>Soma dos primeiros termos de uma8</p><p>Prog. Aritmética: ? $?</p><p>#" 8 ‚ 8</p><p>Prog. Geométrica: ? ‚" "& <</p><p>"& <</p><p>8</p><p>V.S.F.F.</p><p>735/C/1</p><p>PROVA 735/C/12 Págs.</p><p>EXAME NACIONAL DO ENSINO SECUNDÁRIO</p><p>11.º Ano de Escolaridade (Decreto-Lei n.º 74/2004, de 26 de Março)</p><p>Curso Científico-Humanístico</p><p>de Artes Visuais</p><p>Duração da prova: 150 minutos 2.ª FASE</p><p>2006</p><p>PROVA ESCRITA DE MATEMÁTICA B</p><p>COTAÇÕES</p><p>1. 25............................................................................................................</p><p>1.1. ...................................................................................15</p><p>1.1.1. .................................................... 8</p><p>1.1.2. .................................................... 7</p><p>1.2. ...................................................................................10</p><p>2. 32............................................................................................................</p><p>2.1. ...................................................................................22</p><p>2.1.1. .................................................. 12</p><p>2.1.2. .................................................. 10</p><p>2.2. ...................................................................................10</p><p>3. 30............................................................................................................</p><p>3.1. .................................................................................. 15</p><p>3.2. ...................................................................................15</p><p>4. 43............................................................................................................</p><p>4.1. .................................................................................. 30</p><p>4.1.1. .................................................... 5</p><p>4.1.2. .................................................. 15</p><p>4.1.3. .................................................. 10</p><p>4.2. ...................................................................................13</p><p>5. 45............................................................................................................</p><p>5.1. ...................................................................................15</p><p>5.2. ...................................................................................30</p><p>5.2.1. .................................................. 15</p><p>5.2.2. .................................................. 15</p><p>6. 25............................................................................................................</p><p>TOTAL .................................................................................................. 200</p><p>735/C/2</p><p>CRITÉRIOS DE CLASSIFICAÇÃO</p><p>Critérios gerais</p><p>1. Se o examinando se enganar na identificação do item a que está a responder, ou se a omitir,</p><p>mas, pela resolução apresentada, for possível identificá-lo inequivocamente, a resposta</p><p>deve ser vista e classificada.</p><p>2. Se o examinando apresentar mais do que uma resposta a um item, e não indicar, de forma</p><p>inequívoca, a que pretende que seja classificada, deve ser vista e classificada apenas a que</p><p>se encontra em primeiro lugar, na folha de resposta.</p><p>3. A cotação a atribuir a cada item deverá ser sempre um número inteiro, não negativo, de</p><p>pontos.</p><p>4. Num item em que a respectiva resolução exija cálculos e/ou justificações, a cotação a</p><p>atribuir deve estar de acordo com o seguinte critério:</p><p>• Se o examinando se limitar a apresentar o resultado final, a cotação deve ser de 0 (zero)</p><p>pontos.</p><p>• Se o examinando não se limitar a apresentar o resultado final, a cotação deve ser a soma</p><p>algébrica das cotações atribuídas a cada etapa, de acordo com o disposto nos pontos 6,</p><p>7, 8, 9 e 10 destes critérios gerais, e das penalizações previstas nos pontos 11 e 12</p><p>destes critérios gerais. Se a soma for negativa, a cotação a atribuir é de 0 (zero) pontos.</p><p>5. Alguns itens da prova podem ser correctamente resolvidos por mais do que um processo.</p><p>Sempre que o examinando utilizar um processo de resolução não contemplado nos critérios</p><p>específicos, caberá ao professor classificador adoptar um critério de distribuição da cotação</p><p>que julgue adequado e utilizá-lo em situações idênticas. Salienta-se que deve ser aceite</p><p>qualquer processo cientificamente correcto, mesmo que envolva conhecimentos não</p><p>contemplados no programa da disciplina.</p><p>6. A cotação de cada item está subdividida pelas etapas que o examinando deve percorrer</p><p>para o resolver.</p><p>6.1. Em cada etapa, a cotação indicada é a máxima a atribuir.</p><p>6.2. O classificador não pode subdividir, em cotações parcelares, a cotação atribuída a</p><p>cada etapa.</p><p>Caso uma etapa envolva um único passo, testando apenas o conhecimento de um só</p><p>conceito ou propriedade, e a sua resolução não esteja completamente correcta, deve</p><p>ser atribuída a cotação de 0 (zero) pontos.</p><p>Caso uma etapa envolva mais do que um passo (por exemplo, a resolução de uma</p><p>equação, a obtenção de uma expressão em função de uma variável, etc.) e a sua</p><p>resolução esteja incompleta, ou contenha incorrecções, a cotação a atribuir deve estar</p><p>de acordo com o grau de incompletude e/ou a gravidade dos erros cometidos. Por</p><p>exemplo:</p><p>• erros de contas ocasionais devem ser penalizados em um ponto;</p><p>• erros que revelem desconhecimento de conceitos, regras ou propriedades devem</p><p>ser penalizados em, pelo menos, metade da cotação da etapa;</p><p>• transposições erradas de dados do enunciado devem ser penalizadas em um ponto,</p><p>desde que o grau de dificuldade da etapa não diminua;</p><p>• transposições erradas de dados do enunciado devem ser penalizadas em, pelo</p><p>menos, metade da cotação da etapa, caso o grau de dificuldade da etapa diminua.</p><p>V.S.F.F.735/C/3</p><p>6.3. Nas etapas cuja cotação se encontra discriminada por</p>
seleccionado, ao acaso, de entre o
grupo de turistas inquiridos».
1.º Processo:
Representar graficamente o problema .................................................................. 10 pontos
Respeitar a forma da curva de Gauss ............................................ 2 pontos
Assinalar, no gráfico, o valor médio da distribuição ........................ 2 pontos
Assinalar, no gráfico, a região correspondente a P(X > 2900) ... 3 pontos
Assinalar, no gráfico, a região correspondente a P(X < 2000) ... 3 pontos
Concluir que é mais provável que o valor do vencimento mensal individual seja
inferior a 2000 euros .......................................................................................... 5 pontos
P N−� �
Prova 735 • Página C/12/ 15
2.º Processo:
Determinar, com o auxílio da calculadora, o valor aproximado
de P(X > 2900) (0,05) (ver nota 1) ............................................................... 5 pontos
Determinar, com o auxílio da calculadora, o valor aproximado
de P(X < 2000) (0,09) (ver nota 2) ................................................................. 5 pontos
Concluir que é mais provável que o valor do vencimento mensal individual seja
inferior a 2000 euros ............................................................................................ 5 pontos
3.º Processo:
Escrever (ou equivalente) ............................... 4 pontos
Escrever 
(ou equivalente) ..................................................................................................... 5 pontos
Referir que (ou equivalente) ................................. 1 pontos
Concluir que é mais provável que o valor do vencimento mensal individual seja
inferior a 2000 euros .......................................................................................... 5 pontos
4.º Processo:
Escrever (ou equivalente) .............................. 4 pontos
Escrever 
(ou equivalente) .................................................................................................... 5 pontos
Referir que (ou equivalente) .......................................... 1 pontos
Concluir que é mais provável que o valor do vencimento mensal individual seja
inferior a 2000 euros .......................................................................................... 5 pontos
Notas:
1. Se o examinando apresentar o valor 0,04 ou um valor correctamente aproximado de 
P(X > 2900), com mais do que duas casas decimais, a pontuação a atribuir nesta etapa deverá
ser desvalorizada em 1 ponto. 
2. Se o examinando apresentar o valor 0,08 ou um valor correctamente aproximado de
P(X < 2000), com mais do que duas casas decimais, a pontuação a atribuir nesta etapa deverá
ser desvalorizada em 1 ponto. 
( )P X1900≤ < 2000
( ) ( ) ( )P X P X P X< 2000 = < 1900 + 1900 ≤ < 2000
( ) ( )P X P X> 2900 = < 1900
( )P X2800 < ≤ 2900 > 0
( ) ( ) ( )P X P X P X> 2800 = 2800 < ≤ 2900 + > 2900
( ) ( )P X P X< 2000 = > 2800
Prova 735 • Página C/13/ 15
GRUPO IV
......................................................................................................................................................... 20 pontos
Apresenta-se, a seguir, um exemplo de resposta:
«A opção A) não está correcta, pois a expressão apresentada para M define uma função
decrescente, o que significaria que o número de milhafres estava a diminuir em vez de
aumentar. [ver nota 1]
A opção B) não está correcta, devido ao facto de a expressão apresentada para E definir
uma função que nunca toma valores inferiores a 0,8 , porque 7 × 1,03 −x é maior que
zero para qualquer valor de x . Esta propriedade implicaria que o número de esquilos nunca
fosse inferior a oitenta efectivos, contrariando a informação de que o número de esquilos da
Reserva é, actualmente, inferior a cinquenta. [ver nota 2]
A opção C) também não está correcta, devido à expressão apresentada para M, pois,
embora a expressão corresponda a uma função crescente, verificar-se-ia, por exemplo, que
M(100) ≈ 5,05. Na situação descrita, isto significaria que, passados 100 meses desde o
início do ano de 2000, ou seja, no início de Maio de 2008, o número de milhafres já seria
cerca de 505 e, portanto, mais do que 500, o que contrariaria a informação dada. 
[ver nota 3]»
Notas:
1. O examinando também pode justificar a incorrecção desta opção pelo facto de a expressão
apresentada para E definir uma função crescente, o que significaria que a população de esquilos
estaria a aumentar, e não a diminuir, conforme é referido no enunciado.
2. O examinando também pode justificar a incorrecção desta opção referindo-se à assímptota
horizontal de equação y = 0,8 , relativa ao gráfico da função definida pela expressão apresentada
para E.
3. O examinando também pode justificar a incorrecção desta opção sem apresentar um exemplo,
desde que, na sua resposta, esteja explícito que a expressão apresentada para M levaria à
conclusão de que, actualmente, o número de milhafres já teria excedido as cinco centenas. 
No caso de o examinando apresentar um exemplo, tal como o referido, M (100) ≈ 5,05 , deve
interpretar os valores, considerando a situação descrita.
Tal como o exemplo de resposta ilustra, a composição deve abordar os seguintes tópicos:
• apresentação de uma razão cientificamente válida que justifique, inequivocamente, que a
opção A) está incorrecta;
• apresentação de uma razão cientificamente válida que justifique, inequivocamente, que a
opção B) está incorrecta;
• apresentação de uma razão cientificamente válida que justifique, inequivocamente, que a
opção C) está incorrecta.
Prova 735 • Página C/14/ 15
Na tabela seguinte, indica-se como deve ser classificado este item, de acordo com os níveis
de desempenho no domínio da comunicação escrita em língua portuguesa, descritos nos
critérios gerais, e com os níveis de desempenho no domínio específico da disciplina.
** Descritores apresentados nos critérios gerais.
** Apenas podem ser atribuídas classificações correspondentes a um dos valores constantes do
quadro. Não há lugar a classificações intermédias.
No caso de a resposta não atingir o nível 1 de desempenho no domínio específico da disciplina, a
classificação a atribuir é zero pontos. Neste caso, não é classificado o desempenho no domínio da
comunicação escrita em língua portuguesa.
Descritores do nível de desempenho no domínio
da comunicação escrita em língua portuguesa
Descritores do nível de desempenho 
no domínio específico da disciplina
Níveis*
1 2 3
N
ív
e
is
**
3 A composição contempla correctamente os três tópicos. 18 19 20
2 A composição contempla correctamente apenas dois tópicos. 12 13 14
1 A composição contempla correctamente apenas um tópico. 6 7 8
Prova 735 • Página C/15/ 15
Proposta de Resolução
 
1. Lado do quadrado: 12 dm
Figura 1: raio do círculo = 6 dm
Área do círculo = 
Figura 2: raio de cada círculo = 3 dm
Soma das áreas dos círculos = 
2. O número de círculos é sempre o quadrado do n.º de ordem da figura. O número de círculos do 
10º painel é, portanto, 10
3. 1ª tela: 4 � 12 dm = 48
2ª tela: 6 � 12 dm = 72
3ª tela: 8 � 12 dm = 96
.... 
A quantidade de fio em cada tela, em dm, é uma progressão aritmética de primeiro termo 48 e 
razão 2 � 12 � 24. 
Então, a 10ª tela precisa de 
No total, as 10 telas precisam de 
O artista terá de aplicar 156 
 
 
1. Volume do cilindro = Área da base 
Volume de cada cone = Volume do cilindro
Como foram enchidos dois cones, resta um terço do volume de água
Volume de água restante no cilindro = 
Mas o volume de água restante no cilindro é um cilindro com a mesma base e altura 
Então 
 
roposta de Resolução da Prova de Matemática B
(21 de Junho de 2010) 
GRUPO I 
m 
raio do círculo = 6 dm 
Área do círculo = � � 6	 = 36 � 
raio de cada círculo = 3 dm 
Soma das áreas dos círculos = 4 � � � 3	 = 36 � 
O número de círculos é sempre o quadrado do n.º de ordem da figura. O número de círculos do 
10º painel é, portanto, 102